Anda di halaman 1dari 124

Estadstica Empresarial II 21

ESTADSTICA EMPRESARIAL II
PROBLEMAS DE EXAMEN RESUELTOS
1) Supongamos que nos dicen que la v. a. X= Renta anual de las personas en Valladolid (en
millones de ptas.), sigue una distribucin U[0.5, 20].
Comentar esta afirmacin. Crees que puede ser cierta?
SOLUCIN
Existira la misma probabilidad de superar 10.25
que de no hacerlo, de estar entre 0.5 y 1 que de
estar entre 19.5 y 20 (ser muy rico o en absoluto).
Luego esta afirmacin no puede ser cierta.
2) Sea X una v. a. con distribucin binomial, siendo n = 1 000, y su media 500. Calcular la
probabilidad de que X sea menor que 480, comentando los pasos.
SOLUCIN
Puesto que X B(1 000,p) su E(X) = np = 500 p = 0.5
luego X B(1 000,0.5), y la probabilidad pedida sera:
p(X < 480) =
1 000
k
0.5 0.5
k 1 000 k
k 0
479
|
\

|
.
|

=

difcil de obtener.
Entonces, como toda variable binomial se puede escribir como suma de variables aleatorias
independientes idnticamente distribuidas de la siguiente forma:
X = X
i
i 1
n
=

tal que X
i
=
1 exito p
0 fracaso q

con X
i
v. a. i. i. d.
podemos aplicar el Teorema Central del Lmite y tendremos:
p(X < 480 ) =p
X np
npq

480 np
npq

<

|
\

|
.
|
=p
X 500
250

480 - 500
250

<
|
\

|
.
| (-1.26) = 0.1038
Aplicando el Teorema Central del Lmite
0.5 20 10.25 X
f
X
22 Problemas de examen resueltos
3) Se desea estudiar la relacin entre el precio (X) y la cantidad vendida (Y) de un
determinado producto. Para de diferentes establecimientos que venden dicho producto, se ha
obtenido la siguiente tabla de la ley de probabilidad conjunta de ambas variables:
Y
X 1 2 3
20 0 0.1 0.4
30 0 0.2 0
40 0.3 0 0
a) Calcular el valor del coeficiente de correlacin. Comentar el resultado obtenido en relacin
con las variables.
b) Sin realizar ningn clculo, si nos dicen que en una determinada tienda, el precio del
producto es 38, cul sera aproximadamente el nmero de unidades vendida?
c) Calcular p(XY > 50).
SOLUCIN
Y
X 1 2 3 p(X = x)
20 0 0.1 0.4 0.5
30 0 0.2 0 0.2
40 0.3 0 0 0.3
p(Y = y) 0.3 0.3 0.4 1
a) Para calcular el coeficiente de correlacin tenemos que calcular la covarianza entre X e Y y las
desviaciones tpicas de ambas variables, y para ello necesitamos obtener las esperanzas siguientes:
E(X) = (20x0.5) + (30x0.2) + (40x0.3) = 28
E(X
2
) = (20
2
x0.5) + (30
2
x0.2) + (40
2
x0.3) = 860
E(Y) = (1x0.3) + (2x0.3) + (3x0.4) = 2.1
E(Y
2
) = (1
2
x0.3) + (2
2
x0.3) + (3
2
x0.4) = 5.1
E(XY) = (20x1x0) + (30x1x0) + (40x1x0.3) + (20x2x0.1) + (30x2x0.2) + (40x2x0) +
+ (20x3x0.4) + (30x3x0) + (40x3x0) = 52
Var(X) = E(X
2
) - E
2
(X) = 860 - 28
2
= 76
Var(Y) = E(Y
2
) - E
2
(Y) = 5.1 - 2.1
2
= 0.69
Cov (X,Y) = E(XY) - E(X)E(Y) = 52 - (28x2.1) = -6.8

X,Y

Cov (X, Y)
Var(X) Var(Y)
= =
6.8
76 x 0.69
= - 0.9390259
Estadstica Empresarial II 23
La correlacin negativa nos indica que la relacin entre ambas variables es inversa, es decir, cuando
aumenta una variable la otra disminuye y al revs, adems como esta muy prxima a -1, nos dice
tambin que el grado de relacin o de dependencia entre ambas variables es muy alto.
b) Como hemos dicho en el apartado anterior, a medida que aumenta la X, la Y disminuye por que
el coeficiente de correlacin es negativo, luego si el precio es 38, un valor cercano a 40, la cantidad
vendida lo ser a 1 unidad.
c) Obtengamos la distribucin de XY
p(XY = 40) = p(X = 20, Y = 2) + p(X = 40, Y = 1) = 0.1 + 0.3 = 0.4
p(XY = 60) = p(X = 20, Y = 3) + p(X = 30, Y = 2) = 0.4 + 0.2 = 0.6
luego:
XY p(XY = k)
40 0.4
60 0.6
con lo que
p(XY > 50) = p(XY = 60) = 0.6.
4) El nmero de unidades vendidas al da de tres productos A, B y C son v. a. que siguen
respectivamente distribuciones N(100, 10), N(200, 15) y N(150, 10). Si los precios de dichos
productos son 150, 200 y 100 ptas. se pide:
a) Calcular la distribucin de la v. a. ingresos diarios (I).
b) Calcular p( I > 61 580) y el valor de k tal que p( I > k) = 0.975.
c) Calcular la moda de la v. a. ingresos diarios (No se necesita realizar ningn clculo).
d) Calcular la probabilidad de que los ingresos mensuales (20 das) estn entre 1 375 000 y
1 383 000.
SOLUCIN
Definimos las siguientes variables:
A = Nmero de unidades vendidas del producto A N(100,10)
B = Nmero de unidades vendidas del producto B N(200,15)
C = Nmero de unidades vendidas del producto C N(150,10)
a) Sea la v. a. I = Ingresos diarios = 150 A + 200 B + 100 C
E(I) = 150E(A) + 200E(B) + 100E(C) = (150x100) + (200x200) + (100x150) = 70 000
Var(I) = 150
2
Var(A) + 200
2
Var(B) + 100
2
Var(C) = (150
2
x10
2
) + (200
2
x15
2
) + (100
2
x10
2
) =
= 12 250 000

I
= 3 500
24 Problemas de examen resueltos
luego
I N(70 000,3 500)
b) p( I > 61 580) = p
I 70 000
3 500

61 580 70 000
3 500

>

|
\

|
.
| = 1 - (-2.40) = 1 - 0.0082 = 0.9918
0.975 = p( I > k) = p
I 70 000
3 500

k 70 000
3 500
1-
k 70 000
3 500

>

|
\

|
.
| =

|
\

|
.
|

k 70 000
3 500

|
\

|
.
| = 0.025

k 70 000
3 500

= -1.96 k = 63 140
c) I sigue una distribucin unimodal y simtrica como es la distribucin normal, en la que la moda
coincide con la media, luego
Moda de I = Media de I = 70 000
d) Sean I
1
, I
2
, I
3
, , I
20
v. a. i. i. d. con una distribucin N(70 000, 3 500) y sea
IM = Ingreso mensual = I
1
+ I
2
+ I
3
+ + I
20
E(IM) = E(I
1
+ + I
20
) = 20E(I
i
) = 14 x 10
5
Var(IM) = Var(I
1
+ + I
20
) = 20Var(I
i
) = 245 x 10
6
Con lo que
IM N(14x10
5
,10
3
x 245 )
p( 1 375 000 < IM < 1 383 000) =
= p
1 375 000 1 400 000
10 245

IM 1 400 000
10 245

1 383 000 1 400 000
10 245

<

<
|
\

|
.
| =
= p(-1.59 < Z < -1.08) = (-1.08) - (-1.59) = 0.1401 - 0.0559 = 0.0842
5) Sea (X, Y) una v. a. bidimensional con funcin de densidad conjunta:
f (x, y)
k 1 y x 1
0 en el resto
X,Y
=
< < <


Estadstica Empresarial II 25
a) Hallar el valor de k.
b) Comprobar si las v. a. X e Y son o no independientes.
c) Calcular E(3X
2
+ Y).
d) Calcular la funcin de distribucin de la v. a. X.
e) Sea Z = 5 - Y/2. Calcular la funcin de densidad de Z.
f) Calcular la funcin de densidad de X condicionada a que Y = 1/4 y calcular
( ).
1/4 Y
1/2 X
p
=

SOLUCIN
a) k dy dx
y 1
y x
x 1
x 1
=
=
=
=

(
(
= ( | k y dx
y=-1
y=x
x 1
x 1
=
=

=
= k (x 1) dx
x 1
x 1
+
=
=

= k
( x 1)
2
2
x 1
x=1
+

(
=
=
2k = 1
k = 1/2
b) f (x)
1
2
dy
X
y= 1
y=x
=

=
y
2
1
y=x

(
= y
=
x 1
2
+
f (x)
x 1
2
1 x 1
0 en el resto
X
=
+
< <

f (y)
1
2
dx
Y
x=y
x=1
=

=
x
2
x=y
x=1

(
=
1 y
2

f (y)
1 - y
2
1 y 1
0 en el resto
Y
=
< <

Como el producto de las funciones de densidad marginales no es la funcin de densidad conjunta,


las variables X e Y no son independientes.
c) E(X
2
) = x
x 1
2
dx
2
x
x=1
+
|
\

|
.
|
=

1
=
x x
2
dx
3 2
x
x=1
+ |
\

|
.
|
=

1
=
x
4

x
2
4 3
1
x=1
+
|
\

|
.
|

(
(
(
(
(
=
3
x
= 1/3
1 -1
26 Problemas de examen resueltos
E(Y) = y
1 y
2
dy
y=

|
\

|
.
|

1
1 y
=
y y
2
dy
2
y=
y 1
|
\

|
.
|

1
=
x
2

y
3
2
2 3
1
x=1

|
\

|
.
|

(
(
(
(
= x
= -1/3
E(3X
2
+Y) = 3 E(X
2
) + E(Y) = (3 x 1/3) - 1/3 = 2/3
d) Al ser
t 1
2
dt
t=
t x
+

1
=
( t 1)
4
2
1
t=x
+

(
= t
=
(x 1)
4
2
+
la funcin de distribucin de la v. a. X es:
F (x)
X
= p(X x) =
0 x 1
(x 1)
4
1 x 1
1 x 1
2

+

e) Sea la v. a. Z = 5 -
Y
2
, para calcular la funcin de densidad de Z empezaremos viendo que
relacin hay entre las funciones de distribucin de Z y de Y, y luego derivando tendremos la
relacin entre las respectivas funciones de densidad.
F (z)
Z
= p(Z z) = p(5 -
Y
2
z) = p(-
Y
2
z - 5) = p(Y 10 - 2z) = 1 - F (10 2z)
Y

f (z) f (10 2z) ( 2) 2 f (10 2z) 2 ( z
9
2
) 2z 9
Z Y Y
= = = =
Adems, como -1 < y < 1 -1 < 10 - 2z < 1 -11 < -2z < -9 9/2 < z < 11/2
(Tambin se puede hacer -1 < y < 1 1 > - y > -1 1/2 > -y/2 > -1/2 11/2 > z > 9/2)
tenemos finalmente que la funcin de densidad de Z es:
f (z)
2z 9
9
2
z
11
2
0 en el resto
Z
=
< <

f) Nos piden calcular f (x),


X
Y=1 4 /
lo haremos en general y luego particularizaremos en Y = 1/4.
Estadstica Empresarial II 27
f (x)
X
Y y =
=
f (x, y)
f (y)
X,Y
Y
=
1/ 2
(1 y) / 2
=
1
1 y
luego
f (x)
X
Y y =
=
1
1 y
y x 1
0 en el resto

< <

f (x)
X
Y=
=
1 4 /
1
1 1
1
x 1
0 en el resto

< <

/ 4 4
con lo que
( )
p
X 1/ 2
Y 1/ 4
>
=
=
4
3
dx
x=1/2
x=

1
=
4x
3
x=1/2
x=1

(
= 2/3.
6) Sabemos que los ingresos de una empresa son una v. a. X cuya funcin de densidad es una
de estas:
a) Con cual de las dos es mayor el ingreso medio?
b) Cual de las dos tiene mayor dispersin?
c) En cual de ellas la p(350 < X < 500) ser mayor? Razona la respuesta.
SOLUCIN
a) A la vista de los grficos ambas funciones de densidad tienen la misma esperanza, 500.
b) La segunda tiene los valores ms dispersos que la primera que les tiene ms agrupados.
La mas favorable para la empresa ser la primera por tener mayor seguridad en los ingresos (menos
riesgo).
c) La p(350 < X < 500) ser mayor en la primera, por haber menor dispersin.
7) Sea (X, Y) una v. a. bidimensional con funcin de densidad conjunta:
f (x, y)
x
2
0 x 2, 0 y 1
0 en el resto
X,Y
=
< < < <

Funcin de densidad
250 350 450 550 650 750
0
2
4
6
8
(X0,001)
Funcin de densidad
250 350 450 550 650 750
0
1
2
3
4
(X0,001)
28 Problemas de examen resueltos
a) Calcular las marginales de X e Y.
b) Calcular la p(X >1).
c) Calcular Var(X+Y) y Var(X-Y).
SOLUCIN
a) Funcin de densidad marginal de X:
f (x) =
1
2
x dy
X
y=0
y=1

=
x
2
y
x
2
y=1

(
=
= y 0
f (x)
x
2
x 2
0 en el resto
X
=
< <

0
Funcin de densidad marginal de Y:
f (y)
1
2
x dx
Y
x=0
x=2
=

=
x
4
2
x=0
x=2

(
= 1 f (y)
1 y 1
0 en el resto
Y
=
< <

0
X e Y son independientes, pues la funcin de densidad conjunta coincide con el producto de las
funciones de densidad marginales.
b) p(X >1) = f (x) dx
X
x=1
x=

=
1
2
x dx
x=1
x=2

=
x
4
2
x=1
x=2

(
= 1-
1
4
= 3/4.
c) Por ser independientes,
Var(X+Y) = Var(X) + Var(Y)
Var(X-Y) = Var(X) + (-1)
2
Var(Y) = Var(X) + Var(Y)
E(X) = x
1
2
x dx
x=0
x=2

=
x
6
3
x=0
x=2

(
= 4 3 /
E(X
2
) = x
1
2
x dx
2
x=0
x=2

=
x
8
4
x=0
x=2

(
= 2
Var(X) = E(X
2
) - E
2
(X) = 2 - (4/3)
2
= 2/9
E(Y) = y 1 dy
y=0
y=1

=
y
2
2
y=0
y=1

(
= 1 2 /
E(Y
2
) = y 1 dy
y=0
y=1
2

=
y
3
3
y=0
y=1

(
= 1 3 /
Var(Y) = E(Y
2
) - E
2
(Y) = 1/3 - (1/2)
2
= 1/12
Estadstica Empresarial II 29
luego
Var(X+Y) = Var(X-Y) = Var(X) + Var(Y) = 2/9 + 1/12 = 11/36
8) Un estudio de Arquitectura tiene tres tipos de ingresos mensuales independientes, por
Tarifa de Edificacin, de Urbanismo y por Peritaciones, que siguen distribuciones normales
N(14, 2), N(12, 4) y N(3, 5 ), respectivamente, medidas en millones de ptas. Se pide:
a) Calcular la probabilidad de que los ingresos totales mensuales sean mayores de 30millones.
b) Calcular la probabilidad de no cubrir gastos si estos se estiman en 14.5 millones mensuales.
c) Si consideramos 12 meses, cul es la probabilidad de que al menos un mes no se cubran
gastos?
SOLUCIN
Consideremos las variables siguientes:
X = Ingresos mensuales Tarifa de Edificacin N(14, 2)
Y = Ingresos mensuales Urbanismo N(12, 4)
Z = Ingresos mensuales Peritaciones N(3, 5 )
a) Sea I = Ingresos totales = X + Y + Z N(29, 5) ya que:
E(I) = E(X + Y + Z) = E(X) + E(Y) + E(Z) = 14 + 12 + 3 = 29
Var(I) = Var (X + Y + Z) = Var(X) + Var(Y) + Var(Z) = 2
2
+ 4
2
+ ( 5 )
2
= 25
con lo cual la probabilidad pedida es:
p(I > 30) = p
I 29
5

30 29
5

>

|
\

|
.
| = p(Z > 0.2) = 1 - (0.2) = 1 - 0.5793 = 0.4207.
b) p(I < 14.5) =p
I 29
5

14.5 29
5

<

|
\

|
.
| = p(Z < -2.9) = (-2.9) = 0.0019
c) Llamamos T
i
=
1 si en ese mes no se cubren gastos p = 0.0019
0 si en ese mes si se cubren gastos q = 0.9981

Entonces S T
i
i 1
12
=
=

B(12, 0.0019)
p(S 1) = 1 - p(S = 0) = 1 - (0.9981)
12
= 0.0225632.
9) Una industria conservera obtiene al da, por termino medio, 0.5 toneladas de tomate en
conserva, con una desviacin tpica de 30 kilogramos.
30 Problemas de examen resueltos
a) Si la industria conservera vende su producto al precio de 100 pesetas el kilogramo y los
costes de elaboracin, etc., son de 20 pesetas por kilogramo, ms unos gastos fijos de 10 000
pesetas, calcular la probabilidad de que el beneficio en un trimestre (90 das) sea mayor que
4 555 000 pesetas. Comentar el significado de dicha probabilidad.
b) Cuantos das tardarn en obtener al menos 100 toneladas con una probabilidad del
97.5%?
SOLUCIN
a) Sea X
i
= Kilogramos de tomate en el da i-simo que obtiene la empresa. De esta variable
conocemos:
E(X
i
) = 500 Kg.
Xi
= 30 Kg.
Definimos B
i
= Beneficio del da i-simo = I
i
- C
i
= 100 X
i
- (20X
i
+ 10 000) = 80X
i
- 10 000
E(B
i
) = E(80X
i
- 10 000) = 80E(X
i
) - 10 000 = 80x500 - 10 000 = 30 000
Var(B
i
) = Var(80X
i
- 10 000) = 80
2
Var(X
i
) = 80
2
x 30
2
= 5760000
Consideremos la variable B = Beneficio en un trimestre = B
i
i 1
90
=

Calculemos cuanto valen la esperanza y la varianza de B.


E(B) = E( B
i
i 1
90
=

) = E(B
i
i 1
90
)
=

= 2 700 000
Var(B) = Var( B
i
i 1
90
=

) = Var (B
i
i 1
90
)
=

= 90 x 5 760 000
B
= 22 768.39915
Como B
1
, , B
90
son v. a. i. i. d. con esperanza y varianza finita, a la variable B le podemos aplicar
el Teorema Central del Lmite.
p( B > 4 555 000) = p
B 2 700 00
22 768.39915

4 555 000 2 700 00
22 768.39915

>

|
\

|
.
| 1 - (81.4725) = 0
Por el Teorema Central del Lmite
b) Sean X
1
, , X
n
v. a. i. i. d., y sea S X
i
i 1
n
=
=

entonces
E(S) = 500n Var(B) = 900n
Hay que calcular un valor de n tal que p X 100 000 0.975
i
i 1
n
=


|
\

|
.
| =
Estadstica Empresarial II 31
Como no conocemos la distribucin de S y es suma de variables aleatorias independientes e
idnticamente distribuidas con esperanza y varianza finita, podemos aplicar el Teorema Central del
Lmite para obtener el valor de n.
p(S 100 000) = p
S 500n
30 n

100 000 500n
30 n

|
\

|
.
|
1 -
100 000 - 500n
30 n
|
\

|
.
| = 0.975

100 000 - 500n
30 n
= -1.96 n 202.
10) Sea X el tiempo que tarda un taller en detectar una avera en un vehculo, desde el
instante en el que entra, dado en horas. Sea Y el tiempo que transcurre desde que el vehculo
entra hasta que finalmente es reparado (horas). Una vez detectado el problema, comienza la
reparacin.
Se sabe, en teora, que ambas variables se relacionan probabilsticamente segn la funcin
de densidad conjunta siguiente:
( ) f x, y
1
2
x e
X Y
2 y
,
=

, 0 < x < y < , y nula en el resto.
a) Interpretar las restricciones de f
X,Y
(x,y), segn el contenido del problema.
b) Deducir qu distribuciones siguen ambos tiempos. Son independientes?
c) Si Z es el tiempo que se emplea desde que se localiza el fallo hasta que se arregla, calcular,
por trmino medio, cunto debemos esperar. Calcular tambin la varianza de Z.
d) Si el tiempo que permanece un vehculo en el taller, desde que entra hasta que sale
reparado, son 3 horas, cunto tiempo es de esperar que se tarde en averiguar su avera?
(Nota: y e dy k!
k y
y 0
y

=
=

= )
SOLUCIN
a) Como ambas variables miden tiempos, toman valores positivos. Adems el tiempo que tarda un
taller en detectar una avera en un vehculo, desde el instante en el que entra, X es siempre menor
que el tiempo que transcurre desde que el vehculo entra hasta que finalmente es reparado, Y. El
valor infinito le toman cuando no hay posibilidad de reparacin.
b) Con el fin de conocer que distribuciones siguen tanto X como Y, calcularemos sus densidades
marginales
Funcin de densidad marginal de X:
( | f (x)
1
2
x e dy
1
2
x e
1
2
x e
X
2 y
y x
y
2 y
y x
y
2 x
= = =

=
=

=
=

0 < x <
32 Problemas de examen resueltos
f (x)
1
2
x e x 0
0 en el resto
X
2 x
=
>

X (1,3) pues
f (x)
a
(p)
x e x 0
0 resto


1
2!
x e x 0
0 resto
X
p 1 ax
3 1 x
=
>
=

>

Funcin de densidad marginal de Y:


f (y)
1
2
x e dy e
1
6
x
1
6
y e
Y
2 y
x
x=y
y 3
x=0
x=y
3
= =
|
\

(
=

0
y
0 < y <
f (y)
1
6
y e y 0
0 en el resto
Y
3 y
=
>

Y (1,4) ya que
f (y)
a
(p)
y e y 0
0 resto


1
3!
y e y 0
0 resto
Y
p 1 ay
4 1 y
=
>
=

>

Como en general f
X,Y
(x,y) f
X
(x) f
Y
(y) las variables X e Y son dependientes.
c) Sea Z = Y - X
E(Z) = E(Y) - E(X) =
4
1

3
1
= 1 (la esperanza de una (a,p) es
p
a
)
E(YX) = yx
1
2
x e dx dy
1
2
y e
x
4
dy
2 y
x 0
x y
y 0
y
y
4
x 0
x y
y 0
y

=
=
=
=

=
=
=
=

(
(
=
|
\

(
=
1
8
y e dy
5 y
y 0
y

=
=

=

1
8
y e dy
5!
8
15
5 y
y 0
y
= = =

=
=

usando la nota del enunciado


Estadstica Empresarial II 33
Cov(Y,X) = E(YX) - E(Y)E(X) = 15 - 4x3 = 3
Var(Z) = Var(Y) + Var(X) - 2Cov(Y,X) =
4
1

3
1
2 x 3 1
2 2
+ = (la varianza de una (a,p) es
p
a
2
)
d) f (x)
f (x, y)
f (y)

1
2
x e
1
6
y e

3 x
y

X/Y y
X,Y
Y
2 y
3 y
2
3 =

= = = 0 < x < y siempre que y > 0


f (x)
3 x
3
=
x
X/Y 3
2
3
2
=
=
9
0 < x < 3
E(X/ Y 3) x
x
9
dx
x
36

9
4
2.25
2
x 0
x 3 4
x 0
x 3
= = =
(

(
= =
=
=
=
=

11) Sea la v. a. X = Ventas diarias de harina (kilogramos), con distribucin N(3 000, 200).
a) Si suponemos que otra fbrica vende diariamente 1 000 kg. de harina, ms el doble de lo
que vende la primera, cul ser la esperanza y la desviacin tpica de las ventas de la
segunda fbrica?
b) Supongamos que la primera fbrica vende su producto a 50 ptas. el kilogramo. Cul ser
la distribucin de la v. a. I = Ingresos mensuales de la primera fbrica, considerando que
vende 30 das al mes? (Representar grficamente, de forma aproximada, la distribucin).
c) Calcular y representar la probabilidad de que los ingresos mensuales estn comprendidos
entre 4 420 000 y 4 580 000 ptas.
SOLUCIN
Sea X = Ventas diarias de harina (en kg.) N(3 000,200)
a) Definimos Y = 2X + 1 000
E(Y) = E(2X + 1 000) = 2E(X) + 1000 = 2x3 000 + 1 000 = 7 000
Var(Y) = Var(2X + 1 000) = 4Var(X) = 4x200
2
= 160 000
Y
= 400
b) Para cada da: I
i
= 50X
i
N(150 000,10 000)
En un mes I = Ingresos mensuales de la 1 fbrica = I
1
+ + I
30
N(4 500 000, 30 10 000)
34 Problemas de examen resueltos
c) p(4 420 000 < I < 4 580 000) = p
4 420 000 4 500 000
30 1 000
Z
4 580 000 4 500 000
30 1 000

< <
|
\

|
.
| =
= p(-1.46 < Z < 1.46) = (1.46) - (-1.46) = 0.9279 - 0.0721 = 0.8558.
12) Una empresa de transporte tiene tres tipos de camiones. El nmero medio de litros de
combustible que repostan los camiones de tipo A es de 300 litros, con una desviacin de 40; los
de tipo B, 250 de media y desviacin de 30; los de tipo C, 175 y 25.
a) Si en una semana repostan 51 camiones de tipo A, 42 de tipo B y 40 de tipo C, cul es la
probabilidad de que el nmero total de litros repostado est entre 32 500 y 33 500 litros?
b) Si la empresa piensa destinar al consumo de combustible para los camiones de tipo A
durante otra semana un milln quinientas mil pesetas, cul es el nmero mximo de
camiones de tipo A que podrn repostar, con una probabilidad del 95%, sabiendo que esa
semana el combustible estar a 100 ptas./litro?
SOLUCIN
Definimos las variables: A = nmero de litros de combustible que repostan los camiones de tipo A
B = nmero de litros de combustible que repostan los camiones de tipo B
C = nmero de litros de combustible que repostan los camiones de tipo C
Desconocemos la distribucin de estas variables. Slo conocemos sus esperanzas y sus varianzas,
por lo que aplicaremos el Teorema Central del Lmite.
a) Consideremos la variable aleatoria
X = nmero total de litros repostados = A B C
i
i 1
51
i
i 1
42
i
i 1
40
= = =

+ +
E(X) = 51E(A) + 42E(B) + 40E(C) = 51x300 + 42x250 + 40x175 = 32 800
Var(X) = 51Var(A) + 42Var(B) + 40Var(C) = 51x40
2
+ 42x30
2
+ 40x25
2
= 144 400
luego
X
= 380
42 43 44 45 46 47 48
(X 100 000)
0
2
4
6
8
(X 0.000001)
Estadstica Empresarial II 35
p(32 500 < X < 33 500) p
32 500 32 800
380
Z
33 500 32 800
380

< <

|
\

|
.
| = p(-0.79 < Z < 1.84) =
Aplicando el Teorema Central del Lmite y tipificando
= (1.84) - (-0.79) = 0.9671 - 0.2148 = 0.7523.
b) Sea Y = 100 A
i
i 1
n
=

E(Y) = 100 n E(A) = 30 000n


Var(Y) = 100
2
n Var(A) = 16 000 000n
Y
4 000 n =
p(Y < 1 500 000) p Z
1 500 000 30 000 n
4 000 n
<

|
\

|
.
|
= 0.95
Aplicando el Teorema Central del Lmite y tipificando

1 500 000 30 000 n
4 000 n

= 1645 . 30 000n + 6 580 n -1 500 000 = 0


luego
n
6 580 6 580 4 x 3 000 x 1 500 000
60 000

6.9622515
7.1815848
2
=
+
=

con lo que
n
48.47294595
51.57516024
=

la solucin n = 51.57 la tenemos que desechar pues al sustituir su valor en la probabilidad de partida
no da 0.95, que es el valor pedido, sino que da una probabilidad de 0.05.
Por tanto el valor pedido, redondeando, es n = 48.
13) En una pequea empresa, consideramos las siguientes caractersticas:
X = Nmero de empleados
Y = Nmero de artculos fabricados diariamente
36 Problemas de examen resueltos
como variables aleatorias. La ley de probabilidad conjunta es la siguiente:
X \ Y 8 10 12
3 2/9 0 0
4 0 3/9 1/9
5 0 1/9 2/9
a) Son independientes X e Y?
b) Calcular
( )
p
X 4
Y 10
.

c) Obtener el valor de p(3X + 2 14).


d) Calcular al covarianza de X e Y.
e) Indicar, sin hacer ningn clculo, basndose nicamente en los valores de la tabla anterior,
cul es el valor aproximado del coeficiente de correlacin entre X e Y.
SOLUCIN
a) Para que dos variables sean independientes se tiene que verificar que:
f
XY
(x,y) = f
X
(x)f
Y
(y) x,y R
X \ Y 8 10 12 p(X = x)
3 2/9 0 0 2/9
4 0 3/9 1/9 4/9
5 0 1/9 2/9 3/9
p(Y = y) 2/9 4/9 3/9 1
Como f
XY
(3,8) = 2/9 2/9 x 2/9 = f
X
(3)f
Y
(8) X e Y son dependientes
b)
( )
( )
( )
p
X 4
Y 10

p X 4, Y 10
p Y 10

=
( ) ( ) ( ) ( )
( ) ( )

p X 4, Y 8 p X 4, Y 10 p X 5, Y 8 p X 5, Y
p Y 8 p Y 10
=
= = + = = + = = + = =
= + =
=
10
=
0 3 / 9 0 1/ 9
2 / 9 4 / 9

2
3

+ + +
+
=
c) p(3X + 2 14) = p(3X 12) = p(X 4) =
2
9

4
9

2
3
+ =
Estadstica Empresarial II 37
d) E(X) =3
2
9
4
4
9
5
3
9

37
9
+ + =
E(Y) =8
2
9
10
4
9
12
3
9

92
9
+ + =
E(XY) =3x8
2
9
3x10x0 3x12x0 +4x8x0 4x10
3
9
4x12
1
9
+ 5x8x0 5x10
1
9
5x12
2
9
+ + + + + + =
=
386
9

Cov(X,Y) = E(XY) - E(X)E(Y) =
386
9

37
9

92
9

70
81
0.86419753. = =
e) El coeficiente de correlacin vara entre -1 y 1. En este caso como la covarianza calculada en el
apartado anterior es positiva eso no quiere decir que el coeficiente de correlacin es positivo,
adems, por el primer apartado sabemos que es mayor que cero pues las variables son dependientes.
De la observacin de la tabla de probabilidades conjuntas, podemos deducir que tendr un valor
relativamente alto, puesto que a valores bajos de X, la Y toma tambin valores bajos y para valores
intermedios o altos de X, predominan los valores intermedios o altos de Y.
14) Sea (X,Y) una v. a. bidimensional, con funcin de densidad conjunta:
f (x, y)
2x e 0 x 1; y 0
0 en el resto
X Y
y
,
=
< < >

a) Las variables X e Y, son independientes?


b) Qu distribucin sigue cada una de ellas?
c) Calcular el coeficiente de correlacin de estas variables y comentar su significado.
d) Hallar el valor de Var(5X + 2Y).
SOLUCIN
a) Dos variables son independientes si se verifica que:
f
X,Y
(x,y) = f
X
(x)f
Y
(y) x,y R
Calculemos las marginales de las variables X e Y.
( | f (x) 2x e dy 2x e 2x
X
y
y 0
y
y
y 0
y
= = =

=
=

=
=

0 < x < 1
( | f (y) 2x e dx e x e
Y
y
x 0
x 1
-y 2
x 0
x 1
-y
= = =

=
=
=
=

y > 0
38 Problemas de examen resueltos
f
X,Y
(x,y) =
2y e x 0, 0 y 1
0 en el resto
x
> < <

= f
X
(x) f
Y
(y)
luego las variables X e Y son independientes.
b) X (2,1)
f (x)
1
(p, q)
x (1 x) 0 x 1
0 resto

2 x 0 x 1
0 resto
X
p 1 q 1
=
< <
=

< <

pues

1
(2,1)

(2 1)
(2) (1)

2!
1! 0!
2

=
+
= =


Y (1)
f (y)
a e y 0
0 resto

e y 0
0 resto
Y
ay y
=
>
=

>


c) Como X e Y son independientes, son incorreladas, luego
X,Y
= 0.
d) Como sabemos que distribucin sigue tanto X como Y, calculamos su varianza por la frmula
Var(X) =
pq
(p q 1)(p q)

2 x 1
(2 1 1)(2 1)

1
18
2 2
+ + +
=
+ + +
=
Var(Y) =
1
a

1
1
1
2 2
= =
Si no conocemos que distribucin sigue X ni Y tendramos que empezar calculando la esperanza y
la varianza de X y de Y.
E(X) = x 2x dx
2
3
x
2
3

x 0
x 1
3
x 0
x 1
=
=
=
=

=
(

(
=
E(Y) = ( | ( | y e dy - y e - e dy 0 e 1
y 0
y
y 0
y
y 0
y
y 0
y

=
=

=
=

=
=

=
=

= = =
y y y y
Estadstica Empresarial II 39
Obtendremos ahora las esperanzas de X
2
y de Y
2
, para luego ver cuanto valen las respectivas
varianzas.
E(X
2
) = x 2x dx
1
2
x
1
2
2
x 0
x 1
4
x 0
x 1
=
=
=
=

=
(

(
=
E(Y
2
) = ( | y e dy - y e - 2y e dy 0 + 2 y e dy 2E(Y) 2
2
y 0
y
2
y 0
y
y 0
y

=
=

=
=

=
=

=
=

= = = =
y y
y
y
y y
0
luego
Var(X) = E(X
2
) - E
2
(X) =
1
2

2
3

1
18

2

|
\

|
.
| =
Var(Y) = E(Y
2
) - E
2
(Y) = 2 - 1
2
= 1
con lo cual, aplicando las propiedades de la varianza tendremos:
Var(5X + 2Y) = 5
2
Var(X) + 2
2
Var(Y) = 25
1
18
4 x1 = 5.38. +
)
por ser X e Y independientes
15) Sea X una v. a. dada por la demanda diaria, en kilogramos, de un determinado producto,
que sigue una distribucin normal con media 300 y desviacin tpica 20.
Si se vende cada kilogramo a 50 pesetas y sabemos que los costes fijos diarios son 2 000
pesetas y los variables 10 pesetas por kilogramo vendido:
a) Deducir la distribucin del beneficio diario.
b) Obtener la probabilidad de que en un mes (30 das) el beneficio sea mayor que 304 000
pesetas.
c) Como cambiara el problema si no conocisemos la distribucin de la variable X, aunque
s su media y su varianza?
SOLUCIN
X = Demanda diaria (en kg.) N(300,20)
a) B = Beneficios = Ingresos - Costes = 50X - (10X + 2000) = 40X - 2 000
E(B) = E(40X - 2 000) = 40E(X) - 2000 = 40x300 - 2 000 = 10 000
Var(B) = Var(40X - 2 000) = 40
2
Var(X) = 40
2
x 202 = 640 000
B
= 800
luego
B N(10 000,800)
40 Problemas de examen resueltos
b) Llamando B
1
, B
2
, , B
30
, al beneficio de cada uno de los das de un mes, el beneficio mensual
(T) ser:
T = B
1
+ + B
30
E(T) = E(B
1
+ + B
30
) = E(B
1
) + + E(B
30
) = 30x10 000 = 300 000
Var(T) = Var(B
1
+ + B
30
) = Var(B
1
) + + Var(B
30
) = 30x640 000 = 19 200 000
Suponiendo que los beneficios diarios son independientes

T
= 4381.78046
p(T > 304000) = p Z
304 000 300 000
4381.78046
1 (0.91) 1 0.8186 0.1814. >

|
\

|
.
| = = =
Tipificando
c) Si no supiramos que distribucin sigue X, como si conocemos su media y su varianza,
podramos aplicar el Teorema Central del Lmite, ya que la variable T = Beneficio mensual es una
suma de variable aleatorias independientes idnticamente distribuidas con media y varianza finita.
La nica diferencia que habra sera que como consecuencia de aplicar el Teorema Central del
Lmite la probabilidad calculada no sera exacta, si no que sera aproximada.
16) Sea X la v. a. proporcin de tornillos defectuosos en una mquina, con funcin de
densidad:
f (x)
3 (1 x) 0 x 1
0 en el resto
X
2
=
< <

a) Cul es la distribucin de la variable X?


b) Cul es la proporcin esperada de tornillos defectuosos?
c) Hallar la probabilidad de que la proporcin de tornillos defectuosos sea menor que 0.1.
d) Trabajando con 20 mquinas similares a la anterior, cul es la probabilidad de que, al
menos en dos de ellas, la proporcin de tornillos defectuosos sea menor que 0.1?
SOLUCIN
a) X (1,3)
f (x)
1
(p, q)
x (1 x) 0 x 1
0 resto

3 (1 - x) 0 x 1
0 resto
X
p 1 q 1
2
=
< <
=

< <

ya que
Estadstica Empresarial II 41

1
(1,3)

(1 3)
(1) (3)

3!
0! 2!
3

=
+
= =


b) Como X (1,3)
E(X) =
p
p q

1
1 3
0.25
+
=
+
=
Si no conociramos la distribucin de X la esperanza la podramos calcular:
E(X) = x 3(1- x) dx 3
x 2x
3
x
4
3
1
2

2
x 0
x 1
2 3 4
x 0
x 1
=
=
=
=

= +
|
\

(
= +
|
\

|
.
| =
2
2
3
1
4
025 .
c) p(X < 0.1) = ( )
( |
3(1 x) dx 1 x 0.729 1 0.271.
2
x 0
x 0.1
3
x 0
x 0.1
= = + =
=
=
=
=

d) Definimos el suceso E = xito = La proporcin de tornillos defectuosos es menor que 0.1, y


consideramos la variable aleatoria
Y = Nmero de mquinas en las que la proporcin de tornillos defectuosos es
menor que 0.1 de las veinte dadas
Como la variable Y cuenta el nmero de xitos en 20 ensayos de Bernoulli, sigue una distribucin
binomial de parmetros n = 20 y p = p(E) = p(X < 0.1) = 0.271.
Y B(20,0.271)
La probabilidad que nos piden es:
p(Y 2) = 1 - p(Y < 2) = 1 - [p(X = 0) + p(X = 1)] =
= 1
20
0
0.271 0.729
20
1
0.271 0.729
0 20 1 19

|
\

|
.
|
|
\

|
.
| = 0.9848.
17) En una biblioteca pblica han realizado un estudio sobre X = N de libros prestados a
una persona el misma da e Y = N de vdeos prestados a una persona el mismo da con los
siguientes resultados:
X \ Y 0 1 2
0 0 0.1 0
1 0.1 0.1 0.2
2 0.2 0.1 0
3 0.2 0 0
a) Calcular e interpretar el coeficiente de correlacin.
42 Problemas de examen resueltos
b) Obtener la distribucin del nmero de libros prestados para aquellos usuarios de la
biblioteca que no se llevan en prstamo ningn vdeo, cul es su esperanza?
c) Calcular la probabilidad de que el nmero de objetos prestados a un usuario el mismo da
sea mayor que dos.
d) Si en el prstamo de cada libro se tarda 20 segundos y en el de cada vdeo se tarda 30
segundos, cul es el tiempo medio que se tarda en atender a una persona? Y con qu
varianza?
SOLUCIN
a) Tenemos las variables X = N de libros prestados a una persona el misma da e Y = N de
vdeos prestados a una persona el mismo da. Para calcular el coeficiente de correlacin nos hacen
falta los siguientes clculos:
X \ Y 0 1 2 p(X = x)
0 0 0.1 0 0.1
1 0.1 0.1 0.2 0.4
2 0.2 0.1 0 0.3
3 0.2 0 0 0.2
p(Y = y) 0.5 0.3 0.2 1.0
E(X) = (0x0.1) + (1x0.4) + (2x0.3) + (3x0.2) = 1.6
E(X
2
) = (0
2
x0.1) + (1
2
x0.4) + (2
2
x0.3) + (3
2
x0.2) = 3.4
Var(X) = E(X
2
) - E
2
(X) = 3.4 - 1.6
2
= 0.84
E(Y) = (0x0.5) + (1x0.3) + (2x0.2) = 0.7
E(Y
2
) = (0
2
x0.5) + (1
2
x0.3) + (2
2
x0.2) = 1.1
Var(Y) = E(Y
2
) - E
2
(Y) = 1.1 - 0.7
2
= 0.61
E(XY) = (0x0x0) + (0x1x0.1) + (0x0x0) + (1x0x0.1) + (1x1x0.1) + (1x2x0.2) +
+ (2x0x0.2) + (2x1x0.1) (2x2x0) + (3x0x0.2) + (3x1x0) + (3x2x0) = 0.7
Cov(X,Y) = E(XY) - E(X)E(Y) = 0.7 - (1.6x0.7) = -0.42

Cov(X, Y)
Var(X) Var(Y)

0.42
0.84 x 0.61
0.586738694
X,Y
= =

=
Como el coeficiente de correlacin es negativo, quiere decir que cuanto mayor es el nmero de
libros que se lleva una persona en prstamo, menor es el nmero de vdeos que se lleva, y al revs, a
mayor cantidad de vdeos llevados en prstamo, menor es el nmero de libros que se lleva.
b) Empezaremos calculando la ley de probabilidad de X/Y=0.
Estadstica Empresarial II 43
f (0 / Y 0)
f (0,0)
f (0)

0
0.5
0
X/ Y 0
X,Y
Y
=
= = = =
f (1/ Y 0)
f (1,0)
f (0)

0.1
0.5
0.2
X/Y 0
X,Y
Y
=
= = = =
f (2 / Y 0)
f (2,0)
f (0)

0.2
0.5
0.4
X/Y 0
X,Y
Y
=
= = = =
f (3/ Y 0)
f (3,0)
f (0)

0.2
0.5
0.4
X/Y 0
X,Y
Y
=
= = = =
luego
X/Y=0 p(X/Y=0)
1 0.2
2 0.4
3 0.4
con lo cual su esperanza es:
E(X/Y=0) = (1x0.2) + (2x0.4) + (3x0.4) = 2.2
c) p(X + Y > 2) = p(X = 1, Y = 2) + p(X = 2, Y = 1) + p(X = 3, Y = 0) = 0.2 + 0.1 + 0.2 = 0.5
d) Definimos la variable T = 20X + 30Y. Tenemos que calcular su esperanza y su varianza, para
ello aplicaremos las propiedades de ambas.
E(T) = E(20X + 30Y) = 20E(X) + 30E(Y) = (20x1.6) + (30x0.7) = 53 segundos
Var(T) = Var(20X + 30Y) = 20
2
Var(X) + 30
2
Var(Y) + 2x20x30Cov(X,Y) =
= (400x0.84) + (900x0.61) + (1200x(-0.42)) = 381
18) Sea (X,Y) una variable aleatoria bidimensional con funcin de densidad conjunta:
f (x, y)
4 x (1 x) 0 x 1 3 y 8
0 en el resto
X, Y
3
=
< < < <

a) Obtener las densidades marginales de X y de Y. Indicar que distribucin siguen.


b) Son X e Y independientes?
c) Calcular la varianza de 3X - Y + 2.
d) Comparar la p(2 < Y < 4) y la
( )
p
2 Y 4
X 1 / 2

< <
<
comentando el resultado.
44 Problemas de examen resueltos
e) Hallar la funcin de densidad de Z = e
-Y
y calcular su esperanza.
SOLUCIN
a) Funcin de densidad marginal de X
( | f (x) 4 x (1 x) dy 4 x (1 x) y 20 x (1 x) 0 x 1
X
3
y 3
y 8
3
y 3
y 8
3
= = = < <
=
=
=
=

X (4,2)
f (x)
1
(p, q)
x (1 x) 0 x 1
0 resto

20x (1 - x) 0 x 1
0 resto
X
p 1 q 1
3
=
< <
=

< <

ya que

1
(4,2)

(4 2)
(4) (2)

5!
3! 1!
20

=
+
= =


Funcin de densidad marginal de Y
f (y) 4 x (1 x) dx x x
1
5
3 y 8
Y
3
x 0
x 1
4 5
x=0
x=1
= =
|
\

(
= < <
=
=

4
5
Y U(3,8)
f (y)
1
b a
a y b
0 resto

1
5
3 y 8
0 resto
Y
=

< <
=

< <

b) Dos variables son independientes si se verifica que:


f
X,Y
(x,y) = f
X
(x)f
Y
(y) x,y R
en este caso tenemos:
f
X,Y
(x,y) =
4 x (1 x) 0< x , 3 y 8
0 en el resto
3
< < <

1
= f
X
(x) f
Y
(y)
luego las variables X e Y son independientes.
Estadstica Empresarial II 45
c) Var(3X - Y + 2) = 3
2
Var(X) + (-1)
2
Var(Y) = 9 x
2
63

25
12
2.369047619
|
\

|
.
| + =
Por X e Y independientes
Var(X) =
pq
(p q 1)(p q)

4 x 2
(4 2 1)(4 2)

2
63
2 2
+ + +
=
+ + +
=
Var(Y) =
(b a)
12

(8 3)
12

25
12

2 2

=

=
d)
( )
( )
( )
| |
( )
( )
( )
( )
p
2 Y 4
X 1/ 2

p 2 Y 4 X 1 2
p X 1 2

p 2 Y 4 p X 1 2
p X 1 2

< <
<
=
< <

=
< <

=
Por X e Y independientes
= p(2 < Y < 4) =
1
5
dy
1
5
y
1
5
y 3
y 4
y 3
y 4
=
=
=
=

=
|
\

(
=
Ambas probabilidades coinciden por ser X e Y independientes.
e) Sea Z = e
-Y
F
Z
(z) = p(Z z) = p(e-Y z) = p(-Y Ln z) = p(Y -Ln z) = 1 - F
Y
(-Ln z)
f
Z
(z) = f ( Ln z)
1
z

1
5 z

Y


=
3 < -Ln z < 8 -3 > Ln z > -8 e
-8
< z < e
-3
luego
f (z)
1
5 z
e z e
0 en el resto
Z
8 3
=
< <


E(Z) = ( ) z
1
5 z
dz
1
5
z
1
5
e e .
z e
z e
z e
z e
3 8
8
3
8
3
=
=
=
=

=
|
\

(
=
46 Problemas de examen resueltos
19) Sea X la v. a. duracin de un componente electrnico de una mquina, medida en meses,
con distribucin exponencial negativa de parmetro a = 0.1.
a) Calcular p( - 2 < X < + 2).
b) Si tenemos otros cinco componentes de repuesto y cambiamos la pieza en el mismo
momento que deja de funcionar, cul ser la probabilidad de que podamos mantener la
mquina en funcionamiento 60 meses sin pedir repuestos? (Indicar los clculos. No se pide el
valor numrico concreto).
c) Si consideramos 20 componentes. Cul es la probabilidad de que al menos para dos de
ellos su duracin sea inferior a 30 meses?
SOLUCIN
Consideramos la variable aleatoria X = Duracin de un componente electrnico de una mquina en
meses (0.1)
a) E(X) =
1
a

1
0.1
10 = = Var(X) =
1
a

1
0.1
100
2 2
= =
luego
= E(X) = 10 y
X
= Var(X) = 10
con lo que
p( - 2 < X < + 2) = p(10 - 2x10 < X < 10 + 2x10) = p(-10 < X < 30) =
= ( | 0.1 e dx e 1 e 0.950212931.
0.1 x
x 0
x 30
0.1x
x 0
x 30
3
=
=

=
=

= = =
b) Definimos la variable T = X
1
+ + X
6
(0.1,6) pues la suma de exponenciales negativas es
una gamma.
p(T 60)
0.1
(6)
t e dt
6
5 0.1t
t 60
t
=

=
=


c) Sea Y = Nmero de componentes cuya duracin es inferior a 30 meses de los 20 considerados.
Llamando xito al suceso
E = Duracin de un componente electrnico de una mquina inferior a 30 meses
p(E) = p(X < 30) = 0.950212931
la variable Y cuenta en nmero de xitos en 20 ensayos de Bernoulli, luego
Y B(20, 0.950212931)
Estadstica Empresarial II 47
y la probabilidad que nos piden es:
p(Y 2) = 1 - p(Y < 2) = 1 -
20
0
09502 0 0498
20
1
09502 0 0498
0 20 1 19
|
\

|
.
| +
|
\

|
.
|

(
. . . . =
= 1 - 3.351211505 10-24 1
20) En una oficina de una entidad financiera saben que los ingresos que se efectan son por
trmino medio de 100 mil pesetas con una desviacin tpica de 20 mil pesetas y que los
reintegros medios son de 40 mil pesetas con una desviacin de 10 mil. Al inicio del da un
furgn lleva la cantidad de dinero solicitada el da anterior.
a) El da 2 de febrero antes de abrir al pblico, haba 4 millones de pesetas en caja. Se efectan 36
ingresos y 217 reintegros. La cantidad suministrada por el furgn fue de 6 millones, cul es la
probabilidad de que al hacer el arqueo al final del da haya en caja a lo sumo 5 millones?
b) Una vez realizado el arqueo se observa que en caja hay 4 millones y medio para el da
siguiente. Si la previsin de ingresos y de reintegros para el da 3 de febrero coincide con los
que se efectuaron el da 2, qu cantidad de dinero habr que pedir que lleve el furgn para
que con una probabilidad del 97.5% al final del da quede en caja al menos la misma cantidad
que al principio del da?
SOLUCIN
Definimos las siguientes variables aleatorias:
X = Ingresos en una entidad financiera (en miles de pesetas)
Y = Reintegros en una entidad financiera (en miles de pesetas)
sabemos que
E(X) = 100
X
= 20 E(Y) = 40
Y
= 10
a) Sea W = X Y
i
i 1
36
i
i 1
217
= =

E(W) = E X Y = E(X ) E(Y ) 36 x 100 217 x 40 -5 080


i
i 1
36
i
i 1
217
i
i 1
36
i
i=1
217
= = =

|
\

|
.
| = =
Var(W) =Var X Y = Var(X ) Var(Y ) 36 x 20 217 x 10 36 100
i
i 1
36
i
i 1
217
i
i 1
36
i
i=1
217
2 2
= = =

|
\

|
.
| + = + =

W
= 190
La probabilidad pedida es:
48 Problemas de examen resueltos
p X Y 4 000 6 000 5 000 p(W 5 000)
i
i 1
36
i
i 1
217
= =

+ +
|
\

|
.
| = =
p
W 5 080
190

5 000 5 080
190

80
190
(0.42) 0.6628 =
+

+
|
\

|
.
|
|
\

|
.
| = =
Tipificando Aplicando el Teorema Central del Lmite
b) Hay que encontrar un valor k tal que
p(W + 4 500 + k 4 500) = p(W -k) = 0.975
luego
0.975 = p(W - k) = p
W 5 080
190

- k 5 080
190
1 -
- k 5 080
190

+

+
|
\

|
.
|
+
|
\

|
.
|
Tipificando Aplicando el Teorema Central del Lmite

k 5 080
190
0.025
+
|
\

|
.
| =
k 5 080
190
-1.96
+
=
k = 5 452.4 miles de pesetas.
21) Sea X una variable aleatoria con la siguiente ley de probabilidad:
X 0 1 2 3
p(X=x) 1/8 1/4 1/2 1/8
a) Cunto vale la varianza de 2X + 5?
b) Calcular la p(X < 3/X 1).
c) Podemos suponer que X sigue una distribucin binomial, por qu?
SOLUCIN
a) E(X) 0 x
1
8
1 x
1
4
2 x
1
2
3 x
1
8
1.625 =
|
\

|
.
| +
|
\

|
.
| +
|
\

|
.
| +
|
\

|
.
| =
E(X ) 0 x
1
8
1 x
1
4
2 x
1
2
3 x
1
8
3.375
2 2 2 2 2
=
|
\

|
.
| +
|
\

|
.
| +
|
\

|
.
| +
|
\

|
.
| =
Estadstica Empresarial II 49
Var(X) = E(X
2
) - E
2
(X) = 3.375 - 1.625
2
= 0.734375
Var(2X + 5) = 2
2
Var(X) = 4 x 0.734375 = 2.9375
b)
( )
( ) ( )
| |
( )
( ) ( )
( ) ( )
p
X 3
X 1

p X 3 X 1
p X 1

p X 1 p X 2
p X 1 p X 2 p( X 3)

<

=
<

=
= + =
= + = + =
=

1
4

1
2
1
4

1
2

1
8

6
7
=
+
+ +
=
c) La funcin de densidad de una variable aleatoria que sigue una distribucin B(n,p) es:
( ) p X k
n
k
p (1 p) k 0, 1, ..., n
k n k
= =
|
\

|
.
| =

donde n = nmero de experimentos realizados


p = Probabilidad de xito
En este caso n = 3, el valor de p le obtendremos despejando, por ejemplo, en la probabilidad de X
valga cero.
( ) p X 0
1
8

3
0
p (1 p) (1 p)
1
8

0 3 3
= = =
|
\

|
.
| =
= = 1 p
1
2
p
1
2

Luego si X sigue una distribucin binomial, esta es una B(3,1/2). Comprobemos si para el resto de
valores coinciden las probabilidades.

3
1

1
2

1
2

3
8

1
4
p(X 1)
1 2
|
\

|
.
|
|
\

|
.
|
|
\

|
.
| = = = X no sigue una distribucin binomial.
22) Dada la variable aleatoria X con funcin de densidad:
f
X
(x) = 6x(1-x) 0 < x < 1
Definimos la variable Y = 5 - 3X. Obtener su funcin de densidad y deducir qu distribucin
sigue.
SOLUCIN
50 Problemas de examen resueltos
Empezaremos calculando la relacin que hay entre las funciones de distribucin para luego ver que
relacin existe entre las funciones de densidad.
F (y) p(Y y) p(5 3X y) p( 3X y 5) p X
y 5
3
1 - F
5 y
3
Y X
= = = =

|
\

|
.
| =

|
\

|
.
|
luego
( ) ( ) ( ) f y f
5 y
3

1
3
= 6
5 y
3
1
5 y
3

1
3

2
9
y 2 5 y
Y X
=

|
\

|
.
|

|
\

|
.
|

|
\

|
.
| =
0
5 y
3
1 0 5 y 3 5 y 2 2 y 5 <

< < < < < < <
con lo que
( )
( ) ( )
f y
2
9
y 2 5 y 2 y 5
0 en el resto
Y
=
< <

Y (2,2,2,5)
f (y)
1
(p, q)
1
(b- a)
(y- a) (b y) a y b
0 resto

2
9
(y- 2) (5- y) 2 y 5
0 resto
Y
p+q-1
p 1 q 1
=
< <
=

< <

ya que

1
(2,2)

1
(5- 2)

(2 2)
(2) (2)
1
3

3!
1! 1!

1
27

2
9

2+2-1 3

=
+
= =


23) Sea X la v. a. nmero de accidentes laborales diarios en un determinado grupo de
empresas.
a) Sabiendo que p(X = 0) = 0.1353352, calcular la esperanza y la varianza de dicha variable y
la probabilidad de que un da determinado haya ms de 3 accidentes.
b) Probabilidad de que en una semana (5 das) ocurran menos de 2 accidentes.
SOLUCIN
Sea X = nmero de accidentes laborales diarios en un determinado grupo de empresas P()
a) p(X 0) 0.1353352 e
0!
2
0
= = = =

Estadstica Empresarial II 51
Luego
E(X) = = 2 y Var(X) = = 2.
p(X > 3) = 1 - p(X 3) = 1 - 0.8571 = 0.1429.
b) Sea Y = nmero de accidentes laborales semanales = X
i
i 1
5
=

.
Como la suma de variables de Poisson es otra variable de Poisson, y el parmetro es la suma de los
parmetros, tenemos que
Y P(10)
p(Y < 2) = p(Y 1) = 0.0005
24) Sea X e Y dos variables aleatorias con la siguiente funcin de densidad conjunta:
f (x, y)
Ky e x 0 y 0
0 resto
X, Y
(2x y)
=

+
a) Comprobar que la constante K = 2.
b) Calcular las funciones de densidad marginal de la X y de la Y. Qu distribucin siguen?
c) Son independientes las variables X e Y?
d) Calcular la esperanza y la varianza la variable Z = 2X-3Y+5.
e) Plantear la P(X+Y< 1).
SOLUCIN
a) Para comprobar que la constante vale 2, usaremos el hecho de que la integral en todo el dominio
de definicin de la funcin de densidad es 1.
1 K y e dx dy K y e
1
2
e dy
K
2
y e dy
(2x y)
x 0
x
y 0
y
y 2x
x 0
x
y 0
y
y
y 0
y
=

(
=

|
\

(
= =
+
=
=
=
=

=
=
=
=

=
=


K
2
ye dy
K
2
K 2
y
y 0
y
= = =

=
=

pues la integral en todo el dominio de


definicin de una (1,2) vale 1
1
b) Marginal de X
52 Problemas de examen resueltos
f (x) 2y e dy 2 e y e dy 2 e x 0
X
(2x y) 2x y
y 0
y
2x
= = =
+
=
=

=
=

y
y
0
X (2), pues
f (x)
a e x 0
0 resto

2 e x 0
0 resto
X
-ax -2x
=
>
=

>

Marginal de Y
f (y) 2y e dx y e 2 e dx y e y 0
Y
(2x y) y
x 0
x
= = =
+
=
=

=
=

2
0
x y
x
x
Y (1,2)
f (y)
a
(p)
y e y 0
0 resto

y e y 0
0 resto
Y
p
p-1 -ay
-y
=
>
=

>

pues

a
(p)

1
(2)

1
1!
1
p 2

= = =
c) Dos variables son independientes si se verifica que:
f
X,Y
(x,y) = f
X
(x)f
Y
(y) x,y R
en este caso tenemos:
f
X,Y
(x,y) =
2y e x 0 y 0
0 resto
(2x y) +

= f
X
(x) f
Y
(y)
luego las variables X e Y son independientes.
d) Calcularemos primero las esperanzas y las varianzas de X e Y.
E(X)
1
a

1
2
= = = 05 . E(Y)
p
a

2
1
2 = = =
Estadstica Empresarial II 53
Var(X)
1
a

1
2

2 2
= = = 0 25 . Var(Y)
p
a

2
1
2
2 2
= = =
E(2x - 3Y + 5) = 2E(X) - 3E(Y) +5 = (2x0.5) - (3x2) + 5 = 0
Var(2X - 3Y + 5) = 2
2
Var(X) + (-3)
2
Var(Y) = (4x0.25) + (9x2) = 19
Por ser X e Y independientes
e)
p(X Y 1) 2ye dy dx
(2x y)
y 0
y 1 x
x 0
x 1
+ < =

(
(
+
=
=
=
=

25) Un producto se presenta a la venta en un paquete que contiene tres bolsas de producto en
su interior. Sabiendo que:
El peso del envase exterior es una v. a. con distribucin N(50, 6) medida en gramos
El peso de cada bolsa de producto es una v. a. con distribucin N(100, 6) medida en gramos
a) Calcular la distribucin de la v. a. peso total de un paquete.
b) Si para su distribucin los paquetes se envan en cajas de 100, calcular la probabilidad de
que una caja pese ms de 35.2 Kilogramos.
SOLUCIN
Definimos las siguientes variables:
X = Peso del envase exterior N(50,6)
Y = Peso de una bolsa de producto N(100,6)
Las variables X e Y son independientes.
a) Sea Z = Peso total de un paquete = X + Y
1
+ Y
2
+ Y
3
N(,), pues la suma de variables
normales es otra variable normal. Determinemos el valor de sus parmetro.
E(Z) = = E(X + Y
1
+ Y
2
+ Y
3
) = E(X) + E(Y
1
) + E(Y
2
) + E(Y
3
) = 50 + 100 + 100 + 100 = 350
Var(Z) =
2
= Var(X + Y
1
+ Y
2
+ Y
3
) = Var(X) + Var(Y
1
) + Var(Y
2
) + Var(Y
3
) =
= 6
2
+ 6
2
+6
2
+6
2
= 144
X
Y
X + Y = 1
1
1
54 Problemas de examen resueltos
luego Z N(350,12)
b) Sea W = Peso de una caja = Z N(n , n ) = N(100x350, 100 x
i
i 1
100
=

12) = N(35 000,120)


p(W > 35 200) = 1 - p(W 35 200) = 1 p
W 35 000
120

35 200 35 000
120


|
\

|
.
| =
= 1 - (1.67) = 1 - 0.9525 = 0.0475.
26) Una empresa dedicada a la venta de ordenadores se plantea contratar a una persona para
el montaje de los equipos en las siguientes condiciones: cobrar una cantidad fija de 75 mil
pesetas al mes ms una cantidad variable por cada equipo que monte, que seguir una
distribucin uniforme de parmetros mil y dos mil quinientas pesetas, en funcin de los
componentes que lleve.
El precio de cada ordenador se ver incrementado por el concepto de montaje en una
cantidad variable en funcin del nmero de componentes y que seguir una distribucin
uniforme de parmetros dos mil y cuatro mil cuatrocientas pesetas.
a) Calcular la probabilidad de que la cantidad variable del salario del empleado por el
montaje de un equipo oscile entre novecientas y mil trescientas pesetas.
b) Si se estima que el nmero de equipos vendidos mensualmente es 70, cul sera el salario
esperado? Y con que varianza?
c) Cul ser el nmero mnimo de equipos que se debern vender durante un mes, para que
los servicios de este empleado no superen a los ingresos obtenidos por la empresa en concepto
de montaje, con una probabilidad del 97.5%?
SOLUCIN
a) Consideramos la variable X = Cantidad cobrada por el empleado por el montaje de un equipo (en
miles de pesetas) U(1,2.5) segn nos dice el enunciado del problema.
f (x)
1
b- a
a x b
0 resto

1
2.5-1
1 x
0 resto
1
1.5
1 x
0 resto
X
=
< <
=

< <

=
< <

2 5
2 5
.
.
( | p(0.9 X 1.3) f (x) dx 0 dx
1
1.5
dx 0
1
1.5
x 0.2
X
x 0.9
x 1.3
x 0.9
x 1
x 1
x 1.3
x 1
x 1,3
< < = = + = + =
=
=
=
=
=
=
=
=

b) S = Salario mensual del empleado = 75 X
i
i 1
70
+
=

Estadstica Empresarial II 55
E(X)
a b
2

1 2.5
2
1.75 =
+
=
+
=
( ) ( )
Var(X)
b a
12

2.5 1
12
0.1875
2 2
=

=

=
E(S) = E 75 X 75 E(X ) 75 (70 x 1.75) 197.5
i
i 1
70
i
i 1
70
+
|
\

|
.
| = + = + =
= =

miles de pesetas.
Var(S) = Var 75 X Var(X ) 70 x 0.1875 13.125
i
i 1
70
i
i 1
70
+
|
\

|
.
| = = =
= =

c) Sea Y = Incremento en el precio de un equipo en concepto de montaje (en miles de pesetas), que
segn el enunciado U(2,4.4)
Definimos B = Beneficios de la empresa por el concepto de montaje.
B = Ingresos - Gastos = Y 75 X
i
i 1
n
i
i 1
n
= =


El problema nos pide encontrar
n / p(B > 0) = 0.975
Como la distribucin de B no la conocemos y es suma de variables aleatorias independientes
idnticamente distribuidas con esperanza y varianza finitas, podemos aplicar el Teorema Central del
Lmite, es decir,
B E(B)
Var(B)
N(0,1)
L


E(B) = ( ) ( ) E Y 75 X E Y 75 E X
i
i 1
n
i
i 1
n
i
i 1
n
i
i 1
n
= = = =


|
\

|
.
| = =
=
+
|
\

|
.
|
2 4.4
2
n 75 (1.75 n) = 1.45 n - 75
Var(B) = ( ) ( ) Var Y 75 X Var Y + Var X
i
i 1
n
i
i 1
n
i
i 1
n
i
i 1
n
= = = =


|
\

|
.
| = =
=
|
\

|
.
| +
(4.4- 2)
2
n (0.1875 n)
2
= 0.6675 n
56 Problemas de examen resueltos
0.975 = p(B 0) = p
B 1.45n +75
0.6675n

1.45n +75
0.6675n
1 -
1.45n +75
0.6675n
>

>

|
\

|
.
|
|


|
\

|
.
|
|
0 0


Tipificando Aplicando el Teorema Central del Lmite

0-1.45n 75
0.6675n
0.025
+ |
\

|
.
| =
75 1.45 n
0.6675 n
-1.96

=
1.45 n - 0.6675 n - 75 = 0 n
7.479191827
6.915738909
=


n
55.9383
47.8274
=

la solucin n = 47.8274 la tenemos que desechar pues al sustituir su valor en la probabilidad de


partida no da 0.975, que es el valor pedido, sino que da una probabilidad de 0.025.
Por tanto el valor pedido, redondeando, es n = 56 equipos.
27) Sean X e Y variables aleatorias independientes con funciones de densidad:


=

resto el en 0
0 x xe
(x) f
x
X


=
resto el en 0
3 y 1 y
4
1
(y) f
Y
a) Calcular la funcin de densidad conjunta.
b) Calcular la probabilidad de que la variable Y sea mayor que 2.
c) Calcular la esperanza y varianza de la variable: T = 2X - 3Y
d) Comprobar que se verifica: E(XY) = E(X) E(Y), indicando el motivo.
e) Calcular p(X + Y < 3). (Dejar indicada la integral correspondiente).
SOLUCIN
a) Como las variables X e Y son independientes, la funcin de densidad conjunta es el producto de
las marginales, luego


= =

resto el en 0
3 y 1 0, x e y x
4
1
(y) f (x) f y) (x, f
x
Y X Y X,
Estadstica Empresarial II 57
b) p(Y > 2) =
8
5
y
8
1
dy y
4
1
3
2
2
3 y
2 y
=
(

\
|
=
=
=
=
=

y
y
c) Comprobemos que X (1,2)

>

=
>

=
resto 0
0 x e x

resto 0
0 x e x
(p)
a
(x) f
x -
ax - 1 - p
p
X
pues
1
1!
1
(2)
1
(p)
a

2 p
= =

luego
2
1
2

a
p
E(X) = = =
2
1
2

a
p
Var(X)
2 2
= = =
Calculemos ahora la esperanza y la varianza de Y.
6
13
y
12
1
dy y
4
1
y E(Y)
3 y
1 y
3
3 y
1 y
=
(

\
|
= =
=
=
=
=

5 y
16
1
dy y
4
1
y ) E(Y
3 y
1 y
4
3 y
1 y
2 2
=
(

\
|
= =
=
=
=
=

Var(Y) = E(X
2
) E
2
(X) =
36
11
6
13
5
2
= |
.
|

\
|

Con lo que
E(T) = E(2X 3Y) = 2E(X) 3E(Y) = ( )
|
.
|

\
|

6
13
x 3 2 x 2 = -2.5
58 Problemas de examen resueltos
Var(T) = Var(2X 3Y) = 2
2
Var(X) + (-3)
2
Var(Y) = ( )
|
.
|

\
|
+
36
11
x 9 2 x 4 = 10.75
por ser X e Y independientes
d) E(XY) ==

=
=
=
=

(
(

x
0 x
3 y
1 y
x
dx dy e y x
4
1
y x =

=
=
=
=

\
|
x
0 x
3 y
1 y
3 x 2
dx y
12
1
e x =

=
=

x
0 x
x 2
dx e x
6
13
=
=

=
=

x
0 x
x 2
dx e x
6
13
= 2! x
6
13
=
3
13
E(X) E(Y) =
6
13
x 2 =
3
13
= E(XY)
La igualdad se verifica por que las variables X e Y son independientes.
e) p(X + Y < 3) =

=
=
=
=

(
(

2 x
0 x
x 3 y
1 y
x
dx dy e y x
4
1
=
=

=
=
=
=

(
(

3 y
1 y
y 3 x
0
x
dy dx e y x
4
1
x
28) En una poblacin formada por 2 400 trabajadoras, se desea estudiar las siguientes
variables aleatorias:
X = N de hijos ; Y = N de das de absentismo laboral al mes.
Si la ley de probabilidad conjunta de ambas variables es:
X \ Y 0 1 2
0 2/24 2/24 2/24
1 4/24 0 3/24
2 4/24 4/24 0
3 1/24 1/24 1/24
a) Calcular las leyes de probabilidad marginales de X e Y.
X
Y
X + Y = 3
3
3
Y = 3
Y = 1
2
1
Estadstica Empresarial II 59
b) Cuantas empleadas tienen menos de 2 hijos?
c) Calcular:
|
.
|

\
|

2 X
1 Y
p .
d) Calcular:
|
.
|

\
|
= 3 X
Y
E .
SOLUCIN
a) Para obtener las leyes de probabilidad marginales sumaremos por filas y por columnas la ley de
probabilidad conjunta
X \ Y 0 1 2 p(X = X)
0 2/24 2/24 2/24 6/24
1 4/24 0 3/24 7/24
2 4/24 4/24 0 8/24
3 1/24 1/24 1/24 3/24
p(Y = y) 11/24 7/24 6/24 1
luego la ley de probabilidad marginal de X es
X 0 1 2 3
p(X = x) 6/24 7/24 8/24 3/24
y la ley de probabilidad marginal de Y es
Y 0 1 2
p(Y = y) 11/24 7/24 6/24
b) Calcularemos primero la probabilidad de que una mujer tenga menos de dos hijos y a esa
probabilidad le multiplicaremos por el nmero total de mujeres para obtener el valor pedido.
p(X < 2) = p(X = 0) + p(X = 1) =
24
13
24
7
24
6
= +
con lo que
N p(X < 2) = 2 400
24
13
x = 1 300 mujeres tienen menos de 2 hijos.
c)
( )
( )
=


=
|
.
|

\
|

2 X p
2 X 1, Y p
2 X
1 Y
p
( ) ( ) ( ) ( )
=
= + =
= = + = = + = = + = =
=
3) p(X 2) p(X
2 Y 3, X p 1 Y 3, X p 2 Y 2, X p 1 Y 2, X p
60 Problemas de examen resueltos
11
6
24
3
24
8
24
1
24
1
0
24
4
=
+
+ + +
=
d) Empezaremos obteniendo la ley de probabilidad de Y condicionada por X = 3 y luego
calcularemos la esperanza.
La ley de probabilidad de Y condicionada por X = 3 se obtiene
3) p(X
3) X y, p(Y
3 X
y Y
p
=
= =
=
|
.
|

\
|
=
=
por ejemplo para Y = 0 sera:
( )
3
1
24
3
24
1
3) p(X
3) X 0, p(Y
3 X
0 Y
p = =
=
= =
=
=
=
luego la ley de probabilidad de Y condicionada por X = 3 es:
Y/X = 3 0 1 2
p(Y = y/X = 3) 1/3 1/3 1/3
con lo que la esperanza pedida es:
( ) 1
3
1
x 2
3
1
x 1
3
1
x 0
3 X
Y
E =
|
.
|

\
|
+
|
.
|

\
|
+
|
.
|

\
|
=
=
29) Se sabe que la proporcin de grasa en cada lata de pat es una variable aleatoria X que
tiene como funcin de densidad:


=
resto el en 0
1 x 0 x) (1 x 12
(x) f
2
X
a) Qu distribucin sigue la variable X?
b) Si se analizan 10 latas, cul es la probabilidad de que al menos dos latas tengan un
contenido en grasa menor que el 30 por ciento?
c) Si ahora se van analizando las latas una a una, cul es la probabilidad de que la primera
lata que contiene menos del 30 por ciento en grasa sea la quinta?
SOLUCIN
Sea X = Proporcin de grasa en cada lata de pat
Estadstica Empresarial II 61
a) X (2,3)

< <

=
< <

=

resto 0
1 x 0 x) - (1 x 12
resto 0
1 x 0 x) (1 x
q) (p,
1
(x) f
2
1 q 1 p
X
ya que
2 1
2! 1!
4!
(3) (2)
) 3 (2
(2,3)
1
= =

+
=

b) Consideramos el suceso E = El contenido de grasa de una lata de pat es menor que el 30 %.


Calculemos la probabilidad de este suceso.
p(E) = p(X < 0.3) = = + =

=
=
=
=
0.3 x
0 x
3 2
0.3 x
0 x
2
dx ) x 12 x 24 x (12 dx x) (1 x 12
( | 0.3483 x 3 x 8 x 6
0.3 x
0 x
4 3 2
= + =
=
=
Definimos ahora la variable Y = Nmero de latas de las 10 analizadas con menos de un 30 % de
grasa.
Llamando xito al suceso E anterior, la variable aleatoria Y cuenta xitos en 10 ensayos, por lo
tanto, sigue una distribucin binomial.
Y B(10, 0.3483)
La probabilidad que nos piden es:
p(Y 2) = 1 p(Y < 2) =
(

|
|
.
|

\
|
+
|
|
.
|

\
|

9 1 10 0
6517 . 0 3483 . 0
1
10
6517 . 0 3483 . 0
0
10
1 = 0.91232546
c) Definimos ahora la variable Z = Nmero de latas con ms del 30 % de grasa antes de la primera
con menos del 30 % de grasa.
Si se seguimos llamando xito al mismo suceso de antes, la variable Z cuenta los fracasos antes del
primer xito, luego sigue una distribucin geomtrica.
Z G(0.3483)
y la probabilidad pedida es:
p(Z = 4) = 0.6517
4
x 0.3483 = 0.062826715
62 Problemas de examen resueltos
30) El trayecto de una lnea de autobuses urbana se realiza por trmino medio en 48 minutos,
con una desviacin de 4 minutos. Durante un da se realiza el recorrido 64 veces.
a) Calcular la probabilidad de que la duracin total de los sesenta y cuatro trayectos sea mayor que
3 104 minutos.
b) Calcular la probabilidad de que la duracin media de esos sesenta y cuatro recorridos sea
menor que 47 minutos.
c) Sabiendo adems, que el nmero de viajeros diario de esa lnea es de 3 200, que el precio
medio del billete es de 70 pesetas con una desviacin de 10, que el litro de combustible cuesta
96 pesetas, que el consumo en litros de combustible es la vigsimo cuarta parte de la duracin
del trayecto, y que cada trayecto tiene unos costes fijos de 3 300 pesetas, calcular la
probabilidad de tener beneficio al final de un da.
SOLUCIN
Sea la variable aleatoria X = Duracin de un trayecto de una lnea urbana. Sabemos que
E(X) = 48
X
= 4 nmero de trayecto = 64
a) Consideramos la variable aleatoria Y = Duracin total de los sesenta y cuatro trayectos =

=
64
1 i
i
X
( )

= = =
= = |
.
|

\
|
64
1 i
64
1 i
i
64
1 i
i
48 X E X E = 64 x 48 = 3 072
( )

= = =
= = |
.
|

\
|
64
1 i
2
64
1 i
i
64
1 i
i
4 X Var X Var = 64 x 16 = 1 024

=
64
1 i
i
X
= 32
por ser X
i
independientes
La probabilidad pedida es: |
.
|

\
|
>

=
104 3 X p
64
1 i
i
De la variable aleatoria duracin total no conocemos su distribucin, pero si sabemos que es suma
de variables aleatorias independientes idnticamente distribuidas con esperanza y varianza finitas,
con lo cual podemos aplicar el Teorema Central del Lmite, y aproximar el valor de dicha
probabilidad.
) 1 ( 1
32
072 3 104 3
32
072 3 X
p 104 3 X p
64
1 i
i 64
1 i
i

|
|
|
|
.
|

\
|

>

= |
.
|

\
|
>

=
=
= 1 0.8413 = 0.1587
tipificando Aplicando el Teorema Central del Lmite
Estadstica Empresarial II 63
b) Definimos X = Duracin media de los sesenta y cuatro trayectos =
64
X
64
1 i
i
=
.
Estamos en las mismas condiciones del apartado anterior y volveremos a aplicar el Teorema Central
del Lmite. Calcularemos primero la esperanza y la varianza de la variable X.
64
072 3
64
X E
64
X
E
64
1 i
i
64
1 i
i
=
|
.
|

\
|
=
|
|
|
|
.
|

\
|

=
=
= 48
2 2
64
1 i
i
64
1 i
i
64
024 1
64
X Var
64
X
Var =
|
.
|

\
|
=
|
|
|
|
.
|

\
|

=
=
= 0.25
X
= 0.5
( ) ) 2 (
0.5
48 47
0.5
48 X
p 47 X p
|
|
.
|

\
|
<

= < = 0.0228
tipificando Aplicando el Teorema Central del Lmite
c) Disponemos de la siguiente informacin:
Nmero de viajeros de la lnea = 3 200
P = Precio del billete E(P) = 70
P
= 10
Precio del litro de combustible = 96 pesetas
C = Consumo de combustible =
24
X
Costes fijos de cada trayecto = 3 300
Definimos la variable
B = Beneficios = Ingresos Gastos =

= =
|
.
|

\
|
+
64
1 i
i
200 3
1 i
i
300 3
24
X
96 P
Al igual que en los apartados anteriores, aplicaremos el Teorema Central del Lmite.
64 Problemas de examen resueltos
E(B) = ( ) ( ) | |

= = = =
+ =
(

|
.
|

\
|
+
64
1 i
i
200 3
1 i
i
64
1 i
i
200 3
1 i
i
300 3 X E 4 P E 300 3
24
X
96 P E =
= (3 200 x 70) {64 x [(4 x 48) + 3 300]} = 512
Var(B) = ( ) ( ) | |

= = = =
+ =
(

|
.
|

\
|
+
64
1 i
i
2
200 3
1 i
i
64
1 i
i
200 3
1 i
i
X Var 4 P Var 300 3
24
X
96 P Var =
= (3 200 x 10
2
) + (64 x 4
2
x 4
2
) = 336 384

B
= 579.9862067
( ) ) 88 . 0 ( 1
579.986
512 0
579.986
512 B
p 0 B p
|
.
|

\
|
>

= > = 1 - 0.1894 = 0.8106


tipificando Aplicando el Teorema Central del Lmite
31) Para estudiar la relacin entre el precio y la cantidad vendida de un determinado
artculo, se observa durante cierto tiempo ambas variables en varias tiendas, obteniendo la
siguiente ley de probabilidad conjunta:
X = Precio del artculo; Y = Cantidad adquirida por cliente en cada compra.
X \ Y 25 30 35
1 0 0.05 0.25
2 0.1 0.05 0.05
3 0.4 0.1 0
a) Calcular el coeficiente de correlacin de X e Y, comentando el resultado.
b) Calcular la probabilidad de que el gasto total por cliente en cada compra sea mayor que 60
ptas.
SOLUCIN
a) Para calcular el coeficiente de correlacin de X e Y, nos hace falta calcular previamente las
esperanzas y las varianzas de X e Y, as como la covarianza entre ambas variables.
Empezaremos calculando las leyes de probabilidad marginales.
Estadstica Empresarial II 65
X \ Y 25 30 35 p(X = x)
1 0 0.05 0.25 0.3
2 0.1 0.05 0.05 0.2
3 0.4 0.1 0 0.5
p(Y = y) 0.5 0.2 0.3 1
E(X) = (1x0.3) + (2x0.2) + (3x0.5) = 2.2
E(X
2
) = (1
2
x0.3) + (2
2
x0.2) + (3
2
x0.5) = 5.6
Var(X) = E(X
2
) E
2
(X) = 5.6 2.2
2
= 0.76
E(Y) = (25x0.5) + (30x0.2) + (35x0.3) = 29
E(Y
2
) = (25
2
x0.5) + (30
2
x0.2) + (35
2
x0.3) = 860
Var(Y) = E(Y
2
) E
2
(Y) = 860 29
2
= 19
E(XY) = (1x25x0) + (1x30x0.05) + (1x35x0.25) + (2x25x0.1) + (2x30x0.05) +
+ (2x35x0.05) + (3x25x0.4) + (3x30x0.1) + (3x35x0) = 60.75
Cov(X,Y) = E(XY) E(X)E(Y) = 60.75 (2.2x29) = -3.05
=

= =
19 x 0.76
3.05
Var(Y) Var(X)
Y) Cov(X,
Y X,
-0.80263158
El valor del coeficiente de correlacin est prximo a 1 lo que quiere decir que entre las variables
X = Precio del artculo, e Y = Cantidad adquirida por cliente en cada compra hay una relacin lineal
alta y negativa, es decir, cuando aumenta el precio disminuye la cantidad adquirida por el cliente.
b) Sea G = Gasto total por cliente en cada compra = X Y. Nos piden calcular:
p(G > 60) = p(X Y > 60) = p[(X = 2) (Y = 35)] + p[(X = 3) (Y = 25)] +
+ p[(X = 3) (Y = 30)] + p[(X = 3) (Y = 35)] = 0.05 + 0.4 + 0.1 + 0 = 0.55.
32) Sean X e Y dos variables aleatorias con funcin de densidad conjunta:

< < <


=
resto el en 0
4 x y 0
8
1
y) (x, f
Y X,
66 Problemas de examen resueltos
a) Comprobar que es funcin de densidad.
b) Obtener las funciones de densidades marginales de X e Y.
c) Obtener la funcin de densidad condicionada de Y por X.
d) Qu distribuciones siguen X e Y/X=x?
e) Calcular E(Y/X=3).
f) Calcular la p(X+Y 4).
SOLUCIN
a) Para comprobar que es funcin de densidad, tendremos que ver que es siempre mayor o igual que
cero, y que la integral en todo el dominio de definicin vale uno.
Claramente, se puede observar que es mayor o igual que cero, pues o bien vale 1/8 o bien vale 0.
Veamos que la integral vale 1.
( | = =
(
(


=
=
=
=
=
=
=
=
4 x
0 x
x y
0 y
4 x
0 x
x y
0 y
dx y
8
1
dx dy
8
1
1 x
16
1
dx x
8
1
4 x
0 x
2
4 x
0 x
=
(

\
|
= =
=
=
=
=

luego efectivamente es funcin de densidad.


b) Funcin de densidad marginal de X
f
X
(x) = x
8
1
y
8
1
dy
8
1
x y
0 y
x y
0 y
=
(

\
|
=
=
=
=
=

0 < x < 4
luego

< <
=
resto el en 0
4 x 0 x
8
1
(x) f
X
Funcin de densidad marginal de Y
f
Y
(y) = y) - (4
8
1
x
8
1
dx
8
1
4 x
y x
4 x
y x
=
(

\
|
=
=
=
=
=

0 < y < 4
X
Y
4
Y = X
Estadstica Empresarial II 67
luego

< <
=
resto el en 0
4 y 0 y) - 4 (
8
1
(y) f
Y
c) Funcin de densidad condicionada de Y por X.
( )
x
1
x
8
1
8
1
(x) f
y) (x, f
x y/X f
X
Y X,
x Y/X
= = = =
=
0 < y < x
siempre que 0 < x < 4
d) X e(2,1,0,4)

< <

=
< <

=

+
resto el en 0
4 x 0 x
8
1
resto el en 0
b x a x) (b a) - x (
a) - (b
1
q) (p,
1
(x) f
1 q 1 p
1 - q p
X
ya que
8
1
16
1
0! 1!
2!
16
1
(1) (2)
) 1 (2
) 0 4 (
1
(2,1)
1
1 1 2
= =

+
=

+
Y /X=x U(0,x)

< <
=

< <
=
=
resto el en 0
x y 0
x
1
resto el en 0
b y a
a - b
1
(y) f
x Y/X
e) Sustituyendo X = 3 en la funcin de densidad Y/X=x tenemos:
( )
3
1
3 y/X f
3 Y/X
= =
=
0 < y < 3
68 Problemas de examen resueltos
luego
( ) 1.5 y
6
1
dy
3
1
y x Y/X E
3 y
0 y
2
3 y
0 y
=
(

\
|
= = =
=
=
=
=

f)
p(X + Y 4) =

=
=
=
= (
(

2 y
0 y
y 4 x
y x
dy dx
8
1
=
( ) = =
(

\
|
=

=
=
=
=
=
=
2 y
0 y
2 y
0 y
y 4 x
y x
dy 2y 4
8
1
dy x
8
1
( | 0.5 y 4y
8
1 2 y
0 y
2
= =
=
=
33) Sean X e Y dos variables aleatorias con distribuciones (2,3) y T(3,6,12) respectivamente,
y con Cov(X,Y) = 1. Sean S = 2X 3Y + 4 y T = 5X + 8Y. Obtener:
a) La esperanza y la varianza de S y de T.
b) El coeficiente de correlacin entre S y T.
SOLUCIN
a) Para calcular las esperanzas y las varianzas pedidas, aplicaremos las propiedades de ambas.
Empezaremos calculando la esperanza y la varianza de X y de Y.
Como X (2,3)
E(X) = = =
2
3
a
p
1.5 Var(X) = = =
2 2
2
3
a
p
0.75
y como Y T(3,6,12)
E(Y) = =
+ +
=
+ +
3
12 6 3
3
b m a
7
Var(Y) =
( ) ( )( ) ( ) ( )( )
=

=

18
3 6 6 12 3 - 12
18
a m m b a - b
2 2
3.5
E(S) = E(2X 3Y + 4) = 2E(X) 3E(Y) + 4 = (2x1.5) (3x7) + 4 = -14
X
Y
4
X + Y = 4
Y = X
2
Estadstica Empresarial II 69
Var(S) = Var(2X 3Y + 4) = 2
2
Var(X) + (-3)
2
Var(Y) + 2x[2x(-3)]Cov(X,Y) =
= (2
2
x0.75) + [(-3)
2
x3.5] + {2x[2x(-3)]x1} = 22.5
E(T) = E(5X + 8Y) = 5E(X) + 8E(Y) =(5x1.5) + (8x7) = 63.5
Var(T) = Var(5X + 8Y) = 5
2
Var(X) + 8
2
Var(Y) + 2x(5x8)Cov(X,Y) =
= (5
2
x0.75) + (8
2
x3.5) + [2x(5x8)x1] = 322.75
b) Para calcular el coeficiente de correlacin entre S y T necesitamos obtener el valor de la
covarianza entre dichas variables, el cual se obtendr aplicando sus propiedades.
Cov(S,T) = Cov[(2X 3Y + 4),(5X + 8Y)] =
= (2x5)Cov(X,X) + (2x8)Cov(X,Y) + (-3x5)Cov(Y,X) + (-3x8)Cov(Y,Y) =
= 10Var(X) + 16Cov(X,Y) -15Cov(X,Y) 24Var(Y) =
= (10x0.75) + (1x1) (24x3.5) = -75.5
=

= =
75 . 322 x 5 . 22
75.5
Var(T) Var(S)
T) Cov(S,
T S,
-0.885977244.
34) Una tienda vende dos tipos distintos de leche A y B. Las ganancias diarias por la venta de
leche, se distribuye normalmente en cada caso. Se sabe que la ganancia media diaria por la
venta de la marca A es de 600 ptas. con una desviacin tpica de 20, que el 2.28 % de las
ganancias diarias obtenida por la venta de la marca B son superiores a 800 y que el 96.9 %
estn comprendidas entre 690 y 800 ptas.
a) Calcular la media y la varianza de las ganancias diarias obtenidas por la venta de leche de la
marca B.
b) Cul ser el nmero mnimo de das que tiene que vender leche de la marca A para que la
ganancia, por este concepto, supere las 21 900 ptas. con una probabilidad del 0.9938?
c) Qu distribucin sigue la ganancia semanal (7 das) por la venta de leche?
SOLUCIN
a) Sea B = Ganancias obtenidas por la venta de leche de la marca B N(,). Para determinar el
valor de los parmetros disponemos de las siguientes igualdades:
p(B > 800) = 0.0228 p(690 < B < 800) = 0.969
Tipificando y despejando tenemos:
70 Problemas de examen resueltos
p(B > 800) = 0.0228 0.0228
800
1 =
|
.
|

\
|



0.9772
800
=
|
.
|

\
|


2
800
=


p(690 < B < 800) = 0.969
969 . 0
690
- 0.9772
690
-
800
= |
.
|

\
|


= |
.
|

\
|


|
.
|

\
|



0.0082
690
= |
.
|

\
|


4 . 2
690
=


y resolviendo el sistema:

800
= 2

690
= -2.4
se obtiene que
= 750 y = 25
luego B N(750,25).
E(B) = 750 Var(B) = 25
2
= 625
b) Sea A = Ganancias obtenidas por la venta de leche de la marca A N(600,20). Definimos la
variable:
X = Ganancias obtenidas por la venta de leche de la marca A en n das =

=
n
1 i
i
A
Aplicando las propiedades de la normal sabemos que X ) n 20 N(600n, ) n , N(n =
En este apartado piden encontrar un valor de n tal que p(

=
n
1 i
i
A > 21 900) = 0.9938.
Tipificando se tiene:
Estadstica Empresarial II 71
0.9938 = p(

=
n
1 i
i
A > 21 900) =
|
|
.
|

\
|

=
|
|
|
|
.
|

\
|

>

=
n 20
n 600 900 21
1
n 20
n 600 900 21
n 20
n 600 A
p
n
1 i
i

|
|
.
|

\
|

n 20
n 600 900 21
= 0.0062
n 20
n 600 900 21
= -2.5
600n - 50 n - 21 900 = 0

=
6
3 08 . 6
n
)

=
36
37.0069
n
la solucin n = 36 la tenemos que desechar pues al sustituir su valor en la probabilidad de partida no
da 0.9938, que es el valor pedido, sino que da una probabilidad de 0.0062.
Por tanto el valor pedido, redondeando, es n = 37 das.
c) Definimos la variable aleatoria G = Ganancia semanal por la venta de leche = ( )

=
+
7
1 i
i i
B A
Como la variable G es suma de variables aleatorias normales e independientes, G tambin sigue una
distribucin normal, veamos de que parmetros.
E(G) = ( ) ( ) ( ) ( ) ( ) = + = + = |
.
|

\
|
+

= = =
7
1 i
7
1 i
i i
7
1 i
i i
750 600 B E A E B A E 7x1 350 = 9 450
Var(G) = ( ) ( ) ( ) ( ) ( ) = + = + = |
.
|

\
|
+

= = =
7
1 i
2 2
7
1 i
i i
7
1 i
i i
25 20 B Var A Var B A Var 7x1 025 = 7 175
luego G ( ) 175 7 450, 9 N
72 Problemas de examen resueltos
35) El nmero medio de cheques ingresados en un banco es de 50 por da con una desviacin tpica
de 2. Calcular la probabilidad de que en un trimestre (81 das) el nmero de cheques ingresados est
entre 4 000 y 4 100.
SOLUCIN
Sea X = Nmero de cheques ingresados en un banco al da. Sabemos que:
E(X) = 50
X
= 2
Sea Y = Nmero de cheques ingresados en un banco al trimestre =

=
81
1 i
i
X .
( )

= = =
= = |
.
|

\
|
81
1 i
81
1 i
i
81
1 i
i
50 X E X E = 81 x 50 = 4 050
( )

= = =
= = |
.
|

\
|
81
1 i
2
81
1 i
i
81
1 i
i
2 X Var X Var = 81 x 4 = 324

=
81
1 i
i
X
= 18
por ser X
i
independientes
Nos piden: p(4 000 <

=
81
1 i
i
X < 4 100)
De la variable aleatoria nmero de cheques ingresados en un banco al mes no conocemos su
distribucin, pero si sabemos que es suma de variables aleatorias independientes idnticamente
distribuidas con esperanza y varianza finitas, con lo cual podemos aplicar el Teorema Central del
Lmite, y aproximar el valor de dicha probabilidad.

|
|
|
|
.
|

\
|

<

<

= |
.
|

\
|
< <

=
=
18
050 4 100 4
18
050 4 X
18
050 4 000 4
p 100 4 X 000 4 p
81
1 i
i
81
1 i
i
tipificando Aplicando el Teorema Central del Lmite
) 78 . 2 ( (2.78) = 0.9973 0.0027 = 0.9946
Estadstica Empresarial II 73
36) En una cafetera se realiza un estudio sobre las variables X = N de cafs servidos por
mesa e Y = N de infusiones servidas por mesa, con la siguiente ley de probabilidad
conjunta:
X \ Y 0 1 2
0 0 0 0.1
1 0 0.2 0
2 0.3 0.1 0
3 0.1 0.2 0
a) Calcular el coeficiente de correlacin de X e Y. Interpretar su resultado.
b) Obtener la probabilidad de que el nmero total de consumiciones (cafs, infusiones)
servidas por mesa sea al menos tres.
c) Calcular el nmero medio de cafs servidos por cada mesa en la que se ha servido una
infusin.
SOLUCIN
a) Para calcular el coeficiente de correlacin de X e Y, nos hace falta calcular previamente las
esperanzas y las varianzas de X e Y, as como la covarianza entre ambas variables.
Empezaremos calculando las leyes de probabilidad marginales.
X \ Y 0 1 2 p(X = x)
0 0 0 0.1 0.1
1 0 0.2 0 0.2
2 0.3 0.1 0 0.4
3 0.1 0.2 0 0.3
p(Y = y) 0.4 0.5 0.1 1
E(X) = (0x0.1) + (1x0.2) + (2x0.4) + (3x0.3) = 1.9
E(X
2
) = (0
2
x0.1) + (1
2
x0.2) + (2
2
x0.4) + (3
2
x0.3) = 4.5
Var(X) = E(X
2
) E
2
(X) = 4.5 1.9
2
= 0.89
E(Y) = (0x0.4) + (1x0.5) + (2x0.1) = 0.7
E(Y
2
) = (0
2
x0.4) + (1
2
x0.5) + (2
2
x0.1) = 0.9
Var(Y) = E(Y
2
) E
2
(Y) = 0.9 0.7
2
= 0.41
E(XY) = (0x0x0) + (0x1x0) + (0x2x0.1) + (1x0x0) + (1x1x0.2) + (1x2x0) +
+(2x0x0.3) + (2x1x0.1) + (2x2x0) + (3x0x0.1) + (3x1x0.2) + (3x2x0) = 1
Cov(X,Y) = E(XY) E(X)E(Y) = 1 (1.9x0.7) = -0.33
74 Problemas de examen resueltos
=

= =
0.41 x 0.89
0.33
Var(Y) Var(X)
Y) Cov(X,
Y X,
-0.5462947
El valor del coeficiente de correlacin esta aproximadamente a la mitad entre 0 y 1 lo que
quiere decir que entre las variables X = N de cafs servidos por mesa, e Y = N de infusiones
servidas por mesa hay una relacin lineal media y negativa, es decir, cuando aumenta el n de
cafs servidos disminuye el n de infusiones servidas por mesa.
b) p( X + Y 3) = p( X + Y = 3) + p( X + Y = 4) + p( X + Y = 5) =
= p(X = 3, Y = 0) + p(X = 2, Y = 1) + p(X = 1, Y = 2) + p(X = 3, Y = 1) +
+ p(X = 2, Y = 2) + p(X = 3, Y = 2) = 0.1 + 0.1 + 0 + 0.2 + 0 + 0 = 0.4
c)
( ) =
(

|
.
|

\
|
=
=
+
(

|
.
|

\
|
=
=
+
(

|
.
|

\
|
=
=
+
(

|
.
|

\
|
=
=
=
= 1 Y
3 X
p 3x
1 Y
2 X
p x 2
1 Y
1 X
p x 1
1 Y
0 X
p x 0
1 Y
X
E
2
1) Y ( p
1) Y 3 (X p
x 3
1) Y ( p
1) Y 2 (X p
x 2
1) Y ( p
1) Y 1 (X p
x 1 =
(

=
= =
+
(

=
= =
+
(

=
= =
=
37) Sea (X,Y) una variable aleatoria bidimensional con funcin de densidad conjunta:


=
resto 0
2 y x 1 k
y) (x, f
Y X,
a) Comprobar que el valor de k es 2.
b) Son independientes las variables X e Y?
c) Qu distribucin sigue la variable X? y cul la variable Y?
d) Calcular la funcin de densidad de la variable Z = 4 - 2X. Qu distribucin sigue la
variable Z?
e) Calcular: Var( 3X-2Y+1).
f) Calcular:
|
.
|

\
|
=3/2 Y
X
E .
SOLUCIN
a) Para comprobar que la constante vale 2:
( | k
2
1
2
x
2x k dx x) (2 k dx y k dx dy k 1
2 x
1 x
2 2 x
1 x
2 x
1 x
2 y
x y
2 x
1 x
2 y
x y
=
(

\
|
= = =
|
|
.
|

\
|
=
=
=
=
=
=
=
=
=
=
=
=
=

Estadstica Empresarial II 75
Como 1 =
2
1
k k = 2
b) Para ver si son independientes las variables X e Y, comprobaremos si se cumple la igualdad:
(y) f (x) f y) (x, f
Y X Y X,
= , para ello necesitamos calcular primero las marginales de X y de Y.
Funcin de densidad marginal de X
( |

=
=
=
=
= = =
2 y
x y
2 y
x y X
) x (2 2 y 2 dy 2 (x) f 1 < x < 2
luego

< <
=
resto el en 0
2 x 1 ) x 2 ( 2
(x) f
X
Funcin de densidad marginal de Y
( |

=
=
=
=
= = =
y x
1 x
y x
1 x Y
) 1 (y 2 x 2 dx 2 (y) f 1 < y < 2
luego

< <
=
resto el en 0
2 y 1 1) - y ( 2
(y) f
Y
Como (y) f (x) f y) (x, f
Y X Y X,
, entonces X e Y no son independientes.
c) X e(1,2,1,2)

< <

=
< <
=

resto 0
2 y 1 x) - (2 2
resto 0
b y a x) (b a) - (x
a) - (b
1
q) (p,
1
(x) f
1 q 1 p
1 - q + p
X

ya que
2
1!0!
2!
(2) (1)
2) (1
1) (2
1
(1,2)
1
1 - 2 + 1
= =

+
=

Y e(2,1,1,2)
76 Problemas de examen resueltos

< <

=
< <
=

resto 0
2 y 1 ) 1 (y 2

resto 0
b y a y) (b a) - (y
a) - (b
1
q) (p,
1
(y) f
1 q 1 p
1 - q + p
Y

ya que
2
1!0!
2!
(1) (2)
) 1 (2
) 1 - (2
1
(2,1)
1
1 - 2 + 1
= =

+
=

d) Z = 4 2X. Para calcular la funcin de densidad de la variable Z primero calculamos su funcin


de distribucin F
Z
(z).
|
.
|

\
|
=
|
.
|

\
|
=
|
.
|

\
|
= = =
2
z 4
F 1
2
z 4
X p 1
2
z 4
X p z) 2X (4 p z) (Z p (z) F
X Z
2
z
2
z 4
2 2
2
1
2
z 4
f
2
1
2
1
2
z 4
f
2
z 4
F 1 (z) F (z) f
X X X
'
Z Z
=
|
|
.
|

\
|
|
.
|

\
|
=
|
.
|

\
|
=
|
.
|

\
|
|
.
|

\
|
=

|
|
.
|

\
|
|
.
|

\
|
= =
1 < x < 2 1 <
2
z 4
< 2 2 < 4 - z < 4 -2 < -z < 0 0 < z < 2

< <
=
resto 0
2 z 0
2
z
(z) f
Z
Z e(2,1,0,2)

< <

=
< <

=

resto 0
2 z 0
2
z

resto 0
b y a z) (b a) - (z
a) - (b
1
q) (p,
1
(z) f
1 q 1 p
1 - q + p
Z
ya que
2
1
4
1
1!0!
2!
4
1
(1) (2)
) 1 (2
) 0 - (2
1
(2,1)
1
1 - 2 + 1
= =

+
=

e) Como sabemos la distribucin que siguen las variables X e Y aplico las formulas
correspondientes para calcular las varianzas y esperanzas
Si X e(p,q,a,b) E(X) = a + (b-a)
q p
p
+
Var(X) =
( )
2
2
q) (p 1) q (p
q p a b
+ + +

Estadstica Empresarial II 77
Como X (1,2,1,2) e Y (2,1,1,2), tenemos:
E(X) = 1 + ( 2 -1)
2 1
1
+
=
3
4
Var(X) =
18
1
2) (1 1) 2 (1
2 x 1 1) (2
2
2
=
+ + +

E(Y) = 1 + ( 2 -1)
1 2
2
+
=
3
5
Var Y =
18
1
2) (1 1) 2 (1
1 x 2 1) (2
2
2
=
+ + +

=
|
|
|
.
|

\
|
=
|
|
|
.
|

\
|
=

=
=
=
=
=
=
=
=
2 y
1 y
y x
1 x
y
- y
x
x
Y X,
dy dx y x 2 dy dx y) (x, f y x (XY) E
( | 2.25
2
y
4
y
dy 1) (y y dy x y
2 y
1 y
2 4
2 y
1 y
2
2 y
1 y
y x
1 x
2
=
(

\
|
= = =
=
=
=
=
=
=
=
=

Cov (X,Y) = E(XY) E(X)E(Y) = 2.25
|
.
|

\
|
3
5
x
3
4
=
36
1
Por lo tanto: Var(3X-2Y+1) = 3
2
Var(X) + 2
2
Var(Y) 2 x 3 x 2 x Cov (X,Y) =
= 9 Var(X) + 4 Var(Y) 12 Cov (X,Y) =
=
|
.
|

\
|
18
1
x 9 +
|
.
|

\
|
18
1
x 4
|
.
|

\
|
36
1
x 2 1 =
18
7
f) ( )

=
=
=
=
=
x
- x
3/2 Y
X
dx (x) f x
3/2 Y
X
E
Funcin de densidad condicionada de X por Y: (x) f
y Y X/ =
( ) y x 1
1 - y
1
1) - y ( 2
2
(y) f
y) (x, f
x f
Y
Y X,
y X/Y
= = =
=
siempre que 1 y 2

< <
=
=
resto el en 0
y x 1
1 - y
1
(x) f
y X/Y

< <
=
=
resto el en 0
3/2 x 1 2
(x) f
2 / 3 X/Y
78 Problemas de examen resueltos
( ) ( |
4
5
x dx 2 x dx (x) f x
3/2 Y
X
E
3/2 x
1 x
2
3/2 x
1 x
x
- x
3/2 Y
X
= = = =
=
=
=
=
=
=
=
=

38) Para subir a la cima en una estacin de ski, se necesita coger cada uno de los remontes de
dos pistas. El tiempo de espera en minutos para coger el remonte de cada pista es una variable
aleatoria con la siguiente funcin de densidad:

>
=

resto 0
0 x e
(x) f
x
X
a) Qu distribucin sigue la variable X?
b) Cul es la probabilidad de que el tiempo de espera para subir a la cima no sea mayor que
6?
c) Un grupo de 21 personas desciende por una de las pistas con intencin de volver a tomar el
remonte de esa pista. Cual es la probabilidad de que menos de dos personas de ese grupo
tengan que esperar para coger el remonte entre 3 y 4 minutos?
SOLUCIN
a) X (1,1) = (1), pues

>

=
>
=
resto 0
0 x e
resto 0
0 x e a
(x) f
x - x -a
X
b) Sean X = Tiempo de espera para coger el remonte de la pista 1
Y = Tiempo de espera para coger el remonte de la pista 2
Son variables independientes con la misma distribucin (1,1) = (1). Definimos la variable Z =
Tiempo de espera para subir a la cima = X + Y. Entonces por propiedades de la distribucin gamma
y dado que X e Y son independientes, la variable Z (1,2) con lo que su funcin de densidad es:

>

=
>

=
resto 0
0 z e z
resto 0
0 z e z
(p)
a
(z) f
z -
az - 1 - p
p
Z
pues

a
(p)

1
(2)

1
1!
1
p 2

= = =
Nos piden calcular la probabilidad de que el tiempo de espera para subir a la cima no sea mayor que
6, luego
Estadstica Empresarial II 79
( | = + = =

=
=

=
=

=
=

6 z
0 z
z
6 z
0 z
z
6 z
0 z
z
dz e e z dz e z ) 6 Z ( p
( | 9826487 . 0 e 7 1 e e 6
6
6 z
0 z
z 6
= = + =

=
=

c) Sea W = N de personas que esperan entre 3 y 4 minutos de entre las 21 personas que descienden
la pista.
W B(n,p) donde n = 21 y p = p(xito) = p(3 < X < 4)
p(3 < X < 4) = ( | 9 0.03147142 e e e dx e
4 3
4 x
3 x
x
4 x
3 x
x
= = =

=
=

=
=

W B(21, 0.031471429)
Nos piden la probabilidad de que menos de dos personas de ese grupo tengan que esperar para
coger el remonte entre 3 y 4 minutos:
p(W < 2) = p(W = 0) + p(W = 1) =
=
|
|
.
|

\
|
+
|
|
.
|

\
|
=
20 1 21 0
) 031471429 . 0 1 ( 031471429 . 0
1
21
) 031471429 . 0 1 ( 031471429 . 0
0
21
= 0.859575567.
39) Una empresa necesita personal auxiliar administrativo, para su seleccin realiza distintas
pruebas, la primera y eliminatoria consiste en escribir un texto a mquina durante un minuto,
pasa la prueba quien supere 240 pulsaciones correctas por minuto.
Un candidato da 3, 4, 5, 6 pulsaciones por segundo con unas probabilidades de 0.1, 0.3, 0.4 y
0.2 respectivamente, pero comete 0, 1, 2 errores por segundo, con probabilidades 1/5, 3/5 y 1/5
respectivamente. Calcular la probabilidad de que este candidato supere la primera prueba.
(Trabajar con dos decimales).
SOLUCIN
Consideremos X = N pulsaciones por segundo, su esperanza y su varianza es:
X 3 4 5 6
P( X = x) 0.1 0.3 0.4 0.2
E(X) =

=
=
6
3 x
x) p(X x = (3 x 0.1) + (4 x 0.3) + (5 x 0.4) + (6 x 0.2) = 4.7
80 Problemas de examen resueltos
E(
2
X ) =

=
=
6
3 x
2
x) p(X x = (3
2
x 0.1) + (4
2
x 0.3) + (5
2
x 0.4) + (6
2
x 0.2) = 22.9
Var(X) = E(
2
X ) - E
2
(X) = 22.9 4.7
2
= 0.81
y consideremos Y = N errores por segundo, su esperanza y su varianza es:
Y 0 1 2
P( Y = y) 1/5 3/5 1/5
E(Y) =

=
=
2
0 x
y) (Y p y =
|
.
|

\
|
+
|
.
|

\
|
+
|
.
|

\
|
5
1
x 2
5
3
x 1
5
1
x 0 = 1
E(
2
Y ) =

=
=
2
0 x
2
y) (Y p y =
|
.
|

\
|
+
|
.
|

\
|
+
|
.
|

\
|
5
1
x 2
5
3
x 1
5
1
x 0
2 2 2
=
5
7
Var(Y) = E(
2
Y ) - E
2
(Y) =
5
7
1
2
=
5
2
= 0.4
Sea W = N de pulsaciones correctas en un minuto =

= =

60
1 i
i
60
1 i
i
Y X
Nos piden: p(W > 240) = p(

= =

60
1 i
i
60
1 i
i
Y X > 240)
E(W) = E(

= =

60
1 i
i
60
1 i
i
Y X ) = ( )

=
60
1 i
i
X E ( )

=
60
1 i
i
Y E = (60 x 4.7) (60 x 1) = 222
Var (W) = Var (

= =

60
1 i
i
60
1 i
i
Y X ) = ( )

=
60
1 i
i
X Var + ( )

=
60
1 i
i
Y Var = (60 x 0.81) + (60 x 0.4) = 72.6
X e Y independientes
De la variable aleatoria W no conocemos su distribucin, pero si sabemos que es suma y resta de
variables aleatorias independientes idnticamente distribuidas con esperanza y varianza finitas, con
lo cual podemos aplicar el Teorema Central del Lmite, y aproximar el valor de dicha probabilidad.
p(W > 240) = 1 -
|
|
|
|
.
|

\
|

<

= |
.
|

\
|
<


= =
= =
72.6
222 240
72.6
222 Y X
p 1 240 ) Y X ( p
60
1 i
i
60
1 i
i
60
1 i
i
60
1 i
i
tipificando Aplicando el Teorema Central del Lmite
0174 . 0 9826 . 0 1 (2.11) 1 = =
Estadstica Empresarial II 81
40) En una feria, una churrera hace un estudio de las variables X = N de churros que
compra el cliente e Y = N de buuelos que compra el cliente, con la siguiente ley de
probabilidad conjunta:
X \ Y 0 6 12
6 0.2 0 0
12 0.4 0.1 0.1
18 0 0.1 0
24 0.1 0 0
a) Calcular el coeficiente de correlacin. Interpretar su resultado.
b) Calcular el nmero medio de buuelos comprados por un cliente que sabemos ha
comprado ya 12 churros.
SOLUCIN
a) Sean X = N de churros que compra el cliente, e Y = N de buuelos que compra el cliente.
Calculemos sus marginales, sus esperanzas, sus varianzas y la covarianza para poder obtener el
coeficiente de correlacin.
X \ Y 0 6 12 P(X = x)
6 0.2 0 0 0.2
12 0.4 0.1 0.1 0.6
18 0 0.1 0 0.1
24 0.1 0 0 0.1
P(Y = y) 0.7 0.2 0.1 1
E(X) = (6x0.2) + (12x0.6) + (18x0.1) + (24x0.1) = 12.6
E(X
2
) = (6
2
x0.2) + (12
2
x0.6) + (18
2
x0.1) + (24
2
x0.1) = 183.6
Var(X) = E(X
2
) - E
2
(X) = 183.6 - 12.6
2
= 24.84
E(Y) = (0x0.7) + (6x0.2) + (12x0.1) = 2.4
E(Y
2
) = (0
2
x0.7) + (6
2
x0.2) + (12
2
x0.1) = 21.6
Var(Y) = E(Y
2
) - E
2
(Y) = 21.6 - 2.4
2
= 15.84
E(XY) = (6x0x0.2) + (6x6x0) + (6x12x0) + (12x0x0.4) + (12x6x0.1) +(12x12x0.1) +
+ (18x0x0) + (18x6x0.1) + (18x12x0) + (24x0x0.1) +(24x6x0) + (24x12x0) = 32.4
Cov(X,Y) = E(XY) - E(X)E(Y) = 32.4 - (2.4x12.6) = 2.16
1088931 0.
15.84 x 24.84
2.16
Var(Y) Var(X)
Y) Cov(X,
Y X,
= = =
82 Problemas de examen resueltos
Como el coeficiente de correlacin es positivo, quiere decir que cuanto mayor es el nmero de
churros que compra el cliente, mayor es el nmero de buuelos que compra, y al revs. Aunque el
valor es tan pequeo, tan prximo a cero, que la correlacin entre el n de churros y el n de
buuelos que compra el cliente es prcticamente nula.
b) Empezaremos calculando la ley de probabilidad de Y/X=12.
6
4
0.6
0.4
(12) f
(12,0) f
12) X / (0 f
Y
Y X,
12 Y/X
= = = =
=
6
1
0.6
0.1
(12) f
(12,6) f
12) X / (6 f
Y
Y X,
12 Y/X
= = = =
=
6
1
0.6
0.1
(12) f
(12,12) f
12) X / (12 f
Y
Y X,
12 Y/X
= = = =
=
luego
Y/X=12 p(Y/X=12)
0 4/6
6 1/6
12 1/6
con lo cual su esperanza es:
E(Y/X=12) = (0x
6
4
) + (6x
6
1
) + (12x
6
1
) = 3.
41) Sean X e Y dos variables aleatorias continuas con funcin de densidad conjunta:


=
resto el en 0
3 x y 0 kx
y) (x, f
Y X,
a) Comprobar que la constante k vale
9
1
.
b) Obtener las funciones de densidades marginales de X e Y. Qu distribucin sigue la
variable X?
c) Obtener la funcin de densidad condicionada de Y por X. Qu distribucin sigue la
variable Y/X=x?
d) Calcular E(Y/X=1).
e) Calcular la p(X+Y 2).
SOLUCIN
a) Para comprobar que la constante vale 1/9, usaremos el hecho de que la integral en todo el
dominio de definicin de la funcin de densidad es 1.
Estadstica Empresarial II 83

=
=
=
=
=
=
=
=
=
=
=
|
|
.
|

\
|
=
(

\
|
=
(
(

=
3 y
0 y
2 2
3 y
0 y
3 x
y x
2
3 y
0 y
3 x
y x
dy
2
y
2
3
k dy
2
x
k dy dx k x 1
9
1
k k 9
6
y
y
2
9
k
3 y
0 y
3
= =
(

\
|
=
=
=
o bien:
( |
9
1
k k 9
3
x
k dx x k dx y x k dx dy x k 1
3 x
0 x
3
3 x
0 x
2 x y
0 y
3 x
0 x
3 x
0 x
x y
0 y
= =
(

\
|
= = =
(
(

=
=
=
=
=
=
=
=
=
=
=
=
=

b) Marginal de X

=
=
=
=
=
(

\
|
= =
x y
y 0
2
x y
0 y
X
3 x 0
9
x

9
y x
dy
9
x
(x) f
X e(3,1,0,3)

< <

=
< <
=

+
resto el en 0
3 x 0 x
9
1
resto el en 0
b x a x) (b a) - x (
a) - (b
1
q) (p,
1
(x) f
2
1 q 1 p
1 - q p
X

ya que
9
1
27
1
0! 1!
3!
27
1
(1) (3)
) 1 (3
) 0 3 (
1
(3,1)
1
1 1 3
= =

+
=

Marginal de Y

=
=
=
=
=
(

\
|
= =
3 2
3 x
y x
2
Y
3 y 0
18
y
-
2
1
18
x
dx
9
x
(y) f
x
y x
c) Funcin de densidad condicionada de Y por X: (y) f
x X Y/ =
84 Problemas de examen resueltos
( ) x y 0
x
1
9
x
9
x
(x) f
y) (x, f
y f
2
X
Y X,
x Y/X
= = =
=
siempre que 0 x 3
Y/X=x U(0,x)

< <
=

< <
=
=
resto el en 0
x y 0
x
1
resto el en 0
b y a
a - b
1
(y) f
x Y/X
d) Sustituyendo X = 1 en la funcin de densidad Y/X=x tenemos:
( ) 1 y f
1 Y/X
=
=
0 < y < 1
luego
2
1
2
y
dy 1 y 1) Y/X ( E
1 y
0 y
2
1 y
0 y
=
|
|
.
|

\
|
= = =
=
=
=
=

f)
p(X + Y 2) =

=
=
=
= (
(

1 y
0 y
y 2 x
y x
dy dx
9
x
=
=

=
(

\
|
=

=
=
=
=
=
=
dy
9
2 2
dy
18
x
1 y
0 y
y 2 x
y x
1 y
0 y
2
y
= ( |
9
1
y 2y
9
1 1
0
2
=
=
=
y
y
42) Del presupuesto destinado para las fiestas, se tiene que el gasto (en millones de ptas.) para
los fuegos artificiales de un da es una variable aleatoria cuya funcin de densidad es la
siguiente:


=
resto el en 0
1 x 0 x) (1 3
(x) f
2
X
a) Qu distribucin sigue dicha variable?
X
Y
1
X + Y = 2
Y = X
3
1
Estadstica Empresarial II 85
b) Calcular la probabilidad de que el gasto en los fuegos artificiales de un da sea inferior a
100 000 ptas.
c) Hay fuegos artificiales durante nueve das. Calcular la probabilidad de que al menos un da
de esos el gasto sea menor que 100 000 ptas.
SOLUCIN
a) X (1,3)

< <

=
< <
=

resto 0
1 x 0 x) - (1 3
resto 0
1 x 0 x) (1 x
q) (p,
1
(x) f
2
1 q 1 p
X

ya que
3
2! 0!
3!
(3) (1)
) 3 (1
(1,3)
1
= =

+
=

b) p(X< 0.1) = ( | 271 . 0 x) (1 dx x) (1 3


0.1 x
0 x
3
0.1 x
0 x
2
= =
=
=
=
=

c) Sea W = N de das en los que el gasto en fuegos artificiales es menor que 0.1 millones de entre
los nueve das
W B(n,p) donde n = 9 y p = probabilidad de xito = p( X < 0.1) = 0.271, luego
W B(9,0.271)
p(W 1) = 1- p(W < 0) = 1 p(W = 0) =
= 1 -
9 0
0.271) (1 271 . 0
0
9

|
|
.
|

\
|
= 1 - 0.0581497 = 0.9418503
43) Una compaa de teatro tiene dos obras en cartelera: Sueos y Pasiones, cada da
realiza una representacin de cada una. Las ganancias (en miles de pesetas) por funcin se
distribuye normalmente en cada caso.
Se sabe que la ganancia media por representacin de la obra de teatro Sueos es de 600 mil
ptas. con una desviacin tpica de 20, que el 2,28% de las ganancias obtenidas por
representacin de la obra Pasiones son superiores a 800 mil ptas. y que el 96,9% estn
comprendidas entre 690 y 800 mil ptas.
a) Calcular la media y la varianza de las ganancias obtenidas por representacin de la obra
Pasiones.
86 Problemas de examen resueltos
b) Qu distribucin sigue la ganancia total semanal (7 das) por la representacin de ambas
obras?
SOLUCIN
a) Sean X = Ganancia (en miles ptas.) por la representacin de la obra Sueos N(600,20)
Y = Ganancia (en miles ptas.) por la representacin de la obra Pasiones N(,)
Para determinar el valor de los parmetros disponemos de las siguientes igualdades:
p(B > 800) = 0.0228 p(690 < B < 800) = 0.969
Tipificando y despejando tenemos:
p(B > 800) = 0.0228 0.0228
800
1 =
|
.
|

\
|



0.9772
800
=
|
.
|

\
|


2
800
=


p(690 < B < 800) = 0.969
969 . 0
690
- 0.9772
690
-
800
= |
.
|

\
|


= |
.
|

\
|


|
.
|

\
|



0.0082
690
= |
.
|

\
|


4 . 2
690
=


y resolviendo el sistema:

800
= 2

690
= -2.4
se obtiene que
= 750 y = 25
luego Y N(750,25).
E(Y) = 750 Var(Y) = 25
2
= 625
Estadstica Empresarial II 87
b) Sea Z = X + Y N(750 + 600,
2 2
25 20 + ) = N(1350, 1025 ).
Definimos la variable:
G = Ganancia total semanal por la representacin de ambas obras =

=
7
1 i
i
Z
Aplicando las propiedades de la normal sabemos que
G 7175 9450, ( N ) 1025 x 7 1350, x N(7 ) n , N(n = = )
44) Una empresa de alquiler de vehculos es rentable si el nmero total de coches alquilados
en los 80 primeros das del ao es superior al nmero total de motos alquiladas en todo el ao
(360 das).
Al da, el nmero de coches que esa empresa alquila es de 10, 11, 12, 13 y 14 con unas
probabilidades de 0.2, 0.1, 0.4, 0.1 y 0.2 respectivamente, y el nmero de motos alquiladas,
tambin al da, es de 1, 2, 3 y 4 con probabilidades 0.2, 0.3, 0.3 y 0.2 respectivamente. Calcular
la probabilidad de que el negocio sea rentable.
SOLUCIN
Definimos las variables: X = N coches alquilados al da e Y = N motos alquiladas al da.
Calculemos sus medias y sus varianzas.
X 10 11 12 13 14
P(X = x) 0.2 0.1 0.4 0.1 0.2
E(X) =

=
=
14
10 x
x) p(X x = (10 x 0.2) + (11 x 0.1) + (12 x 0.4) + (13 x 0.1) + (14 x 0.2) = 12
E(
2
X ) =

=
=
14
10 x
2
x) p(X x = (10
2
x 0.2) + (11
2
x 0.1) + (12
2
x 0.4) + (13
2
x 0.1) + (14
2
x 0.2) = 145.8
Var(X) = E(
2
X ) - E
2
(X) = 145.8 12
2
= 1.8
Y 1 2 3 4
P(Y = y) 0.2 0.3 0.3 0.2
E(Y) =

=
=
4
1 x
y) p(Y y = (1 x 0.2) + (2 x 0.3) + (3 x 0.3) + (4 x 0.2) = 2.5
E(
2
Y ) =

=
=
4
1 x
2
y) p(Y y = (1
2
x 0.2) + (2
2
x 0.3) + (3
2
x 0.3) + (4
2
x 0.2) = 7.3
Var(Y) = E(
2
Y ) - E
2
(Y) = 7.3 2.5
2
= 1.05
88 Problemas de examen resueltos
El negocio es rentable si

= =
>
360
1 i
i
80
1 i
i
Y X
Sea W =

= =

360
1 i
i
80
1 i
i
Y X
Nos piden: p(W > 0) = p(

= =

360
1 i
i
80
1 i
i
Y X > 0)
E(W) = E(

= =

360
1 i
i
80
1 i
i
Y X ) = ( )

=
80
1 i
i
X E ( )

=
360
1 i
i
Y E = (80 x 12) (360 x 2.5) = 60
Var (W) = Var (

= =

360
1 i
i
80
1 i
i
Y X ) = ( )

=
80
1 i
i
X Var + ( )

=
360
1 i
i
Y Var = (80 x 1.8) + (360 x 1.05) = 522
X e Y independientes
De la variable aleatoria W no conocemos su distribucin, pero si sabemos que es suma y resta de
variables aleatorias independientes idnticamente distribuidas con esperanza y varianza finitas, con
lo cual podemos aplicar el Teorema Central del Lmite, y aproximar el valor de dicha probabilidad.
p(W > 0) = 1 -
|
|
|
|
.
|

\
|


= |
.
|

\
|



= =
= =
522
60 0
522
60 Y X
p 1 0 ) Y X ( p
360
1 i
i
80
1 i
i 360
1 i
i
80
1 i
i
tipificando Aplicando el Teorema Central del Lmite
(-2.63) 1 = 1 0.0043 = 0.9957.
45) El tiempo medido en horas, que un profesor emplea en resolver una duda a un alumno
por medio de tutora electrnica es una variable aleatoria que sigue una distribucin gamma
(2, ).
a) Si en un da resuelve ocho consultas, cul es la probabilidad de que tarde menos de una
hora y cuarto?. Cul es el tiempo medio que tarda en resolver ocho consultas?
b) Si durante el curso recibe 240 consultas, cul es la probabilidad de que emplee entre 25 y
35 horas en responder?
c) Durante todo el curso, este profesor ha realizado 40 horas de atencin al alumno mediante
el procedimiento de tutora electrnica. Cuntas consultas calculas que habr contestado
como mximo, con una probabilidad de 0.9772?
Estadstica Empresarial II 89
SOLUCIN
a) X = Tiempo empleado en contestar a una consulta (2, )
X
1
, X
8
i.i.d. (2, ) ( ) 2 , 2 X
8
1 i
i

=
por lo tanto:
( ) =
(

+ = = = |
.
|

\
|
<


=
1.25
0
2x
1.25
0
2x
1.25
0
2x
1.25
0
2x
8
1 i
i
dx e xe 2 dx 2xe 2 dx 4xe 1.25 X p
0.7127025 1 e e 2.5
2
e
2 e 2.5
2.5 - 2.5 -
1.25
0
2x
2,5
= + =
|
|
.
|

\
|
+ =

1
a
p
X E
8
1 i
i
= = |
.
|

\
|

=
El tiempo medio en resolver 8 consultas es una hora.
b) Sea Y = Tiempo empleado en responder 240 consultas = . X
240
1 i
i
=
X
1
, X
240
i.i.d. (2, ) con = E(X
i
) =
8
1
< y
2
= Var(X
i
) =
16
1
< , aplicando el
T. C. L.
) 1 , 0 N(
15
30 Y
L

( )
T.C.L. el por o tipificand
15
30 35
15
30 Y
15
30 25
p 35 Y 25 p

=
|
|
.
|

\
|
<

<

= < <
( ) ( ) ( ) 803 . 0 1 29 . 1 2 29 . 1 29 . 1
T.C.L. el por
= =


c) Y = Tiempo empleado en responder n consultas = ( )
4
, 2 X
n
1 i
i
n

=
.
Si el tiempo dedicado a tutora electrnica en todo el ao es 40 horas, cabe suponer que el nmero
de consultas sea suficientemente grande (n>30), y podemos aplicar el T. C. L.
X
1
, X
n
i.i.d. (2, ) con = E(X
i
) =
8
1
< y
2
= Var(X
i
) =
16
1
< , por el T. C. L.
1) N(0,
16 n
8 n X
L
n
1 i
i

=
( ) 0.9772
4 n
8 n 40
4 n
8 n Y
p 40 Y p
T.C.L. el por o tipificand

=
|
|
.
|

\
|

<

= <
0 320 - n 4 n 2
4 n
8 n 40
= + =

=
vlida 256
vlida no 400
n
16
20
n
90 Problemas de examen resueltos
46) Sea la variable aleatoria bidimensional (X, Y), donde X es el nmero de kg. de aceitunas e
Y es el nmero de kg. de pepinillos vendidos diariamente en una tienda de encurtidos.
Conocemos las siguientes funciones de densidad:
( ) 0 x 0
50
x 10
x f
X
1 < <

=
( ) 10 y x
x 10
1
y/x f
Y/X
< <

=
a) Calcular la funcin de densidad conjunta de (X, Y) y la marginal de Y.
b) Las marginales siguen alguna distribucin conocida? Son independientes X e Y?
c) Calcular la esperanza y la varianza del nmero total de kg. vendidos diariamente entre
aceitunas y pepinillos.
d) Calcular la probabilidad de que se vendan como mximo 10 kg. entre aceitunas y
pepinillos.
e) Calcular el nmero de kg. de pepinillos que se espera vender al cabo de un da, sabiendo
que se venden 4 kg. de aceitunas.
f) Si los pepinillos se venden a 1 000 ptas. el kg., qu distribucin siguen los ingresos diarios
por las ventas de este producto?
SOLUCIN
a) ( ) 10 y x 0
50
1
x 10
1
50
x 10
y x, f
xy
< < < =

=
( ) 10 y 0
50
y
50
x
dx
50
1
y f
y
0
y
0
Y
< < =
(

= =

b) X e(1, 2, 0, 10) e Y e(2, 1, 0, 10)
La constante en ambos casos va a ser la misma:
( )( )
( ) ( )
( )
50
1
100
2
10
3
2 1
1
a b q p,
1
1 2 1
1 q p
= =


=
+
+

X e Y son dependientes pues f


X
(x) f
Y
(y) f
XY
(x, y)
c) ( ) ( )
3
10
q p
p
a b a X E =
+
+ = ( ) ( )
3
20
q p
p
a b a Y E =
+
+ =
( )
( )
( )( )
9
50
q p 1 q p
pq a b
X Var
2
2
=
+ + +

= ( )
( )
( )( )
9
50
q p 1 q p
pq a b
Y Var
2
2
=
+ + +

=
( ) ( ) = =
(

= =

10
0
10
x
10
0
2
10
x
10
0
2
dx x 100 x
100
1
dx
2
y
50
x
dy dx
50
xy
XY E
Estadstica Empresarial II 91
25
4
x
50x
100
1
10
0
4
2
=
(

=
Por tanto, ( )
9
25
3
20
3
10
25 Y X, Cov = =
Esperanza del nmero total de kg. de aceitunas y pepinillos vendidos en un da:
( ) ( ) ( ) 10
3
20
3
10
Y E X E Y X E = + = + = + kg.
Varianza del nmero total de kg. de aceitunas y pepinillos vendidos en un da:
( ) ( ) ( ) ( )
3
50
9
25
2
9
50
9
50
Y X, 2Cov Y Var X Var Y X Var = + + = + + = +
d)
5
10
10 5
x = y
y = 10 -x
( ) =

=
(

= = < +

dx
50
2x 10
dx
50
y
dy dx
50
1
10 Y X p
5
0
x 10
x
5
0
5
0
x 10
x
| | | |
2
1
25 50
50
1
x 10x
50
1 5
0
2
= = =
e) ( ) 10 y 4
6
1
4 y/x f
Y/X
< < = =
( ) 7
2
10 4
4 Y/X E =
+
= = , por ser uniforme la funcin de densidad condicionada.
f) Sea Z = Ingresos por las ventas diarias de pepinillos = 1000 Y
( ) ( ) ( )
|
.
|

\
|
= |
.
|

\
|
= = =
1000
z
F
1000
z
Y p z Y 1000 p z Z p z F
Y Z
( ) 10.000 z 0
50.000.000
z
1000
1
1000
z
f z f
Y Z
< < = |
.
|

\
|
=
Por tanto, Z e(2, 1, 0, 10.000).
92 Problemas de examen resueltos
47) El consumo mensual de carne de vacuno por persona, en cuatro pases de la Comunidad
Europea antes de la crisis de las vacas locas, se distribua normalmente. Los datos de las
medias y desviaciones tpicas figuran en la siguiente tabla:
N Medio de kg. D. Tpica
Gran Bretaa 2.6 0.2
Francia 2.8 0.3
Alemania 3.6 0.6
Espaa 2.5 0.5
Se puede considerar que estas cantidades son independientes, por depender
fundamentalmente de los hbitos de consumo y costumbres gastronmicas de los distintos
pases. A raz del problema de las vacas locas, se ha observado que Gran Bretaa ha
disminuido su consumo en un 30%, Francia en un 10%, Alemania en un 15% y Espaa en un
8%.
a) Cul es la probabilidad de que actualmente un individuo en Gran Bretaa consuma
menos de 2.1 kg. de carne de vacuno al mes?
b) Cul es la probabilidad de que un alemn actualmente consuma menos carne de vaca que
un espaol antes de la crisis?
c) A un curso de un mes de duracin programado para el verano de 2001, piensan acudir un
estudiante de cada uno de estos cuatro pases. Cul es la probabilidad de que el consumo de
los cuatro est entre 9 y 10.4 kg. de carne de vaca?
SOLUCIN
Sean las variables:
X = Consumo mensual en kg. por persona, de carne de vaca en Gran Bretaa antes de la crisis.
Y = Consumo mensual en kg. por persona, de carne de vaca en Francia antes de la crisis.
Z = Consumo mensual en kg. por persona, de carne de vaca en Alemania antes de la crisis.
T = Consumo mensual en kg. por persona, de carne de vaca en Espaa antes de la crisis.
X N(2.6, 0.2) Y N(2.8, 0.3) Z N(3.6, 0.6) T N(2.5, 0.5)
a) ( ) ( ) ( ) 9772 . 0 2
0.2
2.6 3
0.2
2.6 X
p 3 X p 2.1 X 0.7 p
o tipificand
= = |
.
|

\
|
<

= < = <


b) Debemos considerar la variable 0.85Z T, que por ser una combinacin lineal de variables
normales independientes, sigue una distribucin normal de parmetros:
0.85Z T N(0.85 x 3.6 2.5,
2 2 2
5 . 0 6 . 0 85 . 0 + ) N(0.56, 5101 . 0 )
( ) ( ) 2177 . 0 78 . 0
0.5101
0.56
0.5101
0.56 T Z 0.85
p 0 T Z 0.85 p = =
|
|
.
|

\
|
<

= <
c) Sea la variable S = 0.70 X + 0.9 Y + 0.85 Z + 0.92 T. Por ser combinacin lineal de variables
aleatorias independientes y normales, sigue una distribucin normal.
( ) 9.7 2.5 x 0.92 3.6 x 0.85 2.8 x 0.9 2.6 x 0.7 S E = + + + =
Estadstica Empresarial II 93
( ) 0.5642 0.25 x 0.92 0.36 x 0.85 0.09 x 0.9 0.04 x 0.7 S Var
2 2 2 2
= + + + =
Entonces, ( ) 0.5642 9.7, N S
( ) =
|
|
.
|

\
|
<

<

= < <

0.5642
9.7 10.4
0.5642
9.7 S
0.5642
9.7 9
p 5 . 10 S 9 p
o tipificand
( ) 0.6476 1 0.8238 2x 1 93 . 0 2 = = =
48) El dimetro de las piezas que fabrica una mquina es una variable aleatoria X, con
distribucin uniforme. Se sabe que el dimetro posible ms pequeo es 50 y que el dimetro
medio es 53 mm.
a) Calcular la distribucin de la v.a. X.
b) Una pieza es apta para su venta si su dimetro est comprendido entre 51 y 55 mm.
Calcular la probabilidad de que una pieza sea defectuosa.
c) Si en un da se fabrican 10 piezas, calcular la probabilidad de que al menos 8 sean
correctas.
SOLUCIN
a) X U(a, b), con a = 50. ( ) 53
2
b a
X E =
+
= ; luego b = 56.
b) p(Defectuosa) = 1 - ( ) = = < <

55
51
dx
6
1
1 55 X 51 p
3
1
3
2
1
6
51 55
1
6
x
1
55
51
= =

=
(

= .
c) Y = N de piezas correctas en un da. Y B(10, 2/3)
( ) ( ) ( ) ( ) 10 Y p 9 Y p 8 Y p 8 Y p = + = + = = =
=
0 10 1 9 2 8
3
1
3
2
10
10
3
1
3
2
9
10
3
1
3
2
8
10
|
.
|

\
|
|
.
|

\
|
|
|
.
|

\
|
+ |
.
|

\
|
|
.
|

\
|
|
|
.
|

\
|
+ |
.
|

\
|
|
.
|

\
|
|
|
.
|

\
|
= 2991413 . 0
3
2
3
2
10
3
2
45
10
10
10
9
10
8
= + + .
49) En un determinado sector de empresas, consideramos las variables aleatorias, X = N de
empleados fijos e Y = N de empleados temporales, con la siguiente ley de probabilidad
conjunta:
X\Y 0 1 2
1 2/9 1/9 0
2 2/9 2/9 0
3 0 1/9 1/9
94 Problemas de examen resueltos
a) Son X e Y independientes?
b) Calcular .
1 Y
3 Y X
p
|
.
|

\
|

+
c) Calcular el coeficiente de correlacin de X e Y.
SOLUCIN
X\Y 0 1 2 p(X=x)
1 2/9 1/9 0 3/9
2 2/9 2/9 0 4/9
3 0 1/9 1/9 2/9
p(Y=y) 4/9 4/9 1/9
a) No son independientes puesto que p(X=x, Y=y) p(X=x). p(Y=y)
b)
|
.
|

\
|

+
1
3
Y
Y X
p =
( )
=

+
) 1 (
1 ; 3
Y p
Y Y X p
=
= + =
= = + = = + = = + = = + = =
=
) 1 ( ) 0 (
) 0 , 3 ( ) 1 , 2 ( ) 0 , 2 ( ) 1 , 1 ( ) 0 , 1 (
Y p Y p
Y X p Y X p Y X p Y X p Y X p
7/8
c)
9
17
9
2
3
9
4
2
9
3
1 ) ( = + + = X E
9
37
9
2
3
9
4
2
9
3
1 ) (
2 2 2 2
= + + = X E
81
44
9
17
9
37
) (
2
= |
.
|

\
|
= X Var
3
2
9
1
2
9
4
1
9
4
0 ) ( = + + = Y E
9
8
9
1
2
9
4
1
9
4
0 ) (
2 2 2 2
= + + = Y E
9
4
3
2
9
8
) (
2
=
|
.
|

\
|
= Y Var
9
14
9
1
. 2 . 3
9
1
. 1 . 3
9
2
. 1 . 2
9
1
. 1 . 1 ) ( = + + + = XY E
27
8
3
2
9
17
9
14
) , ( = = Y X Cov 603022 . 0
9
4
81
44
27
8
) , ( = = Y X .
Entre X e Y hay una relacin directa.
50) Un agente decide invertir en unos fondos con inters variable, sin riesgo. El
comportamiento de las ganancias obtenidas (X) sigue una distribucin con funcin de
densidad.
( )
( )

< <
=
resto el en 0
2 x 0 x 2 x
4
3
x f
X
Estadstica Empresarial II 95
Esta ganancia es reinvertida en fondos con inters variable, en esta ocasin con riesgo de
prdida. Dependiendo del valor de las ganancias dadas por X, las nuevas ganancias/prdidas
(Y) siguen una distribucin con funcin de densidad:
( ) y f
x X
Y
=
=

< <
resto el en 0
2x y x
3x
1
a) Indicar cul sera la distribucin de las ganancias X.
b) Describir el comportamiento de la reinversin, en funcin de los posibles valores que toma
y de su distribucin (variable
x X
Y
=
).
c) Por trmino medio, si las ganancias han sido de 1 u.m., cunto cabe esperar que ganemos
tras reinvertirlas?
d) Obtener la covarianza entre las dos variables, explicando el resultado. Para ello, tener en
cuenta que:
( )
( )
( )

< <
< < +
=
resto el en 0
4 y 0 4 y
32
1
0 y 2 2 y
8
1
y f
2
2
Y
SOLUCIN
a) X
e
(2,2,0,2)
b)
x X
Y
=
U(-x, 2x). Esto significa que podemos perder una cantidad x ganar una cantidad 2x,
teniendo en cuenta que la distribucin es uniforme, es decir, para intervalos con la misma longitud,
la probabilidad es la misma. En nuestro caso, la probabilidad de perder x de ganar x sera la
misma (estamos considerando los intervalos (-x,0) y (0,x).
c)
x X
Y
=
U(-x, 2x). Por tanto, una de las formas de obtener esta esperanza es:
| |
2 2
2
2
x x x b a
x X
Y
E =
+
=
+
=
=
. En el caso concreto en que X = 1, el valor medio es 0.5.
d) | | X E = ( ) a a b
q p
p
+
+
= ( ) 0 0 2
2 2
2
+
+
= 1
| | Y E = ( ) dy y y

+
0
2
2
2
8
1
+ ( ) dy y y


4
0
2
4
32
1
= dy
y y y

+ +
0
2
2 3
8
4 4
+ dy
y y y

+
4
0
2 3
32
16 8
=
=
0
2
2
3 4
2
3
4
4 8
1

+ + y
y y
+
4
0
2
3 4
8
3
8
4 32
1
(

+ y
y y
=
6
1
+
3
2
=
2
1
.
96 Problemas de examen resueltos
( ) y x f
XY
, = ( ) ( ) x f y f
X
x X
Y
=
= ( ) x x
x
2
4
3
3
1
=

< < < <

resto el en
x y x x
x
0
2 , 2 0
4
2
| | XY E = dx dy
x
xy
x
x

|
.
|

\
|
2
0
2
4
2
= dx dy
x x
y
x
x

|
|
.
|

\
|
2
0
2
2
4
2
= dx dy
y x x
x
x

|
|
.
|

\
|
2
0
2
2 2
2 4
2
=
= dx dy
x x x

|
|
.
|

\
|
2
0
2 2
2
3
4
2
= dx dy
x x

|
|
.
|

\
|
2
0
4 3
8
3 6
=
2
0
5 4
8
5
3
4
6
(
(


x x
=
5
3
= 0.6.
Entonces, Cov(X,Y) = | | XY E - | | X E | | Y E =
5
3
-1 x
2
1
=
10
1
. Si las ganancias en el primer caso son
elevadas, parece ser que las ganancias/prdidas de la reinversin tienden a ser elevadas (signo
positivo de la covarianza), aunque tendramos que conocer el coeficiente de correlacin para poder
dar un grado de asociacin entre las dos variables.
51) Una empresa se dedica a la venta de un producto que ofrece al pblico en un envase de
metal. Se sabe que el peso del contenido es una variable aleatoria, X, con distribucin normal
de parmetros 1000 y 10 gramos. Por otra parte, el peso del envase es otra v.a., Y, tambin
normal e independiente de X, con parmetros 20 y 2 gramos.
a) Calcular la distribucin de la v.a. peso total del producto.
b) Para que el producto cumpla con las especificaciones legales, su peso no debe ser menor de
1005 gramos. Calcular (con dos decimales) la probabilidad de que el producto cumpla las
especificaciones.
c) Si tenemos 20 productos. Calcular la probabilidad de que, como mucho, 2 de ellos sean
defectuosos.
SOLUCIN
a) X N(1000, 10); Y N(20, 2); X e Y independientes. Peso total = T = X + Y.
T N(1020, 104 )
E(T) = E(X) + E(Y) = 1000 + 20 = 1020
Var(T) = Var(X) + Var (Y) = 100 + 4 = 104; 104 =
T

b) p (el producto cumpla las especificaciones) = p(T 1005) = 1- p(T < 1005) =
= 1 -
|
|
.
|

\
|
<

104
1020 1005
104
1020 T
p = 1- (-1.47) = 1- 0.07 = 0.93 (con dos decimales)
c) Sea Z = n de productos defectuosos en los 20. Z B(20, 0.07)
p (defectuoso) = 0.07
p(Z 2) = p(Z = 0) + p(Z = 1) + p(Z = 2) = =
|
|
.
|

\
|
+
|
|
.
|

\
|
+
|
|
.
|

\
|
18 2 19 1 20 0
93 . 0 07 . 0
2
20
93 . 0 07 . 0
1
20
93 . 0 07 . 0
0
20
= 0.8389971.
Estadstica Empresarial II 97
52) Una compaa de alquiler de vehculos ha determinado que la probabilidad de que un
coche necesite una revisin en un mes es 0.2. La compaa tiene 1000 automviles. Si
suponemos que la distribucin del nmero de revisiones es una binomial:
a) Calcular la probabilidad aproximada de que el nmero de vehculos revisados est
comprendido entre 179 y 221. Comentar todos los pasos a seguir a la hora de buscar esta
aproximacin y el resultado compararlo con la probabilidad exacta que toma el valor 0.90303.
b) Teniendo en cuenta el nmero medio de vehculos revisados y la probabilidad obtenida en
el apartado anterior, cmo cabra esperar que fuese la probabilidad de que el nmero de
vehculos revisados fuese superior a 250? Razonar la respuesta, puesto que no hace falta
realizar ninguna operacin adicional.
SOLUCIN
a) Para cada vehculo, definimos una variable de Bernoulli


=
contario caso 0
revisado es simo i vehculo el 1
X
i
Son variables B(1,p), donde p es la probabilidad de ser revisado, es decir, 0.2. La suma de estas
variables,
i
X sigue una distribucin binomial B(n,p). En este caso n = 1000 (n 30) y p = 0.2.
Para calcular la probabilidad pedida, podemos aplicar el Teorema Central del Lmite. Tan slo
necesitamos conocer la esperanza del total de vehculos revisados, es decir
i
X , y su varianza.
| |
i
X E = np = 1000 x 0.2 = 200
| |
i
X Var = npq = 1000 x 0.2 x 0.8 = 160
| |

< < 221 179


i
X p
Tipif
=
(
(


<

<

160
200 221
160
200
160
200 179
i
X
p
. . . L C T
= | | 66 . 1 66 . 1 < < Z p =
= | | 66 . 1 < Z p - | | 66 . 1 < Z p = 0.9515 0.0485 = 0.903. La aproximacin es casi perfecta.
b) La probabilidad dentro del intervalo [179, 221] es elevada: | |

< < 221 179


i
X p 0.9. Esto
indica que, para un nmero superior a 221, como es 250, la probabilidad | |

> 250
i
X p ser muy
baja.
53) Una librera ha observado que la densidad conjunta de sus ventas mensuales en miles de
euros, correspondientes al Diccionario de la Real Academia en la edicin de sencilla (X) y en
la edicin de lujo (Y), es de la forma:
( ) 6 x y 0
36
y
y x, f
XY
< < < =
98 Problemas de examen resueltos
a) Calcular las marginales. Alguna de ellas sigue una distribucin de tipo conocido? Son
independientes las variables X e Y?
b) Calcular las esperanzas y varianzas marginales y probar que Cov(X,Y) = 0.9.
c) Cules son las ventas esperadas en la edicin de lujo, sabiendo que las ventas
correspondientes a la edicin sencilla han sido 3000 ?
d) Si el 40% de las ventas de la edicin sencilla y el 20% de las ventas de la edicin de lujo se
realizan por internet, cul es la esperanza y varianza de las ventas realizadas por este medio?
SOLUCIN
a)
( ) 6 0
72 72 36
2
0
2
0
< < =
(

= =

x
x y
dy
y
x f
x
x
X
( ) | |
( )
6 0
36
6
36 36
6
6
< <

= = =

y
y y
x
y
dx
y
y f
y
y
Y
Las variables son dependientes porque ( ) ( ) ( ) y f x f y x, f
Y X XY

X e (3, 1, 0, 6) Y e (2, 2, 0, 6)
b) ( ) 5 . 4 X E = ( ) 3 Y E =
( ) 1.35
20
27
X Var = = ( ) 1.8
5
9
Y Var = =
( ) | |
| |
14.4
540
x
dx
108
x
dx y
108
x
dxdy
36
xy
XY E
6
0
5
6
0
4
x
0
3
6
0
6
0
x
0
2
= = = = =

Cov(X,Y) = E(XY) E(X)E(Y) = 14.4 4.5x3 = 0.9
c) ( )
( )
( )
x y 0
x
2y
72
x
36
y
x f
y x, f
y/x f
2 2
X
XY
Y/X
< < = = =
Concretamente, si x = 3, la funcin de densidad condicionada es:
( ) 3 y 0
9
2y
x
2y
y/x f
2
Y/X
< < = =
6
6
y=x
Estadstica Empresarial II 99
( ) | | 2 y
27
2
dy
9
2y
3 Y/X E
3
0
3
3
0
2
= = = =

por lo que se espera vender 2000 por trmino medio de la edicin de lujo, cuando se han vendido
3000 de la edicin sencilla.
d) Z = Ventas por internet = 0.40 X + 0.20 Y
E(Z) = 0.40 E(X) + 0.20 E(Y) = 0.40 x 4.5 + 0.20 x 3 = 2.4
Var(Z) = 0.40
2
Var(X) + 0.20
2
Var(Y) + 2 x 0.20 x 0.40 Cov(X,Y) = 0.432
54) Las distribuciones de las variables aleatorias, X e Y, n de enciclopedias vendidas al da
por dos vendedores, en zonas independientes de una ciudad, son:
X 0 1 2 3 Y 0 1 2
P(X = x
i
) 0.2 0.4 0.3 0.1 P(Y = y
j
) 0.5 0.3 0.2
a) Calcular la ley de probabilidad conjunta de X e Y.
b) Calcular la probabilidad de que, entre los dos vendedores, en un da vendan ms de 3
enciclopedias.
c) Si el primer vendedor vende sus enciclopedias a 300 la unidad y el otro a 400 , calcular la
media y desviacin tpica de los ingresos obtenidos al da en total.
SOLUCIN
a) Por ser independientes: p (X = x
i
; Y = y
j
) = p(X = x
i
) . p/(Y = y
j
) i, j
X \ Y 0 1 2 P(X = x
i
)
0 0.10 0.06 0.04 0.2
1 0.20 0.12 0.08 0.4
2 0.15 0.09 0.06 0.3
3 0.05 0.03 0.02 0.1
P(Y = y
j
) 0.5 0.3 0.2 1
b) P(X + Y > 3) = p(X = 2; Y = 2) + p(X = 3; Y = 1) + p(X = 3; Y = 2) = 0.06 + 0.03 + 0.02 = 0.11
c) I = Ingresos diarios = 300 X + 400 Y
E ( X ) = 0. 0.2 + 1. 0.4 + 2. 0.3 + 3. 0.1 = 1.3
E ( X
2
) = 0
2
. 0.2 + 1
2
. 0.4 + 2
2
. 0.3 + 3
2
. 0.1 = 2.5
Var( X ) = E( X
2
) (E( X ))
2
= 2.5 1.3
2
= 0.81
E ( Y ) = 0. 0.5 + 1. 0.3 + 2. 0.2 = 0.7
E ( Y
2
) = 0
2
. 0.5 + 1
2
. 0.3 + 2
2
. 0.2 = 1.1
100 Problemas de examen resueltos
Var( Y ) = E( Y
2
) (E( Y ))
2
= 1.1 0.7
2
= 0.61
E ( I ) = 300.E ( X ) + 400.E ( Y ) = 300. 1.3 + 400. 0.7 = 670
Por ser X e e independientes, Cov (X, Y ) = 0, por tanto:
Var ( I ) = 300
2
.Var ( X ) + 400
2
.Var ( Y ) = 300
2
.0.81 + 400
2
. 0.61 = 170500
170500 =
I
= 412.91646
55) Una fbrica de alfombras confecciona determinados modelos a medida. La superficie de
las alfombras que los clientes demandan es una variable aleatoria que sigue una distribucin
uniforme entre 3 y 6 m
2
. El mes pasado fabricaron 81 alfombras.
a) Si el precio de venta es de 12 el m
2
. De alfombra, calcular la probabilidad de que el valor
total de las ventas del ltimo mes supere los 4500 .
b) Se desea estudiar el tipo de alfombra que suele solicitar el cliente. Para ello se calcula la
media aritmtica de la superficie de las ochenta y una alfombras. Con qu probabilidad
estar comprendida entre 4.2 y 4.8 m
2
.?
SOLUCIN
a) T = Cantidad de m
2
. de alfombra vendidos el mes pasado =

=
81
1 i
i
X .
Las variables aleatorias X
1
, X
2
, , X
81
son independientes idnticamente distribuidas segn una
U(3, 6) y por lo tanto tienen esperanza y varianza finitas,
( ) < =
+
= 4.5
2
X E
i
b a
( )
( )
< =

= 0.75
12
X Var
2
i
a b
por el Teorema Central del Lmite, la suma de estas variables, tipificada, converge en ley a la N(0,
1).
( ) ( ) 5 . 64 3 4.5 x 81 X E X E T E
81
1 i
i
81
1 i
i
= = = |
.
|

\
|
=

= =
( ) ( ) 75 . 0 6 0.75 x 81 X Var X Var T Var
81
1 i
i
.
81
1 i
i
= = = |
.
|

\
|
=

= =
indep
La probabilidad pedida es:
( ) ( ) =
|
|
.
|

\
|
<

= > = >
60.75
5 . 64 3 75 3
60.75
5 . 64 3 T
p 1 375 T p 4500 12T p
o tipificand
( ) 0.0885 0.9115 1 1.35 1
. porelT.C.L
= = =
Estadstica Empresarial II 101
b) X = Superficie de las alfombras U(3, 6)
Y = Superficie media =
81
X
X
81
1 i
i
=
=
Las variables aleatorias X
1
, X
2
, , X
81
son independientes idnticamente distribuidas con
esperanza y varianza finitas, luego por el T.C.L, la media aritmtica de estas variables, tipificada,
converge en ley a la N(0, 1).
( ) ( ) 4.5 X E X E
i
= = ( )
( )
108
1
n
X Var
X Var
i
= =
( ) ( ) 1 0, N 108 4.5 X
L

( ) ( ) ( ) ( ) ( ) = < < = < < 108 4.5 4.8 108 4.5 X 108 4.5 4.2 p 4.8 X 4.2 p
o tipificand
( ) ( ) ( ) 0.9982 1 0.9991 x 2 1 3.12 2 3.12 3.12
. porelT.C.L
= = = = .
56) En una fbrica donde se hacen cajas de cartn, tenemos en cuenta las variables:
X = cantidad de cartn fabricada diariamente (en kg.) e Y = cantidad de cartn que, al
final del da, se ha encontrado en mal estado (en kg.) (deteriorado por la fabricacin, el
transporte, la humedad, etc.). Sabemos que siguen una distribucin normal, con medias 1500,
20, respectivamente y varianzas, 24 y 10, respectivamente.
a) Supongamos que el control de calidad exige que, cada da, la cantidad de cartn en mal
estado no supere los 22 kg. Consideremos un periodo de un mes (30 das). Calcular la
probabilidad de que el nmero de das en los que la cantidad de cartn defectuoso no supere
los 22 kg. sea inferior a 2.
b) Supongamos que Cov(X,Y) = 11. Si T es la variable cantidad de cartn que al final de la
jornada se encuentran en perfecto estado (en kg.), con distribucin normal, obtener su
esperanza y su varianza.
SOLUCIN
a) Sabemos que X N(1500, 24 ) y que Y N(20, 10 ).
Entonces: p[Y < 22] = p[Z <
10
20 22
= 0.63] = 0.7357, siguiendo Z una distribucin normal
tipificada. sta sera la probabilidad de nuestro xito. En 30 das, el nmero de das en los que se
verifica el xito sigue una distribucin binomial:
N = n de das en un mes en los que la cantidad de cartn defectuosa no supera los
22 kg. B(30, 0.7357).
La probabilidad que nos piden es: p[N < 2] = p[N 1] = p[N = 0] + p[N = 1] =
102 Problemas de examen resueltos
= 7357 0 x 2643 0
1
30
7357 0 x 2643 0
0
30
29 0 30
. . . .
|
|
.
|

\
|
+
|
|
.
|

\
|
0.
a) Cov(X,Y) = 11.
T = X Y E[T] = E[X Y] = E[X] E[Y] = 1500 20 = 1480.
Var[T] = Var[X Y] = Var[X] + Var[Y] 2 Cov(X,Y) = 24 + 10 2x11 = 12.
57) Sea (X, Y) una v.a. bidimensional con funcin de densidad conjunta;

< < < <


=
resto el en 0
2 y 0 1; x 0 y x k
y) (x, f
2
Y X,
a) Obtener el valor de k, y comprobar si estas variables son o no independientes. Qu
distribucin sigue cada una de las variables?
b) Obtener la funcin de densidad de Z = 3Y + 2. Sigue algn tipo de distribucin conocida?
c) Calcular Var(3X 2Y).
SOLUCIN
a) Para que
XY
f sea funcin de densidad debe cumplir que:
1 dy dx y) (x, f
Y X,
=


por tanto:
1 k
3
2
3
x
2k dx 2 x k dx
2
y
x k dx dy y kx
1
0
3 1
0
2
1
0
2
0
2
2
1
0
2
0
2
= =
(

= =
(

=
(


de donde
2
3
k =
Calculamos las marginales de X e Y:
1 x 0 3x
2
y
x
2
3
dy y x
2
3
y)dy (x, f (x) f
2
2
0
2
2
2
0
2
Y X, X
< < =
(

= = =


2 y 0
2
y
3
x
y
2
3
dy y x
2
3
y)dx (x, f (y) f
1
0
3 1
0
2
Y X, Y
< < =
(

= = =


La distribucin de X puede comprobarse que es (3,1)
La distribucin de Y es una e(2,1,0,2)
Las variables son independientes, puesto que ( ) ( ) ( ) y f x f y x,
Y X XY
= f , para todo valor de x e y.
Estadstica Empresarial II 103
b) Sea Z = 3Y + 2
c)
|
.
|

\
|
=
|
.
|

\
|
= + = =
3
2 z
F
3
2 z
Y p z) 2 p(3Y z) p(Z (z) F
Y Z

<

<

=
|
.
|

\
|
= =
resto el en 0
2
3
2 z
0
18
2 z
3
1
3
2 z
f
dz
(z) dF
(z) f
Y
Z
Z
, esto es:

< <

=
resto el en 0
8 z 2
18
2 z
(z) f
Z
, es decir, sea una e(2,1,2,8)
d) Var(3X 2Y) por ser X e Y independientes es: Var(3X 2Y) = 3
2
Var(X) + 2
2
Var(Y)
Calculamos las varianzas de X e Y.
Puesto que conocemos su distribucin:
80
3
q) 1)(p q (p
pq
Var(X)
2
=
+ + +
= ;
9
2
q) 1)(p q (p
pq a) (b
Var(Y)
2
2
=
+ + +

= , de donde
720
883
2Y) Var(3X = .
58) Se consideran las variables X = Tiempo de espera de conexin a la red, expresada en
minutos, e Y = Tiempo que se permanece conectado, tambin en minutos. Tras observar su
comportamiento durante cierto periodo de tiempo, se llega a la conclusin de que su
distribucin conjunta es:
X \ Y 60 120 180
1 0.04 0.08 0.30
5 0.02 0.20 0.19
30 0.10 0.02 0.05
a) Calcular la probabilidad de que un usuario est frente al ordenador dos horas y media
ms, desde que se intenta conectar hasta que apaga el ordenador.
b) Son independientes las variables? Caso contrario, calcular la covarianza e comentar su
significado.
c) Suponiendo que el tiempo de conexin ha sido de 30 minutos, ni ms ni menos, indicar, por
trmino medio, cul sera el tiempo que se ha estado conectado.
SOLUCIN
a) Definimos la variable X + Y = Tiempo que un usuario est frente al ordenador, desde que se
intenta conectar hasta que desconecta. Esta variable viene expresada en minutos. Entonces:
| | 150 Y X p + = | | 130 Y 30, X p = = + | | 180 Y 1, X p = = + | | 180 Y 5, X p = = +
+ | | 180 Y 30, X p = = = 0.02 + 0.30 + 0.19 + 0.05 = 0.56.
104 Problemas de examen resueltos
b) | | 60 1 = = Y , X p = 0.04 | | 1 = X p | | 60 = Y p = 0.42 x 0.16 = 0.0672.
X \ Y 60 120 180 | | x X p =
1 0.04 0.08 0.30 0.42
5 0.02 0.20 0.19 0.41
30 0.10 0.02 0.05 0.17
| | y Y p = 0.16 0.30 0.54 1
E[X] = | |

= x X p x = 7.57 E[Y] = | |

= y Y p y = 142.8
E[XY] = | |

= =
i j
j i j i
y ,Y x X p y x = 885
Cov(X,Y) = E[XY] E[X] E[Y] = 885 7.57 x 142.8 = -195.996. La relacin entre las variables
es inversa. Cuanto ms tardemos en conectarnos, ms pronto lo dejaremos.
c) La distribucin condicionada
30 = X
Y
:
30 = X
Y
(

=
=
30 X
y Y
p
60 10/17
120 2/17
180 5/17
1
| |
30 = X
Y
E =
17
5
180
17
2
120
17
10
60 + + =
17
1740
= 102.3529
59) Un sistema electrnico compuesto por 30 componentes independientes acta de la forma
siguiente: al principio slo funciona el primer componente y cuando ste falla, empieza a
funcionar automticamente el segundo y as sucesivamente hasta el ltimo, de forma que el
sistema slo deja de funcionar cuando han fallado los 30 componentes de forma sucesiva.
Sabiendo que la duracin (en horas) de cada componente es una v. a. con distribucin
exponencial y tal que p( X 1) = 0.6321205:
a) Comprobar que el valor del parmetro a es 1.
b) Calcular la duracin media del sistema.
c) Calcular de forma exacta (plantear) y aproximada la probabilidad de que la duracin del
sistema sea mayor que 31 horas.
SOLUCIN
a) Sea X = Duracin (en horas) de un componente. Sabemos que X sigue una distribucin (a),
por tanto:

<

=

0 0
0
x
x ae
f
ax
) x ( X
Estadstica Empresarial II 105
Luego: | | . . e e dx ae ) X ( p
a ax ax
6321205 0 1 1
1
0
1
0
= = = =

Despejando a = -ln(0.3678795) = 1.
b) D = Duracin total del sistema = X
1
+ X
2
++ X
30
. Puesto que 1
1
= =
a
) X ( E
E(D) = E(X
1
) + E(X
2
)

++ E(X
30
) = 30 x 1 = 30
Hallamos la distribucin de D. Puesto que X sigue una distribucin (a), es decir una (a,1), por la
propiedades de la distribucin gamma:
) , ( ) ... , ( X D
i
i
30 1 una decir es 1 1 1 1
30
1
+ + + =

=
por tanto, de forma exacta:

= >
31
1 30
30
30
1
31 dx e x
) (
) D ( p
x
Como hay que calcular esta integral, podemos hacerlo de forma aproximada utilizando el Teorema
Central del Lmite, puesto que:
) ) n , n ( N X D
i
i
30 N(30, caso este en
30
1
=

=
de donde:
4286 0 5714 0 1 18 0 1 1825 0
30
30 31
30
30
31 . . ) . ( ) . Z ( p
D
p ) D ( p = = = >
|
|
.
|

\
|
>

= >
60) La cantidad diaria de leche que se sirve en una cafetera se comporta con independencia a
lo largo de los distintos das del ao. De lunes a viernes sigue una distribucin normal de
media 50 litros y desviacin tpica 10 litros. Los sbados y domingos, dicha cantidad se
incrementa en un 40%.
a) Qu distribucin sigue la cantidad de leche consumida en un da de fin de semana?
b) Qu distribucin sigue la cantidad de leche que se sirve semanalmente?. Calcular la
probabilidad de que en una semana se utilicen ms de 450 litros de leche.
c) Teniendo en cuenta que un ao tiene 52 semanas, calcular la probabilidad de que al menos
en una semana al ao, la cantidad de leche consumida en esa cafetera sea superior a 450
litros.
SOLUCIN
a) X = Cantidad de leche consumida los das de diario N(50, 10)
Y = Cantidad de leche consumida los das de fin de semana = 1.4 X N(70, 14)
pues teniendo en cuenta las propiedades de la distribucin normal,
106 Problemas de examen resueltos
si X N(, ) Y = bX N(b, b)
b) Z = Cantidad de leche consumida semanalmente =

= =
+
2
1
5
1 i
i
i
i
Y X . Por ser suma de normales
independientes, Z tambin sigue una distribucin normal, con los siguientes parmetros:
( ) ( ) ( ) 390 70 2 50 5
2
1
5
1
= + = + =

= =
x x Y E X E Z E
i
i
i
i
( ) ( ) ( ) 892 196 2 100 5
2
1
5
1
= + = + =

= =
x x Y Var X Var Z Var
i
i
i
i
Z N(390, 892 )
( ) ( ) 0228 0 9772 0 1 2 1
892
390 450
1 450 . . Z p = = =
|
|
.
|

\
|
= >
c) T = N de semanas al ao con consumo de leche superior a 450 litro B(52, 0.0228)
( ) ( ) =
|
|
.
|

\
|
= = =
52 0
9772 0 0228 0
0
52
1 0 1 1 . . T p T p 1 0.3013976 = 0.6986024
61) Contestar las siguientes cuestiones de forma razonada (incluso si la decisin ha sido
aleatoria):
a) Para explicar el comportamiento estadstico del tiempo que se tarda en realizar una
operacin bancaria (evaluada en das) se puede optar entre un modelo gamma o una
distribucin beta extendida. Indicar cul de los dos modelos sera el ms acertado en el caso
en el que el tiempo mximo de la operacin fuera de 12 das.
b) Sea X una v.a. con distribucin (5,1) e Y una v.a. con distribucin T(1,2,4). Si la varianza
de X+Y es 450 103 , podemos afirmar que las variables son independientes?
c) Siguiendo un plan de calidad, los auditores de una empresa analizan durante cinco das la
existencia de incidencias y si es posible una solucin de manera efectiva. Suponiendo que el
nmero de incidencias diario es una v.a. de Poisson de parmetro 0,45, calcular la
probabilidad de que el nmero total de incidencias durante los cinco das sea superior a 2 e
inferior a 5.
SOLUCIN
a) En el caso de tener extremos finitos, sera ms adecuado utilizar una beta extendida.
b) Var(X) =
2
a
p
=
25
1
; Var(Y) =
18
a) m)(m (b a) (b
2

=
18
7
.
Var(X+Y) = Var(Y) + Var(Y) + 2 Cov(X,Y) =
25
1
+
18
7
+ 2 Cov(X,Y) =
450
103

Estadstica Empresarial II 107
Cov(X,Y) = -0.1 0 existe cierto grado de dependencia (lineal, al menos).
c) X
i
( ) 45 . 0 Y =

=
5
1 i
i
X ( ) 25 . 2
p(2 < Y < 5) = p(Y = 3) + p(Y = 4) =
2.25
4
2.25
3
e
4!
2.25
e
3!
2.25

+ = 0.312646625
62) Sea X una v.a. con distribucin (4,1) e Y otra v.a. con funcin de densidad igual a:

< <
=
resto el en 0
1 y 0 )
3
y (1
3
4
(y)
Y
f
Calcular:
a) La esperanza y la varianza de X e Y.
b) Sabiendo que la Cov(X,Y) = 1/75, la esperanza y la varianza de 4X 8Y.
SOLUCIN
a) E(X) =
q p
p
+
=
1 4
4
+
=
5
4
.
Var(X) =
2
q) 1)(p q (p
pq
+ + +
=
2
1) 1)(4 1 (4
4x1
+ + +
=
75
2
;
E(Y) =

=
=

1 y
0 y
3
)dy y (1
3
4
y =
1 y
0 y
5 2
y
15
4
y
3
2
=
=
(

\
|
=
15
4
3
2
=
5
2
.
E(Y
2
) =

=
=

1 y
0 y
3 2
)dy y (1
3
4
y =
1 y
0 y
6 3
y
9
2
y
9
4
=
=
(

\
|
=
9
2
9
4
=
9
2
.
Var(Y) = E(Y
2
) - E(Y) =
9
2
-
2
5
2
|
.
|

\
|
=
225
14
;
b) E(4X 8Y) = 4E(X) 8E(Y) =
5
4
4x -
5
2
8x = 0.
Var(4X 8Y) = 4
2
Var(X) + 8
2
Var(Y) 2x4x8Cov(X,Y) =
75
2
16x +
225
14
64 -
75
1
64 =
9
32
108 Problemas de examen resueltos
63) La normativa de contratos de obra en un determinado sector exige que se concedan
mediante una subasta. El presupuesto (en millones de ) que presenta cada empresa se puede
considerar, desde un punto de vista probabilstico, que es una variable aleatoria con
distribucin N(1,23; 0,1).
a) Obtener, en porcentaje, el nmero de empresas que probablemente presenten un
presupuesto inferior a 1 milln de .
b) Supongamos que antes de iniciada la obra, de forma inesperada y, por tanto,
independientemente del presupuesto inicial, se detecta que se tendr que hacer frente a unos
costes adicionales, que tambin se distribuyen normalmente. La previsin es que, por trmino
medio, este coste adicional sea de 100.000 , con una desviacin tpica de 0,24 millones de .
Obtener la distribucin del coste final de la obra y calcular la probabilidad de que dicho coste
sea inferior a 2 millones de .
SOLUCIN
a) Sea X = Presupuesto que presenta una empresa (millones de ) N(1.23,0.1).
( ) 1 X p < =
|
.
|

\
|
<

0.1
1.23 1
0.1
1.23 X
p = ( ) 2.3 Z p < = ) 3 . 2 ( = 0.0107 1% de empresas.
b) Sean Y = Costes adicionales N(0.1, 24 . 0 ) y C = Coste final de la obra = X + Y. Como
X e Y estn normalmente distribuidas su suma tambin es una normal, veamos de que parmetros.
E(C) = E(X+Y) = E(X) + E(Y) = 1.23 + 0.1 = 1.33.
Var (C) = Var(X+Y) = Var(X) + Var(Y) = 0.01 + 0.24 = 0.25 0.5
C
= .
X e Y independientes
C N(1.33,0.5) ( ) 2 C p < =
|
.
|

\
|
<
0.5
1.33 2
Z p = ) 34 . 1 ( = 0.9099.
tipificando
64) En la elaboracin de un determinado tipo de galleta, una fbrica utiliza, entre otros, dos
componentes (A y B). Si definimos las variables X = N de gramos del producto A por cada
galleta (no excede de 1 gramo) e Y = N de gramos del producto B por cada galleta (no
excede de 10 gramos), con funcin de densidad conjunta:
( ) y x
25
1
y x, f = , cuando 0 < x < 1; 0 < y < 10.
a) Obtener las distribuciones marginales de cada una de las variables. Indicar si pertenece a
alguno de los modelos conocidos.
b) Calcular el nmero medio de gramos de cada ingrediente en cada galleta.
Estadstica Empresarial II 109
c) Son independientes? Obtener el valor de E(XY).
d) Si un paquete de galletas se compone de 20 unidades, indicar con qu probabilidad el
nmero de galletas que contienen menos de 2 gramos del producto B en su composicin es 1 a
lo sumo.
SOLUCIN
a) ( ) x f
X
=

=
=
10 y
0 y
dy xy
25
1
=
10 y
0 y
2
2
y
x
25
1
=
=
(

\
|
= 2x 0 < x < 1 X (2,1).
( ) y f
Y
=

=
=
1 x
0 x
dx y x
25
1
=
1 x
0 x
2
2
x
y
25
1
=
=
(

\
|
= 10 y 0 y,
50
1
< < Y e(2,1,0,10).
b) ( )
q p
p
X E
+
= =
3
2
( ) ( ) a a b
q p
p
Y E +
+
= =
3
20
c) Claramente, las variables son independientes pues:
( ) y x
25
1
y x, f
Y X,
= = 2x y
50
1
= ( ) x f
X
( ) y f
Y
.
Como son independientes, la covarianza es cero E(XY) = E(X) E(Y) =
3
2
3
20
=
9
40
.
d) S = nmero de galletas con menos de 2 gramos en su composicin en un paquete de 20.
S seguir una distribucin B(20, | | 2 Y p < ).
| | 2 Y p < =

=
=
2 y
0 y
ydy
50
1
=
2 y
0 y
2
100
y
=
=
(

\
|
=
100
4
= 0.04 S B(20,0.04).
| | 1 S p = | | 0 S p = + | | 1 S p = =
20 0
96 . 0 04 . 0
0
20
|
|
.
|

\
|
+
19 1
96 . 0 04 . 0
1
20
|
|
.
|

\
|
= 0.810337795.
110 Problemas de examen resueltos
65) El tiempo, en horas, que tarda un empleado en completar un expediente, sigue una
distribucin gamma de media 2 horas y varianza igual a 1. Si dicho empleado realiza 50
expedientes al mes:
a) Calcular la probabilidad aproximada de que el tiempo total que emplea sea mayor de 95
horas. Indicar asimismo como se calculara esta probabilidad de forma exacta.
b) Si por cada expediente el empleado recibe 12 fijos, ms un plus de 10 por hora
trabajada, calcular la media y la desviacin tpica de la v.a. ingresos mensuales.
SOLUCIN
a) Sea la v.a. T = tiempo (en horas) en completar un expediente.
T (a,p), tal que 2
a
p
E(T) = = y 1
a
p
Var(T)
2
= = de donde a = 2 y p =4
Sea X = tiempo total en completar 50 expedientes = T
1
+ T
2
++ T
50
.
Su distribucin exacta por ser suma de variables aleatorias independientes con distribucin gamma,
con el mismo parmetro a, ser (2, 200) y por tanto:

=
=

= >
x
95 x
2x 199
200
dx e x
(200)
2
95) p(X que manualmente es casi imposible de resolver
La distribucin aproximada de T por ser suma de v.a independientes, utilizando el Teorema Central
del Lmite es N(50x2, 1x 50 ) es decir ) 50 N(100, , por tanto
0.7611 0.2389 1 0.71) ( 1 0.707) p(Z
50
100 95
50
100 X
p 95) p(X
Z
= = = > |
.
|

\
|
>

= >
tipificando Aplicando el Teorema Central del Lmite
b) Sea G= ingresos mensuales (por los 50 expedientes)
G = 12x50 + 10 (T
1
+ T
2
++ T
50
) = 600 + 10 X
E(G) = 600 + 10E(X) = 600 + 10x100 = 1 600
Var(G) = Var(10X) = 10
2
Var(X) = 100x50 = 5 000 5000
G
=
Estadstica Empresarial II 111
66) Una empresa comercializa un producto en lotes de 10 unidades que se embalan en una
caja de madera. Se sabe que el peso de cada unidad es una variable aleatoria con distribucin
normal de media 90 gramos y varianza 4. Adems el peso de la caja de madera, en gramos, es
otra variable aleatoria con distribucin N(300, 20).
a) Si cada unidad, segn su etiqueta, debe pesar al menos 92 gramos, calcular la probabilidad de que
una unidad no cumpla lo estipulado.
b) En un lote, cul es la probabilidad de que todas las unidades del mismo no cumplan lo
indicado en su etiqueta?
c) Calcular la distribucin de la v.a. peso total del lote embalado y la probabilidad de que
dicho peso sea mayor que 1.250 gramos.
SOLUCIN
a) Sea X la v.a. peso en gramos de una unidad, puesto que Var(X) = 4, X N(90,2)
0.8413 (1) 1) p(Z
2
90 92
2
90 X
p 92) p(X
Z
= = < =
|
.
|

\
|
<

= <
siendo la distribucin de Z una N(0,1).
b) Sea T = nmero de unidades que no cumplen lo estipulado en las 10 unidades de un lote.
Llamando xito = una unidad no cumple lo estipulado, la v.a. T sigue una distribucin binomial de
parmetros n = 10 y p = p(X < 92) = 0.8413. Nos piden:
0.1776 0.8413 0.1587 0.8413
10
10
10) p(T
10 0 10
= =
|
|
.
|

\
|
= =
c) Sea Y = peso de la caja de madera (en gramos) N(300, 20) y sea P = peso total de una lote
embalado. Podemos escribir esta variable como:
Y X ... X X P
10 2 1
+ + + + =
y puesto que:
200 1 300 10x90 E(Y) ) E(X ... ) E(X ) E(X E(P)
10 2 1
= + = + + + + =
440 400 10x4 Var(Y) ) Var(X ... ) Var(X ) Var(X Var(P)
10 2 1
= + = + + + + =
La distribucin de P ) 440 200, N(1
Por tanto:
0.0087 0.9913 - 1 (2.38) 1 2.38) p(Z
440
200 1 250 1
440
200 1 P
p 250) 1 p(P
Z
= = = > = |
.
|

\
|
>

= > .
112 Problemas de examen resueltos
67) Sea la variable aleatoria bidimensional (X,Y), de la que poseemos los siguientes datos de
su ley de probabilidad conjunta:
X\Y 1 2 3 p(X=x
i
)
0 0 a b 3/24
1 4/24 c d 9/24
2 e f 0 9/24
3 g 0 0 3/24
p(Y=y
j
) 8/24 12/24 4/24
a) Sabiendo que
4
1
2) Y p(X = + comprobar que p(X=0, Y=2) = 1/12.
b) Completar la tabla.
c) Calcular la esperanza y la varianza de 2X - 3Y.
d) Calcular ( )
2 Y
X
E
=
.
SOLUCIN
a) 1/4 4/24 a 0 1) Y 1, p(X 2) Y 0, p(X 1) Y 0, p(X 2) Y p(X = + + = = = + = = + = = = +
de donde a = 2/24.
b) La tabla de la ley de probabilidades conjunta es:
X\Y 1 2 3 p(X = x
i
)
0 0 2/24 b 3/24
1 4/24 c d 9/24
2 e f 0 9/24
3 g 0 0 3/24
p(Y = y
j
) 8/24 12/24 4/24
sabiendo como se calculan las distribuciones marginales a partir de la conjunta es sencillo
completar la tabla. Por ejemplo de la primera fila:
0 + 2/24 + b = 3/24 de donde b = 1/24
La tabla completa es la siguiente:
X\Y 1 2 3 p(X = x
i
)
0 0 2/24 1/24 3/24
1 4/24 2/24 3/24 9/24
2 1/24 8/24 0 9/24
3 3/24 0 0 3/24
p(Y = y
j
) 8/24 12/24 4/24
Estadstica Empresarial II 113
c) Calculamos las esperanzas y varianzas de X e Y y la covarianza entre ambas
2
3
24
3
3
24
9
2
24
9
1
24
3
0 E(X) = + + + =
3
24
3
3
24
9
2
24
9
1
24
3
0 ) E(X
2 2 2 2 2
= + + + =
4
3
2
3
3 E(X) ) E(X Var(X)
2
2 2
=
|
.
|

\
|
= =
6
11
24
4
3
24
12
2
24
8
1 E(Y) = + + =
6
23
24
4
3
24
12
2
24
8
1 ) E(Y
2 2 2 2
= + + =
36
17
6
11
6
23
E(Y) ) E(Y Var(Y)
2
2 2
=
|
.
|

\
|
= =
2
5
24
3
3
24
8
4
24
1
2
24
3
3
24
2
2
24
4
1 E(XY) = + + + + + =
4
1
6
11
2
3
2
5
E(X)E(Y) E(XY) Y) Cov(X, = = =
2
5
6
11
3
2
3
2 3E(Y) 2E(X) 3Y) E(2X = = =
4
41
4
1
12
36
17
9
4
3
4 Y) X, 2x2x3xCov( Var(Y) 3 Var(X) 2 3Y) Var(2X
2 2
= + + = + =
d) Calculamos la ley de probabilidad de la variable condicionada X/Y=2.
2) p(Y
2) Y , x p(X
2 Y
x X
p
i
i
=
= =
=
|
.
|

\
|
=
=
114 Problemas de examen resueltos
de donde se obtiene:
2 Y
X
=
0 1 2
|
.
|

\
|
=
=
2 Y
x X
p
i
1/6 1/6 2/3
(La probabilidad para X = 3 es cero)
( )
2
3
3
2
2
6
1
1
6
1
0
2 Y
x X
p x
2 Y
X
E
i
x
i
i
= + + =
|
.
|

\
|
=
=
=
=

68) Sea (X, Y) una variable aleatoria bidimensional con funcin de densidad conjunta:
f(x,y) = 10x
2
y 0 < y < x < 1
Calcular:
a) Las funciones de densidad marginal de X e Y, son independientes?
b) La covarianza entre X e Y.
c) La funcin de densidad de Y/X=1/2. Qu distribucin sigue?
d) La esperanza de Y/X=1/2.
SOLUCIN
Sea la funcin de densidad conjunta f(x,y) = 10x
2
y 0 < y < x < 1
a) Funcin de densidad marginal de X
f
X
(x) = ( |
4
x y
0 y
2 2
x y
0 y
2
x 5 y 5x dy y 10x = =
=
=
=
=

0 < x < 1
luego

< <
=
resto el en 0
1 x 0 x 5
(x) f
4
X
Funcin de densidad marginal de Y
f
Y
(y) = ) y - y(1
3
10
yx
3
10
dx y 10x
3
1 x
y x
3
1 x
y x
2
=
(

\
|
=
=
=
=
=

0 < y < 1
X
Y
1
Y = X
1
Estadstica Empresarial II 115
luego

< <
=
resto el en 0
1 y 0 ) y - y(1
3
10
(y) f
3
Y
Dos variables son independientes f
XY
(x,y) = f
X
(x) f
Y
(y) (x,y). Veamos si se verifica la
igualdad en este caso.
Como f
X
(x) f
Y
(y) = ) y - y(1
3
10
5x
3 4
y 10x
2
= f
XY
(x,y)
X e Y son estadsticamente dependientes.
b) X (5,1) pues

< <

=
< <

=

resto 0
1 x 0 x 5
resto 0
1 x 0 x) (1 x
q) (p,
1
(x) f
4
1 q 1 p
X
ya que
5
0! 4!
5!
(1) (5)
) 1 (5
(5,1)
1
= =

+
=

luego
E(X) =
q p
p
+
=
1 5
5
+
=
6
5
.
Var(X) =
2
q) 1)(p q (p
pq
+ + +
=
2
1) 1)(5 1 (5
5x1
+ + +
=
252
5
;
E(Y) =

=
=

1 y
0 y
3
)dy y y(1
3
10
y =
1 y
0 y
6 3
6
y
3
y
3
10
=
=
(

\
|
=
9
5
.
E(Y
2
) =

=
=

1 y
0 y
3 2
)dy y y(1
3
10
y =
1 y
0 y
7 4
7
y
4
y
3
10
=
=
(

\
|
=
14
5
.
116 Problemas de examen resueltos
Var(Y) = E(Y
2
) - E(Y) =
14
5
-
2
9
5
|
.
|

\
|
=
134 1
55
;
E(XY) = =
|
|
.
|

\
|
=
|
|
.
|

\
|

=
=
=
=
=
=
=
=
1 x
0 x
y
0 y
2 3
x
- x
y
y
Y X,
dx dy y x 10 dx dy y) (x, f y x
x
21
10
x
21
10
dx x
3
10
dx
3
y
10x
1 x
0 x
7
1 x
0 x
6
1 x
0 x
y
0 y
3
3
=
(

\
|
= =
(

\
|
=
=
=
=
=
=
=
=
=

x
Cov (X,Y) = E(XY) E(X)E(Y) =
21
10

|
.
|

\
|
9
5
x
6
5
=
378
5
c) Funcin de densidad condicionada de Y por X.
( )
2 4
2
X
Y X,
x Y/X
x
y 2
x 5
y x 10
(x) f
y) (x, f
x y/X f = = = =
=
0 < y < x
siempre que 0 < x < 1.
En particular, para X = 1/2, tenemos:
( )
( )
y 8
2
1
y 2
1/2 y/X f
2 1/2 Y/X
= = =
=
0 < y < 1/2
Veamos que Y/X=1/2 sigue una distribucin e(2,1,0,1/2)

< <

=
< <
= =

+
=
resto el en 0
1/2 y 0 y 8
resto el en 0
b y a y) (b a) - (y
a) - (b
1
q) (p,
1
1/2) (y/X f
1 q 1 p
1 - q p
1/2 Y/X
ya que
8 4
0! 1!
2!
4
(1) (2)
) 1 (2
) 0 2 / 1 (
1
(2,1)
1
1 1 2
= =

+
=

+
d) E(Y/X = 1/2) =

=
=
1/2 y
0 y
dy y 8 y = ( |
1/2 y
0 y
3
y
3
8 =
=
=
3
1
.
Estadstica Empresarial II 117
69) El nmero de servicios que realiza un chfer de una empresa al da (X) tiene la siguiente
ley de probabilidad
X 7 8 9 10
P(X = x) 0,2 0,3 0,4 0,1
a) Calcular la probabilidad de que en un trimestre (81 das) el nmero total de servicios
realizados (T) est comprendido entre 650 y 700.
b) Sabiendo que por cada servicio cobra 9,25 , que el nmero de kilmetros recorridos por
servicio es de 11,5, que cada kilmetro recorrido tiene un coste en combustible de 0,07 y que
al mes tiene unos gastos fijos de 700 , calcular la probabilidad de que la ganancia del chfer
en un trimestre sea mayor que 3.750 .
SOLUCIN
a) Sean X = nmero de servicios que realiza el chofer de una empresa al da, y
T = nmero de servicios que realiza el chofer de una empresa al trimestre =

=
81
1 i
i
X
Hay que calcular p(650 < T < 700), pero como no conocemos la distribucin de T y es una suma de
variables aleatorias independientes idnticamente distribuidas con esperanza y varianza finitas, la
probabilidad pedida la podemos aproximar aplicando el Teorema Central del Lmite.
E(X) = 7x0.2 + 8x0.3 + 9x0.4 + 10x0.1 = 8.4
E(X
2
) = 7
2
x0.2 + 8
2
x0.3 + 9
2
x0.4 + 10
2
x0.1 = 71.4
Var(X) = E(X
2
) E(X) = 71.4 8.4
2
= 0.84
E(T) = ( )

= = =
= = |
.
|

\
|
81
1 i
81
1 i
i
81
1 i
i
8.4 X E X E = 81 x 8.4 = 680.4
Var(T) = ( )

= = =
= = |
.
|

\
|
81
1 i
81
1 i
i
81
1 i
i
0.84 X Var X Var = 81 x 0.84 = 68.04
por ser X
i
independientes

=
81
1 i
i
X
= 8.248636251

|
|
|
|
.
|

\
|

<

<

= |
.
|

\
|
< <

=
=
248636251 . 8
4 . 0 68 700
248636251 . 8
4 . 0 68 X
248636251 . 8
4 . 0 68 650
p 700 X 650 p
81
1 i
i 81
1 i
i
tipificando Aplicando el Teorema Central del Lmite
118 Problemas de examen resueltos
) 69 . 3 ( (2.38) = 0.9913 0.0001 = 0.9912
b) G = Ganancia = 100 2 X 0.07 x 11.5 X 25 . 9
81
1 i
i
81
1 i
i


= =
= 8.445T 2 100
E(G) = E(8.445T 2 100) = 8.445E(T) 2 100 = 8.445 x 680.4 2 100 = 3 645.978
Var(G) = Var(8.445T 2 100) = 8.445
2
Var(T) = 8.445
2
x 68.04 = 4 852.478421

T
= 69.65973314
( )
|
.
|

\
|
>

= >
4 69.6597331
645.978 3 750 3
4 69.6597331
645.978 3 G
p 750 3 G p
tipificando Aplicando el Teorema Central del Lmite
) 49 . 1 ( 1 = 1 0.9319 = 0.0681.
70) Las variables X
1
= N de delitos cometidos anualmente en la ciudad A y X
2
= N de
delitos cometidos anualmente en la ciudad B se consideran independientes. Se sabe que las
distribuciones de estas variables son normales con media 1.520 y desviacin tpica 50, en el
primer caso, y con media 480 y desviacin tpica 20, en el segundo caso. El nmero de
efectivos policiales que anualmente se destina en cada ciudad es una v.a. que depender del
nmero de delitos cometidos. La relacin en ambos casos es la siguiente:
24
20
X
Y
1
1
+ = 12
10
X
Y
2
2
+ =
Con qu probabilidad la diferencia del nmero de efectivos entre las ciudades A y B ser
inferior a 36?
SOLUCIN
( ) 10,25 40, N 12
10
X
20
X
Y Y
2 1
2 1
+ =
p(Y
1
Y
2
< 36) = ) 25 , 1
25 , 10
40 36
p(Z =

< = 0,1056.
Estadstica Empresarial II 119
71) Sea X =N de das que una mercanca se encuentra depositada en un almacn e Y =
Coste diario de almacenamiento (cientos de euros), que depende del peso de la mercanca.
La distribucin conjunta de estas variables es la siguiente:
X \ Y 1 2
1 0,2 0,1
2 0,1 0,2
3 0,3 0,0
4 0,0 0,1
a) Calcular el coste medio diario sabiendo que el nmero de das que estar almacenada la
mercanca ser menor que 3. Compara este resultado con el coste medio diario de
almacenamiento e indica si este resultado induce a pensar en la dependencia entre las
variables.
b) Si la variable producto, XY, representa el coste total (nmero de das que estuvo
depositada la mercanca x coste diario), calcular la probabilidad de que el coste total sea
inferior a 500 .
SOLUCIN
a) E(Y/X 2) = 1,5 150 . E(Y) = 1,4 140 .
Si las variables fuesen independientes, las medias deberan ser iguales.
b) p(XY < 5) = 1 p(XY 5) = 1 p(X=3,Y=2) p(X=4,Y=2) = 1 0 0,1 = 0,9.
72) Sea T = aX + bY + 2 donde a, b R y X e Y son dos variables aleatorias tales que:
E(X) = 3 E(Y) = 6 E(X
2
) = 34 E(Y
2
) = 136 E(XY) = 58.
Indicar qu relacin deben cumplir a y b para que los coeficientes de correlacin
Y T, X T,
=

sean iguales
SOLUCIN
Var(X) = (1)
2
Var(X) = 25 Var(Y) = 100
( ) ( ) ( ) ( ) 40b 25a Y X, bCov X aVar X bY, aX Cov X T, Cov = = + =
( ) ( ) ( ) ( ) 0b 10 a 40 Y bVar Y X, aCov Y bY, aX Cov Y T, Cov + = + = + =
( ) ( )
Y T,
T Y T X
X T,


Y T, Cov

X - T, Cov
= = =

Y T, X T,

10
100b 40a
5
40b 25a -
=
+
=

a = 2b
120 Problemas de examen resueltos
73) Sea (X,Y) v.a. bidimensional con funcin de densidad conjunta:

< < < < +


=
resto el en 0
2 y 0 4; x 0 xy) (1
24
1
y) (x, f
Y X,
Calcular ( )
2 X
Y
E
=
.
SOLUCIN
( )

=
=
=
=
=
y
y
dy (y) f y
2 X
Y
E
2 X
Y
Puesto que
(x) f
y) (x, f
(y) f
X
Y X,
x X
Y
=
=
calculamos la marginal de X.
( ) 4 x 0
12
x 1
2x 2
24
1
2
y
x y
24
1
dy xy) (1
24
1
dy y) (x, f (x) f
2
0
2
2
0
Y X, X
< <
+
= + =
(

\
|
+ = + = =
=
=
=
=
=
=

y
y
y
y
y
y
2 y 0 4; x 0
x) 2(1
xy 1
12
x 1
xy) (1
24
1
(x) f
y) (x, f
(y) f
X
Y X,
x X
Y
< < < <
+
+
=
+
+
= =
=
Sustituyendo x por 2: 2 y 0
6
2y 1
(y) f
2 X
Y
< <
+
=
=
( )

=
=
=
=
=
=
=
= =
(

\
|
+ =
+
= =
=
y
y
9
11
18
22
3
y
2
2
y
6
1
dy
6
2y 1
y dy (y) f y
2 X
Y
E
2 y
0 y
3 2
2 y
0 y
2 X
Y
74) La duracin en horas de un determinado servicio tiene distribucin gamma de
parmetros a = 3 y p = 1. Una persona alquila tres veces seguidas dicho servicio. Sabiendo que
la duracin de un servicio es independiente de los anteriores y que el coste es de 11 la hora,
calcular la distribucin de la variable C, coste total y la esperanza de la v.a. C
2
+ 3C 5.
SOLUCIN
a) Sea X = Duracin de un servicio en horas (3,1)
X
1
, X
2
, X
3
, i.i.d (3,1) Duracin total =
3 2 1
X X X + + (3, 1 +1 + 1) (3,3)
Estadstica Empresarial II 121
Coste total = ( )
3 2 1
X X X 11 C + + = |
.
|

\
|
,3
11
3

b) 5 3E(C) ) E(C 5) 3C E(C


2 2
+ = +
11
11
3
3
E(C) = = ;
3
121
11
3
3
(C) E ) E(C Var(C)
2
2 2
=
|
.
|

\
|
= =
de donde
3
484
11
3
121
(C) E Var(C) ) E(C
2 2 2
= + = + =
3
568
5 3x11
3
484
5 3E(C) ) E(C 5) 3C E(C
2 2
= + = + = +
75) El nmero medio de entradas diarias que se venden en una sala de cine es 100 con una
desviacin tpica de 50. El dueo quiere cerrar la sala para reformarla y los gastos de dicha
reforma ascienden a 17.000 .
a) Si el precio de la entrada es de 4 y los costes fijos diarios son 100 , qu probabilidades
tiene el propietario de recuperar el dinero invertido en un plazo de dos meses sin subir el
precio de la entrada?
b) Cuntos das necesitara para recuperarse de la inversin con una probabilidad del 95%?
SOLUCIN
a) Sea X = N de entradas vendidas diariamente E(X) = 100 Var(X) = 2500
y B = Beneficio diario = 4X 100 E(B) = 300 Var(B) = 40000
B
1
, , B
60
, sucesin de v.a.i.id. con esperanza y varianza finitas, por el T.C.L.
( ) ( ) 60 200 18000, N 60 , 60 N B
L
60
1 i
i

=
= |
.
|

\
|
= |
.
|

\
|
= |
.
|

\
|
>

= =
60 200
18000 17000
1 17000 B p 1 17000 B p
TCL el por
60
1 i
i
60
1 i
i
( ) 0,7422 0,2578 1 0,65 1 = = =
122 Problemas de examen resueltos
b) ( ) ( ) n 200 300n, N n , n N B
L
n
1 i
i

=
0,95
n 200
300n 17000
1 17000 B p 1 17000 B p
TCL el por
n
1 i
i
n
1 i
i
= |
.
|

\
|
= |
.
|

\
|
= |
.
|

\
|
>

= =
1,645
n 200
300n 17000
0,05
n 200
300n 17000
=

= |
.
|

\
|

0 17000 n 329 300n =


Absurdo 6,99 n
65,55 n 8,096 n
=
= =
La solucin vlida es n 66
76) Un propietario afirma que la ganancia en un determinado negocio, expresada en miles de
euros, viene representada por una v.a. X que sigue una distribucin N(50,2). Tiene dos
opciones: si elige la primera, correr con unos gastos adicionales, cantidad que se puede
interpretar como una v.a. Y
1
con distribucin N(14,2). En la segunda opcin, los gastos se
presentan como una v.a. Y
2
que sigue una distribucin N(10,8). Gasto y ganancia se supone
que son independientes.
a) Indicar qu opcin es mejor desde el punto de vista de obtener un mayor beneficio por
trmino medio.
b) Si su meta se fija en obtener unos beneficios de 30.000 euros como mnimo, cul ser la
mejor opcin desde el punto de vista probabilstico? Comentar el resultado, comparndolo
con el del apartado anterior.
SOLUCIN
X N(50,2) Y
1
N(14,2) Y
2
N(10,8)
B
1
= X - Y
1
N(36, 8 ) B
2
= X Y
2
N(40, 68 )
a) Se obtiene mayor beneficio por trmino medio con la segunda opcin: E(B
1
) = 36; E(B
2
) = 40.
b) p(B
1
30) = |
.
|

\
|
=

2,12
8
36 30
Z p = 1 0,0170 = 0,9830.
Estadstica Empresarial II 123
P(B
2
30) = |
.
|

\
|
=

1,21
68
40 30
Z p = 1 0,1131 = 0,8869.
Con la segunda opcin, los beneficios medios son superiores a los de la primera; sin embargo, la
probabilidad pedida es mayor en el primer caso.
77) De la distribucin conjunta entre dos variables, conocemos la siguiente informacin:
X\Y 1 2 3 p(X=x)
1 - - - 0,20
2 0,06 0,04 0,20 0,30
3 0,20 0,10 0,20 0,50
p(Y=y) - - - 1,00
Adems, se sabe que las variables X y Y/X=1 (condicionada) estn igualmente distribuidas.
Completar la tabla y hallar el coeficiente de correlacin. Comentar el resultado.
SOLUCIN
Si X y X/Y=1 estn igualmente distribuidas, toman los mismos valores con las mismas
probabilidades.
X p(X=x) Y/X = 1 P(Y = y/X = 1)
1 0,20 1 0,20
2 0,30 2 0,30
3 0,50 3 0,50
p(X = 1, Y = y) = p(Y = y/X = 1) p(X = 1) = p(Y = y/X = 1) x 0,20
Por ejemplo:
p(X = 1,Y = 1) = p(Y = 1/X = 1) p(X = 1) = p(Y = 1/X = 1) x 0,20 = 0,20 x 0,20 = 0,04.
X\Y 1 2 3 P(X=x)
1 0,04 0,06 0,10 0,20
2 0,06 0,04 0,20 0,30
3 0,20 0,10 0,20 0,50
P(Y=y) 0,30 0,20 0,50 1,00
E(X) = 2,3 E(X
2
) = 5,9 Var(X) = 0,61 E(Y) = 2,2 E(Y
2
) = 5,6 Var(Y) = 0,76
E(XY) = 4,94 Cov(X,Y) = 0,12
0,76 x 0,61
0,12

XY

= = 0,1762.
78) Sean X e Y dos variables aleatorias independientes con media 0 y desviacin tpica 1, y
sean S y T:
124 Problemas de examen resueltos
3
b Y 2X
S
+ +
=
a
2Y X
T

= con a, b R
Calcular el valor de a y b para que E(S) = 1 y Var(T) = 5.
Calcular Cov(S,T) para cualquier par de valores a y b. Son S y T variables independientes?
SOLUCIN
( ) ( ) ( ) 1
3
b
3
b
Y E
3
1
X E
3
2
S E = = + + = b = 3
( ) ( ) ( ) ( ) 5
2
a
5
4 1
2
a
1
4Var(Y) X Var
2
a
1
T Var = = + = + =
luego a = 1 a = 1.
=
|
.
|

\
| + +
=
a
2Y X
,
3
b Y 2X
Cov T) Cov(S,
( ) ( ) ( ) ( ) ( ) 0 Y 2Var Y X, Cov Y X, 4Cov X 2Var
3a
1
= + =
por lo tanto, S y T estn incorreladas, pero no son necesariamente independientes.
79) Sea (X,Y) v.a. bidimensional con funcin de densidad conjunta:

> < <


=

resto el en 0
0 y 1; x 0 e 6x
y) (x, f
2y 2
Y X,
Qu distribucin siguen X e Y? Son independientes? Hallar las esperanzas de X e Y.
Calcular ( ) 1 Y 0,5; X p
SOLUCIN
a) =

= =

=
=
=
=
y
y 0
y
- y
Y X, X
dy
2y
e
2
6x dy y) (x, f (x) f
1 x 0
2
3x
2
2y
e
2
6x dy
2y
e
2
6x
y
0 y
y
0 y
< < =
(
(

\
|
=

=
=
=
=
=

0 y
2y
e 2
3
2x
2y
e dx
2
6x
2y
e dx
2y
e
2
6x dx y) (x, f (y) f
1 x
0 x
1 x
0 x
1 x
0 x
x
- x
Y X, Y
>

=
(

\
|

= =
=
=
=
=
=
=
=
=

Estadstica Empresarial II 125
luego X e Y son independientes pues y x, (y) f (x) f y) (x, f
Y X Y X,
=
adems X (3, 1) e Y (2).
Por tanto, E(X) = 3/4 y E(Y) = 1/2.
b)
( ) =
(

\
|
=
(

= =

=
=
=
=

=
=
=
=

=
=
=
=
0,5 x
0 x
y
1 y
2y
2
0,5 x
0 x
y
1 y
2y 2
0,5 x
x
y
1 y
Y X,
dx
2
e
6x dx dy e 6x dy dx y) (x, f 1 Y 0,5; X p
( | 0,0169
8
e
x e dx 3x e
2
0,5
0
3 2
0,5
0
2 2
= = = =

=
=

=
=

x
x
x
x
(= p(X 0,5) p(Y 1)).
80) El tiempo de fabricacin de una determinada pieza (en horas) es una variable aleatoria
con distribucin triangular de parmetros 2, 4 y 7.
Calcular la probabilidad de tardar ms de 3 horas en fabricar una pieza.
Si a lo largo de una semana se fabrican 10 piezas. Cul es la probabilidad de que al menos en
dos de las piezas se hayan invertido como mucho tres horas en su fabricacin?
SOLUCIN
Sea X = Tiempo de fabricacin de una pieza (en horas).
a) Sustituyendo el valor de los parmetros, la funcin de densidad de X ser:



=
resto el en 0
7 x 4 x) (7
15
2
4 x 2 2) (x
5
1
(x) f
X
0,9
2
x
7x
15
2
2x
2
x
5
1
dx x) (7
15
2
dx 2) (x
5
1
dx (x) f 3) p(X
7
4
2
4
3
2
7
4
4
3 3
X
=
(

\
|
+
(

\
|
= + = = >

b) Sea la v.a. Y = Nmero de piezas en las que se han invertido menos de tres horas en su
fabricacin de las 10 producidas.
Y B(10, p) siendo la probabilidad de xito: 0,1 0,9 1 3) p(X 1 3) p(X p = = > = = .
| | = = + = = < = 1) p(Y 0) p(Y 1 2) p(Y 1 2) p(Y 0,2639. 0,9 0,1
1
10
0,9 0,1
0
10
1
9 1 10 0
=
(

+
|
|
.
|

\
|
+
|
|
.
|

\
|

126 Problemas de examen resueltos


81) Unos pintores estiman que en pintar una casa de hasta 120 m.2, tardan por trmino
medio 3 das con una desviacin tpica de 0,6 y en pintar una casa de mayor tamao, tardan
por trmino medio 5 das con una desviacin tpica de 1,4. Se supone que el equipo de pintores
no realiza de manera simultnea trabajos en casas de tamaos diferentes. Este ao se han
comprometido con 40 casas pequeas y 30 casas grandes.
a) Calcular la probabilidad de que pasen ms das pintando casas pequeas que grandes.
b) Calcular la probabilidad de que les queden al menos 120 das libres al ao entre fines de
semana y vacaciones.
SOLUCIN
X = Tiempo que tardan en pintar una casa pequea E(X) = 3 Var(X) = 0,36
Y = Tiempo que tardan en pintar una casa grande E(Y) = 5 Var (Y) = 1,96
X
1
, , X
40
es una sucesin de v.a.i.i.d. con esperanza y varianza finitas (T.C.L.)
( ) 14,4 120, N X
L
40
1 i
i

=
Y
1
, , Y
30
es una sucesin de v.a.i.i.d. con esperanza y varianza finitas (T.C.L.)
( ) 8 , 58 150, N Y
L
30
1 i
i

=
a) ( ) 73,2 , 30 N Y X
L
30
1 i
i
40
1 i
i


= =
( ) 0,0002 0,9998 1 3,51 1
73,2
30
1 0 Y X p
30
1 i
i
40
1 i
i
= = =
|
|
.
|

\
|
= |
.
|

\
|
>

= =
b) ( ) 73,2 270, N Y X
L
30
1 i
i
40
1 i
i
+

= =
( ) 0,0018. 2,92
73,2
270 245
5 24 Y X p
30
1 i
i
40
1 i
i
= =
|
|
.
|

\
|

= |
.
|

\
|
+

= =
82) Sea (X,Y) una variable aleatoria bidimensional con funcin de densidad conjunta:
f
X,Y
(x,y)

< < < <


=
resto 0
1 y 0 2 x 0 x) (2 y x
2
3
2
a) Calcular las funciones de densidad marginal de las variables aleatorias X e Y.
Estadstica Empresarial II 127
b) Qu distribucin sigue cada una de las variables?
c) Son X e Y v. a. independientes?
d) Calcular P(X > 1, Y <
2
1
).
e) Calcular E(X) y
|
.
|

\
|
= y Y
X
E . Comenta los resultados.
f) Calcular Cov(3X - 8, 2Y + 5).
SOLUCIN
a) f
X
(x) = x) (2 x
4
3
2
y
x) (2 x
2
3
dy y x) (2 x
2
3
dy x) (2 y x
2
3
2
1 y
0 y
2
2
1 y
0 y
2
1 y
0 y
2
=
(

\
|
= =
=
=
=
=
=
=

Luego f
X
(x) =

< <
resto 0
2 x 0 x) (2 x
4
3
2
f
Y
(y) = 2y
8
3x
x y dx x) (2 x
2
3
y dx x) (2 y x
2
3
2 x
0 x
4
3
2 x
0 x
2
2 x
0 x
2
=
(

\
|
= =
=
=
=
=
=
=

< <
resto 0
1 y 0 2y
b) X sigue una distribucin
e
(3, 2, 0, 2) e Y sigue una distribucin (2, 1).
c) S son independientes X e Y porque f
X,Y
(x,y) = f
X
(x) f
Y
(y), para todo x, y
d) p(X >1, Y<
2
1
) = p(X >1) p(Y<
2
1
), por ser independientes X e Y
p(X >1) =
16
11
dx x) (2 x
4
3
2 x
1 x
2
=

=
=
; p( Y<
2
1
) =
4
1
dy y 2
1/2
0
=

=
=
y
y
Entonces, p(X >1, Y<
2
1
)=
4
1
x
16
11
=
64
11
e) Como X e Y son independientes,
|
.
|

\
|
= y Y
X
E = E(X)
Como X sigue una
e
(3, 2, 0, 2) E(X) = a + (b a)
q p
p
+
=
5
6
.
f) Cov(3X 8, 2Y + 5) = 3x2xCov(X,Y) = 0 puesto que las variables son incorreladas.
83) En unos grandes almacenes, el nmero medio de devoluciones por hora en un
determinado departamento es de 4. Calcular:
a) La probabilidad de que no haya ninguna devolucin en una hora.
b) La probabilidad de que en una hora haya al menos una devolucin.
128 Problemas de examen resueltos
c) La probabilidad de que en 4 horas haya 10 devoluciones.
d) De las 12 horas que permanece abierto ese gran almacn, cul es la probabilidad de que
haya como mximo 2 horas en las que no se produce ninguna devolucin?
e)Se sabe que en lo que va de hora ha habido ya al menos una devolucin. Cul es la
probabilidad de que no se produzcan ms de dos devoluciones durante esa hora?
f) Si T = X
2
5X + 3 es el tiempo en minutos que tarda el dependiente en realizar la
devolucin, calcular el tiempo medio que tarda el dependiente en realizar una devolucin.
SOLUCIN
a) X = n de devoluciones por hora. X sigue una P(4). E(X) = 4. p(X = 0) =
4
0
e
0!
4

= 0,0183.
b) p(X 1) = 1 p(X < 1) = 1 p(X = 0) = 1 0,0183 = 0.9817.
c) X
i
= n devoluciones en la hora i-sima. X
i
son independientes.
Y = X
1
+ X
2
+ X
3
+ X
4
= n de devoluciones en cuatro horas, que sigue una P(4x4) P(16).
p(Y = 10) =
16
10
e
10!
16

= 0,03401
d) Z = n de horas en las que no hay devoluciones de las 12h. que permanece abierto.
Z sigue una B(n, p) B(12, 0,0183), puesto que p = p(xito) = p(X = 0) = 0,0183.
p(Z 2) = p(Z = 0) + p(Z = 1)+ p(Z = 2) =
=
10 2 11 1 12 0
9817 , 0 0183 , 0
2
12
9817 , 0 0183 , 0
1
12
9817 , 0 0183 , 0
0
12
|
|
.
|

\
|
+
|
|
.
|

\
|
+
|
|
.
|

\
|
= 0,9988.
e) ( ) 1 X 2 X p = =
= + =
=


0,9817
2) p(X 1) p(X
1) p(X
2) X p(1

0,9817
e
2!
4
e
1!
4
4
2
4
+
2239 , 0
9817 , 0
2198 , 0
= =
f) E(X) = Var(X) = 4 Var(X) = E(X
2
) E(X)
2
= E(X
2
) 4
2
. Entonces E(X
2
) = 20.
E(T)= E(X
2
5X + 3) = E(X
2
) 5E(X) + 3 = 20 5x4 + 3 = 3 minutos es el tiempo medio que tarda
en hacer la devolucin.
84) Sean X e Y variables aleatorias de las que sabemos los siguientes datos:

< <
=
resto el en 0
1 x 0 x) (1 x 6
(x) f
X
E(Y) = 2; Var(4 Y + 5) = 48; Cov(3X, 2Y) = 6.
Calcular:
Estadstica Empresarial II 129
a) Var(X + Y) y Var (X Y).
b) Cov(2X + 3, X + Y).
c) E(XY 3Y
2
+ 5).
SOLUCIN
X sigue una distribucin beta de parmetros 2, 2. Por tanto:
20
1
2) 1)(2 2 (2
2x2
Var(X) ;
2
1
2 2
2
E(X)
2
=
+ + +
= =
+
=
3 Var(Y) 48 Var(Y) 4 5) Var(4Y
2
= = = + 1 Y) Cov(X, 6 Y) 2x3xCov(X, 3Y) Cov(2X, = = =
a)
20
101
Y) Cov(X, 2 Var(Y) Var(X) Y) Var(X = + + = + ;
20
21
Y) Cov(X, 2 Var(Y) Var(X) Y) Var(X = + =
b) = + = + + + = + + Y) Cov(X, 2 X) Cov(X, 2 Y) 3, X Cov(2 X) 3, X Cov(2 Y) X 3, X Cov(2
10
21
Y) Cov(X, 2 Var(X) 2 = + =
c) E(Y) E(X) - Y) E(X Y) Cov(X, = 2 E(Y) E(X) Y) Cov(X, Y) E(X = + =
(Y) E - ) E(Y Var(Y)
2 2
= 7 (Y) E Var(Y) ) E(Y
2 2
= + =
Luego, 14 5 ) E(Y 3 E(XY) 5) 3Y E(XY
2 2
= + = +
85) Las facturas cobradas por los servicios tcnicos de reparacin de una empresa, constan
de dos conceptos: piezas y mano de obra. La media del primer concepto es 3,60 y la del
segundo 12 . Las desviaciones tpicas respectivas son 0,90 y 1,20 . La relacin entre las
cantidades facturadas por ambos conceptos viene expresada por un coeficiente de correlacin
igual a 0,56.
a) Calcula la esperanza y varianza de la facturacin total de un servicio.
b) Cul es la probabilidad de que con 100 servicios se alcance al menos una facturacin total
de 1.530 ?
c) En el supuesto de que solo se realizasen nueve servicios y las cantidades facturadas en
ambos conceptos fueran independientes y normalmente distribuidas, calcular la probabilidad
de que las cantidades ingresadas en concepto de mano de obra superen a las obtenidas por
recambio de piezas en 79,2 como mximo. Qu ocurrira si las distribuciones no fueran
normales?
SOLUCIN
130 Problemas de examen resueltos
a) Sean: X
i
= facturacin por piezas en el servicio i-simo
Y
i
= facturacin por mano de obra en el servicio i-simo
T
i
= facturacin total en el servicio i-simo = X
i
+ Y
i
E(X
i
) = 3,6 Var (X
i
) = 0,81 E(Y
i
) = 12 Var (Y
i
) = 1,44
XY
= 0,56
Y) Var(X)Var(
Y) Cov(X,

XY
= Cov(X,Y) = Y) Var(X)Var(
XY
= 0,56 x 0,9 x 1,2 = 0,6048
Luego: E(T
i
) = E(X
i
) + E(Y
i
) = 15,6
Var(T
i
) = Var (X
i
) + Var (Y
i
) + 2 Cov(X
i
, Y
i
) = 3,4596
i
T
= 1,86
b) Sea Z = facturacin de 100 servicios =

=
100
1 i
i
T
Las T
i
son variables aleatorias i.i.d. con esperanza y varianza finitas, luego teniendo en cuenta que:
1.560 15,6 x 100 T E
100
1 i
i
= = |
.
|

\
|

=
96 , 45 3 4596 , 3 x 100 T Var
100
1 i
i
= = |
.
|

\
|

=
por el Teorema Central del Lmite:
1) N(0,
18,6
1.560 T
L
100
1 i
i

=
( ) ( ) 0,9463 0,0537 1 1,61 1
18,6
1.560 1.530
1 1.530 Z p
T.C.L.
= = = |
.
|

\
|

c) Si las v.a. son normales e independientes, por las propiedades de la normal:
E(D
i
) = E(Y
i
) E(X
i
) = 8,4 Var(D
i
) = Var(X
i
) + Var (Y
i
) = 2,25
D
i
= Y
i
X
i
N(8,4, 1,5)

=
9
1 i
i
D N(75,6, 4,5) ( ) 0,7881 0,8 79,2 D p
9
1 i
i
= = |
.
|

\
|

=
Si las variables no son normales no se puede calcular la probabilidad.
86) Un agente inmobiliario est interesado en averiguar cul es la relacin entre el nmero de
lneas de un anuncio en prensa sobre un apartamento y el volumen de demanda de
informacin por parte de posibles inquilinos. Llamemos X a la variable aleatoria nmero de
lneas e Y a la variable aleatoria volumen de demanda, que toma el valor 0, si despierta poco
Estadstica Empresarial II 131
inters, 1 para un inters moderado y 2, si despierta un fuerte inters. El agente estima que la
ley de probabilidad conjunta de ambas variables es:
Volumen de demanda (Y)
Nmero de lneas (X) 0 1 2
3 0,03 0,06 0,08
4 0,04 0,20 0,16
5 0,10 0,18 0,15
a) Calcular las leyes de probabilidad marginales de X e Y. Son independientes?
b) Calcular la ley de probabilidad de Y, sabiendo que el nmero de lneas es 4 y calcular su
esperanza.
c) Calcular ( )
4 X
1 Y
p
>

e interpretar el resultado.
d) Calcular la covarianza de X e Y e interpretar el resultado.
SOLUCIN
a)
X\Y 0 1 2 p(X=x)
3 0,03 0,06 0,08 0,17
4 0,04 0,20 0,16 0,40
5 0,10 0,18 0,15 0,43
p(Y=y) 0,17 0,44 0,39 1,00
No son independientes; por ejemplo: 0,17 x 0,17 0,03.
b)
Y/X=4 |
.
|

\
|
=
=
4 X
y Y
p
j
0 0,04/0,40 = 0,10
1 0,20/0,40 = 0,50
2 0,16/0,40 = 0,40
E(Y/X=4) = 0x0,10 + 1x0,5 + 2x0,4 = 1,3
c) ( ) 0,7674
43
33
0,43
0,15 0,18
5) p(X
1) Y 5; p(X
4 X
1 Y
p = =
+
=
=
=
=
>

.
Cuando el nmero de lneas del anuncio es muy alto (5), existe una alta probabilidad (76,74%) de
que la demanda sea moderada o alta.
d) E(XY) = 5,14 E(X) = 4,26 E(Y) = 1,22 0,0572 E(Y) E(X) Y) E(X Y) Cov(X, = = .
Esto implica que hay una relacin inversa entre el nmero de lneas y el volumen de demanda.
87) Sea X
1
la recaudacin, en euros, por un artculo en un kiosko de lunes a viernes. Sabemos
que se distribuye normalmente, con media 150 y desviacin tpica 3. Sea X
2
la recaudacin
por el mismo concepto durante el fin de semana. Su distribucin es N(175, 6).
132 Problemas de examen resueltos
a) Indicar en qu caso est ms asegurada la recaudacin, en los cinco primeros das de la
semana o durante el fin de semana, teniendo en cuenta la dispersin en cada caso.
b) Calcular la probabilidad de que la recaudacin total semanal sea superior a 315 .
c) Calcular la probabilidad de que se recaude ms de lunes a viernes que durante el fin de
semana.
d) Supongamos que las variables son dependientes y su coeficiente de correlacin es
3
2
.
Sabiendo que la recaudacin total bajo el caso de dependencia sigue una distribucin normal,
calcular la esperanza y la varianza, as como la probabilidad de que tome un valor superior a
315 . Compararla con el caso de independencia.
SOLUCIN
a) Para el caso del fin de semana (X
2
), la varianza es superior que la de la primera variable. Esto
induce a pensar que podemos obtener recaudaciones diferentes a la media, ms que con el primer
caso.
b) X
1
+ X
2
) 45 N(325,
p(X
1
+ X
2
> 315) = |
.
|

\
|
=

> 1,49
45
325 315
Z p = 1 ( ) 1,49 = 1 0,0681 = 0,9319.
c) p(X
1
> X
2
) = p(X
1
X
2
> 0). Como X
1
X
2
) 45 25, N( .
P(X
1
> X
2
) = p(X
1
X
2
> 0) = |
.
|

\
|
=

> ,73 3
45
25) ( 0
Z p = 1 ( ) ,73 3 = 1 0,9999 = 0,0001.
d) Como
3
2
Y) Var(X)Var(
Y) Cov(X,

XY
= = Cov(X,Y) = Y) Var(X)Var(
XY
= 6 x 3 x
3
2
= 12
E(X
1
+ X
2
) = 325; Var(X
1
+ X
2
) = Var(X
1
) + Var(X
2
) + 2 Cov(X,Y) = 9 + 36 + 2x12 = 69
Entonces, X
1
+ X
2
) 69 N(325, .
P(X
1
+ X
2
>315) = |
.
|

\
|
=

> 1,20
69
325 315
Z p = 1 ( ) 1,20 = 1 0,1151 = 0,8849.
En este caso, la probabilidad pedida es algo inferior al caso de independencia.
88) El tiempo que tarda un trabajador de un servicio tcnico en realizar cada reparacin,
expresada en das, es una v.a. que sigue una distribucin |
.
|

\
|
2
1
4, .
a) Si suponemos que las tareas que realiza son independientes, calcular la probabilidad de que
tarde menos de medio da en cubrir dos servicios.
Estadstica Empresarial II 133
b) Si se tuvieran que realizar 50 reparaciones, plantear la probabilidad exacta de que el
tiempo empleado en total no fuese superior a los 5 das y buscar mediante alguna
aproximacin su valor, comentando las condiciones que se deben cumplir para ello.
SOLUCIN
Por cada trabajo defino una v.a.: T
i
= Tiempo que se tarda en una reparacin |
.
|

\
|
2
1
4, . Para dos
tareas independientes, el tiempo total T ( ) 4,1 ( ) 4 .
a)
|
.
|

\
|
< +
2
1
T T p
2 1
= dt e 4
0,5
0
t 4


=
2
e 1

= 1 0,1353 = 0,8647.
b) T =

=
50
1 i
i
T ( ) 4,25 .
La probabilidad exacta: 5) p(T < =
( )
dt e t
25
4
t 4 24
5
0
25

. Se resolvera integrando por partes.


Dado que disponemos de 50 v.a.i.i.d., con esperanza y varianza finitas, podemos aplicar el T.C.L.
para resolver esta cuestin.
E(T) =
4
25
; Var(T) =
16
25
1) p(Z 5) p(T
T.C.L.
< < = 0,1587.
89) Sea X = Tiempo que se dedica a realizar una actividad, expresada en horas, que sigue
una distribucin con funcin de densidad:
( )

< < +
=
resto el en 0
2 x 0 1) x
4
1
x f
X
(
a) Calcular E(X) y la Var(X).
b) Calcular la probabilidad de que el tiempo empleado sea superior a hora y media.
c) Si se realizasen 10 actividades del mismo tipo, bajo las mismas condiciones, calcular la
probabilidad de que, exactamente en 3 de ellas el tiempo que se emplea en realizar cada una
sea superior a hora y media.
SOLUCIN
a)E(X) = dx 1) (x
4
1
x
2
0
+

=
6
7
E(X
2
) = dx 1) (x
4
1
x
2
0
2
+

=
3
5
Var(X) = E(X
2
) E
2
(X) =
3
5

2
6
7
|
.
|

\
|
=
36
11
b) p(X > 1,5) = dx 1) (x
4
1
2
5 , 1
+

=
32
11
= 0,34375.
134 Problemas de examen resueltos
c) Y = n actividades entre las 10 en las que el tiempo que se emplea en realizar es mayor que 1,5 h.
Y B(10,
32
11
) P(Y = 3) =
7 3
32
21
32
11
3
10
|
.
|

\
|
|
.
|

\
|
|
|
.
|

\
|
= 0,2555.
90) Sean X e Y dos variables aleatorias continuas con funcin de densidad conjunta:

< < <


=
resto el en 0
4 x y 0
8
1
y) (x, f
Y X,
a) Calcular las funciones de densidad marginales de las variables X e Y.
b) Qu distribucin conocida siguen cada una de las variables X e Y?
c) Son X e Y variables independientes?
d) Calcular las funciones de densidad condicionadas (x) f
y X/Y=
y ) (y f
x Y/X=
.
e) Calcular ( ) 3 Y/X E = .
f) Calcular el coeficiente de correlacin lineal e interpretar su resultado.
g) Calcular la esperanza y la varianza de la variable Z = 3X 2Y + 1.
h) Calcular p(X 2Y > 0).
i) Calcular la funcin de densidad de la variable
8
X
- 2 T= .
SOLUCIN
a)

< <
= =

=
=
resto el en 0
4 x 0
8
x
dy
8
1
(x) f
x y
0 y
X

< <

= =

=
=
resto el en 0
4 y 0
8
y 4
dx
8
1
(y) f
4 x
y x
Y
b) X e(2, 1, 0 ,4) Y e(1, 2, 0 ,4)
c) X e Y no son independientes porque (y) f (x) f y) (x, f
Y X Y X,

d)
4 y 0 que siempre
resto el en 0
4 x y
y 4
1
(y) f
y) (x, f
(x) f
Y
Y X,
y X/Y
< <

< <

= =
=
4 x 0 que siempre
resto el en 0
x y 0
x
1
(x) f
y) (x, f
(y) f
X
Y X,
x Y/X
< <

< <
= =
=
e)

< <
=
=
resto el en 0
3 y 0
3
1
(y) f
3 Y/X
( )
2
3
dy
3
1
y dy (y) f y 3 Y/X E
3 y
0 y
3 y
0 y
3 Y/X
= = = =

=
=
=
=
=
f)
3
8
q p
p
a) (b a E(X) =
+
+ =
3
4
q p
p
a) (b a E(Y) =
+
+ =
Estadstica Empresarial II 135
(Y) Var
9
8
1) q (p q) (p
pq a) (b
(X) Var
2
2
= =
+ + +

=
4 dx dy
8
1
y x dx dy y) (x, f y x E(XY)
4 x
0 x
x y
0 y
4 x
0 x
x y
0 y
Y X,
=
|
|
.
|

\
|
= |
.
|

\
|
=

=
=
=
=
=
=
=
=
Cov(X, Y) = E(XY) E(X) E(Y) = 4 8/3 4/3 = 4/9
2
1
9 / 8 9 / 8
9 / 4
Var(Y) Var(X)
Y) X, ( Cov

Y X,
= = =
g) E(Z) = E(3X 2Y + 1) = 3E(X) 2E(Y) + 1 = 19/3
Var(Z) = Var(3X 2Y + 1) = 9Var (X) + 4Var (Y) 2x3x2Cov (X, Y) = 56/9
h)
2
1
dx dy
8
1
dx dy y) (x, f 0) 2Y X p(
4 x
0 x
x/2 y
0 y
4 x
0 x
x/2 y
0 y
Y X,
=
|
.
|

\
|
= |
.
|

\
|
= >

=
=
=
=
=
=
=
=
j) t)) (8(2 F 1 ) t) 8(2 X p( t) T ( p (t) F
X T
= > = < =

< <
= =
resto el en 0
2 t 3/2 t) (2 8
8) ( t)) (8(2 f (t) f
X T
91) En una gasolinera se hace un estudio sobre dos aspectos de su funcionamiento: el gasto
que realizan sus clientes y el tiempo que se tarda en realizar el pago. El gasto de los clientes
por el llenado del depsito sigue una distribucin normal de media 50 y desviacin tpica 4
, pero los pagos con tarjeta de crdito tienen un coste para la gasolinera de un 3% en
comisin bancaria. Por otra parte, el tiempo en minutos que tarda un cliente en realizar el
pago sigue una distribucin U(1, 3) si lo efecta al contado, y una distribucin (3, 9) si utiliza
tarjeta.
a) Cul es la distribucin de los ingresos de un da en el que se realizan 30 pagos al contado y
100 con tarjeta? Cul es la probabilidad de dichos ingresos sean como mucho 6.400 ?
b) En ese mismo da, cul es la probabilidad de que la persona encargada de la caja haya
estado ocupada cobrando, al menos durante cinco horas y media?
c) Cierto da, solo se realizaron pagos con tarjeta. Con una probabilidad de 0,9772, cuntos
pagos es previsible que se hicieran, si la persona que atiende la caja no estuvo ocupada ms
de cinco horas y veinte minutos?
SOLUCIN
Consideremos las siguientes variables:
E = Ingresos por pagos en efectivo N(50, 4)
T = Ingresos por pagos con tarjeta = 0,97E N(48,5, 3,88)
X = tiempo de un pago en efectivo U(1, 3) E(X) = 2 Var(X) = 1/3
136 Problemas de examen resueltos
Y = tiempo de un pago con tarjeta (3, 9) E(Y) = 3 Var(Y) = 1
a) ( ) 1.985,44 6.350, N T E I
100
1 i
i
30
1 i
i
+ =

= =
( ) 0,8686 1,12
1.985,44
6.350 6.400
6.400) p(I = =
|
|
.
|

\
|

= <
b) ( ) 10 60, N X
L
30
1 i
i

=
( ) 10 300, N Y
L
100
1 i
i

=
( ) 110 360, N Y X T
L
100
1 i
i
30
1 i
i
+ =

= =
( ) ( ) 9979 , 0 86 , 2 1
110
360 330
110
360 T
p 1 0 3 3 T p
T.C.L. o tipificand
= = |
.
|

\
|
<

=
c) ( ) n n, 3 N Y
L
n
1 i
i

=
0,9772
n
3n 320
n
3n 320
n
3n Y
p 320 Y p
T.C.L.
n
1 i
i
o tipificand
n
1 i
i
=
|
|
.
|

\
|

=
|
|
|
|
.
|

\
|

= |
.
|

\
|

=
=
2
n
n 3 0 32
=

3n + 2 n 320 = 0
n = 10 n = 100 n = 10,6667 (esta solucin no vale)
92) Un comercial de una empresa ha estado estudiando los valores de X = Nmero de visitas
que realiza al da, e Y = Nmero de ventas efectuadas al da; obteniendo los siguientes
resultados de la distribucin conjunta:
X \ Y 2 3 4
4 0,08 0,02 0,00
5 0,07 0,07 0,06
6 0,04 0,10 0,16
7 0,01 0,11 0,28
Calcular:
Estadstica Empresarial II 137
a) el nmero medio y la varianza de las visitas y de las ventas efectuadas.
b) son independientes las variables?
c) el coeficiente de correlacin entre ambas variables. Interpretarlo.
d) el nmero medio de ventas al da cuando realiza 5 menos visitas al da.
e) la probabilidad de que haya efectuado como mnimo 5 visitas si ha efectuado 3 ventas.
f) la probabilidad de que en 10 das de trabajo en al menos 6 haya efectuado 4 ventas.
SOLUCIN
X \ Y 2 3 4 P(X = x)
4 0,08 0,02 0,00 0,10
5 0,07 0,07 0,06 0,20
6 0,04 0,10 0,16 0,30
7 0,01 0,11 0,28 0,40
P(Y = y) 0,20 0,30 0,50 1,00
a) E(X) = 6 E(Y) = 3,3 E(X
2
) = 37 E(Y
2
) = 11,5
Var(X) = 1 Var(Y) = 0,61
b) f
X
(4) f
Y
(2) = 0,10 x 0,20 = 0,02 0,08 = f
X,Y
(4, 2) X e Y son dependientes
c) E(XY) = 20,24 Cov(X, Y) = 0,44 (X, Y) = 0,56336227
d)
Y/X 5 2 3 4
P(Y/X 5) 15/30 9/30 6/30
E(Y/X 5) = 2x0,5 + 3x0,3 + 4x0,2 = 2,7
e) p(X 5 / Y = 3) = p(X 5, Y = 3) / p(Y = 3) = 28/30
Sea Z = Nmero de das en los que efecta 4 ventas de los 10 das B(10, 0,5)
p(Z 6) = 1 p(Z < 6) = 1 p(Z 5) = 1 0,6230 = 0,3770.
93) Sea la variable aleatoria X = Porcentaje (en tanto por uno) de las visitas realizadas
mensualmente de las previamente asignadas a un comercial. Se sabe que esta variable sigue
una distribucin (2, 1).
a) Obtener el porcentaje medio de visitas y su varianza.
b) Calcular p(X < 0,1) y p(X > 0,9). Comparando ambas probabilidades, se podra deducir la
idea de que es ms probable que el comercial realice un nmero elevado de visitas?
c) Sabiendo que el salario del comercial se compone de una parte fija de 900 y otra variable
que es proporcional al porcentaje de visitas realizadas con un mximo de 300 , construir la
variable salario mensual del comercial. Demostrar que sigue una distribucin e(2, 1, 900,
1.200). Obtener el salario medio y su varianza.
d) La empresa tiene a su disposicin 150 comerciales en activo, todos ellos con el mismo
sistema salarial. Calcular la probabilidad aproximada de que la cantidad total dedicada a
salarios por la empresa mensualmente sea superior a 168.000 .
138 Problemas de examen resueltos
SOLUCIN
a) X (2, 1) E(X) =
q p
p
+
=
3
2
; Var(X) =
2
q) (p 1) q (p
pq
+ + +
=
18
1
b) ( )

< <
=
resto el en 0
1 x 0 x 2
x f
X
p(X < 0,1) =

0,1
0
dx x 2 = ( |
0,1
0
2
x = 0,01; p(X > 0,9) =

1
0,9
dx x 2 = ( |
1
0,9
2
x = 0,19.
Es ms probable que realice un porcentaje alto visitas.
c) Si X (p, q), aplicando el cambio de variable: Y = (b a)X + a
e
(p, q, a, b).
En nuestro caso, p = 2, q = 1, a = 900 y b = 1.200. Entonces, Y
e
(2, 1, 900, 1.200).
E(Y) = (b a) E(X) + a = 300
3
2
+ 900 = 1100 Var(Y) = (b a)
2
Var(X) = 300
2

18
1
= 5000
d) Defino Y
1
, , Y
150
, v.a.i.i.d., siendo Y
i
= salario mensual del i-simo comercial.
E(Y
i
) =1100; Var(Y
i
) = 5000. Entonces,

=
150
1 i
i
Y = Salario total mensual.
Su esperanza: |
.
|

\
|

=
150
1 i
i
Y E = 150 x E(Y
i
) = 150 x 1100 = 165000
Su varianza: |
.
|

\
|

=
150
1 i
i
Y Var = 150 x Var(Y
i
) = 150 x 5000 = 750000
El problema nos pregunta sobre |
.
|

\
|
>

=
150
1 i
i
168000 Y p . Como el nmero de variables implicadas es
superior a 30, podemos aplicar el teorema central del lmite para resolver esta cuestin. Entonces:
|
.
|

\
|
>

=
150
1 i
i
168000 Y p
. . . L C T

|
|
.
|

\
|
>
750000
165000 168000
Z p = ( ) 46 , 3 Z p > = 1 - ( ) 46 , 3 Z p =
= 1 0,9997 = 0,0003.
La probabilidad de que ocurra es muy baja.
Estadstica Empresarial II 139
94) Sea (X, Y) una variable aleatoria bidimensional con funcin de densidad conjunta:

< < < <


=
resto el en 0
2 y 0 1; x 0 xy 1
y) (x, f
Y X,
a) Obtener las funciones de densidad marginales de X e Y, e indicar cul es su distribucin y
si coincide con alguna de las conocidas. Son X e Y independientes?
b) Calcular Cov(X, Y), sabiendo que Var(3X + 2Y) = 2.
c) Calcular |
.
|

\
|
<
>
0,5 X
1 Y
p .
SOLUCIN
a) 1 x 0 x) 2(1 2x 2
2
xy
y xy)dy (1 (x) f
2 y
0 y
2
2 y
0 y
X
< < = =
(

\
|
= =
=
=
=
=

X (1, 2)
2 y 0 y) (2
2
1
2
y
1
2
yx
x xy)dx (1 (y) f
1 x
0 x
2
1 x
0 x
Y
< < = =
(

\
|
= =
=
=
=
=

Y e(1, 2, 0, 2)
Las variables no son independientes puesto que:
(y) (x)f f ) y (2
2
1
x) 2(1 xy 1 y) (x, f
Y X Y X,
= =
b) Puesto que sabemos las distribuciones de X e Y:
18
1
q) 1)(p q (p
pq
Var(X)
2
=
+ + +
= ;
9
2
q) 1)(p q (p
pq a) (b
Var(Y)
2
2
=
+ + +

=
2 Y) Cov(X, 2 3 2 Var(Y) 2 Var(X) 3 2Y) Var(3X
2 2
= + + = +

216
11
12
9
2
2
18
1
3 2
Y) Cov(X,
2 2
=

=
c) |
.
|

\
|
<
>
0,5 X
1 Y
p =
0,5) p(X
0,5) X 1, p(Y
<
< >
( ) ( ) =
(

= = < >

=
=
=
=
=
=
=
=
0,5 x
0 x
2 y
1 y
0,5 x
0 x
2 y
1 y
dx dy xy 1 dy dx xy 1 0,5) X 1, p(Y

16
5
2
x
2
3
x dx
2
3x
1 dx
2
xy
y
0,5 x
0 x
2
5 , 0
0 x
5 , 0
0
2 y
1 y
2
=
(

\
|
=
|
.
|

\
|
=
(

\
|
=
=
=
=
=
=
=
=
=

x x
x
140 Problemas de examen resueltos
4
3
2
x
x 2 x)dx 2(1 0,5) p(X
0,5 x
0 x
2
0,5 x
0 x
=
(

\
|
|
|
.
|

\
|
= = <
=
=
=
=

|
.
|

\
|
<
>
0,5 X
1 Y
p =
0,5) p(X
0,5) X 1, p(Y
<
< >
= 0,4167
12
5
4
3
16
5
= =
95) En una sala de cine con capacidad para 100 personas, se sabe que el gasto en otros
productos (palomitas, chuches, refrescos, etc.) por cada persona que entra al cine es una
variable aleatoria que sigue una distribucin normal de media 6 euros y que el 98,3% de las
personas que entran en el cine se gastan en otros productos ms de 1,76 euros cada una.
a) Calcular la probabilidad de que el gasto total en otros productos de todas las personas que
entran en el cine un da que se llena la sala, est comprendido entre 550 y 620 euros.
b) Las 10 ltimas sesiones se ha llenado la sala. Cul es la probabilidad de que en ms de
ocho sesiones el gasto en otros productos estuviera comprendido entre 550 y 620 euros?
SOLUCIN
Sea X = gasto en otros productos por cada persona que entra al cine N(6, )
0,983 )

4,24
p(Z 1 )

6 1,76

6 X
( p 1,76) (X p =

>

= >
0,0170 0 0,983 1 )

4,24
p(Z = =

2,12

4,24
=

= 2 XN(6, 2)
a) Sea

=
=
100
1 i
i
X Y = gasto total en otros productos de todas las personas que entran en el cine
cuando se llena la sala N(600, 20)
0,8351 0,0062 0,8413 2,5) ( (1)
20
600 620

20
600 Y
20
600 550
p 620) Y p(550 = = =
|
.
|

\
|

=
b) Sea W = nmero de sesiones en las que el gasto en otros productos estuvo entre 550 y 620 euros
W B(10; 0,8351)
P(W > 8) = p(W = 9) +p(W = 10) =
0 10 1 9
0,1649 0,8351
10
10
0,1649 0,8351
9
10
|
|
.
|

\
|
+
|
|
.
|

\
|
= 0,490699288.
Estadstica Empresarial II 141
96) En una subasta donde el lote se compone de un nico artculo, sea X = la cantidad
ofrecida por el artculo (en cientos de euros) una variable aleatoria con funcin de densidad:
f
X
(x) =

< <
resto 0
20 x 8
12
1
a) Comenta el comportamiento del modelo probabilstico de la cantidad ofertada.
b) Calcular la probabilidad de que la oferta por un artculo no supere los 1.600.
c) Una empresa desea adquirir 60 objetos para decorar una de sus salas de exposicin. Cul
ser la probabilidad de que no gaste ms de 85.000?
d) Si el presupuesto fuese de 100.000, cuntos objetos podra adquirir como mximo, con
unas garantas de, al menos, un 99,65%?
SOLUCIN
a) X U(8, 20)
b) p(X < 16) = dx
12
1
16 x
8 x

=
=
= 0,6667.
c) X
1
, , X
100
v.a.i.i.d., E(X
i
) = 14 y Var(X
i
) = 12
720 12 60 X Var ; 840 14 60 X E
60
1 i
i
60
1 i
i
= =
|
|
.
|

\
|
= =
|
|
.
|

\
|

= =
p(

=
60
1 i
i
X 850) = p(
720
840 850
720
840 X
60
1 i
i

=
)
TCL
p( 37 , 0 Z ) = 0,6443
d) X
1
, , X
n
v.a.i.i.d., 12 n X Var ; 14 n X E
n
1 i
i
n
1 i
i
=
|
|
.
|

\
|
=
|
|
.
|

\
|

= =
p(

=
n
1 i
i
X 1.000) 0,9965 p(
12n
14n 1.000
12n
14n X
n
1 i
i

=
) 0,9965
Aplicando el Teorema Central del Lmite,
12n
14n 1.000
= 2,70 14n + 9,3531 n 1000 = 0.
Las dos soluciones para n son 8,1241 y 8,7922. La segunda no tiene sentido, por tanto, n = 66.
142 Problemas de examen resueltos
97) Sea (X, Y) una v.a. bidimensional con funcin de densidad conjunta:

> < <


=
resto el en 0
0 y 1; x 0 e x 2
y) (x, f
-y
Y X,
a) Obtener las funciones de densidad marginales de X e Y. Su distribucin es conocida? Son
X e Y independientes?
b) Calcular la esperanza y desviacin tpica de la variable Z = 3X 5Y + 3.
c) Calcular |
.
|

\
|
<
>
0,5 X
1 Y
p .
SOLUCIN
a) ( | 1 x 0 2x 2xe dy 2xe (x) f
y
0 y
y
y
0 y
y
X
< < = = =
=
=

=
=

X (2,1)
( | 0 y e x e dx 2xe (y) f
y
1 x
0 x
2 y
1 x
0 x
y
Y
> = = =

=
=

=
=

Y (1)
Las variables son independientes puesto que:
(y) (x)f f 2xe 2xe y) (x, f
Y X
y y
Y X,
= = =

b) Puesto que conocemos la distribucin:
E(X) =
3
2
q p
p
=
+
; E(Y)= 1
1
1
a
1
= =
18
1
q) 1)(p q (p
pq
Var(X)
2
=
+ + +
= ; Var(Y) = 1
1
1
a
1
1 2
= =
y puesto que son independientes la covarianza es cero
= + = + 3 5E(Y) 3E(X) 3) 5Y E(3X 0 3 5x1
3
2
3x = +
( )
2
51
25x1
18
1
9x Var(Y) 5 Var(X) 3 5Y) Var(3X 3) 5Y Var(3X
2 2
= + = + = = +
de donde la desviacin tpica ser: 5,04975 3) 5Y Var(3X = +
c) |
.
|

\
|
<
>
0,5 X
1 Y
p =
0,5) p(X
0,5) X 1, p(Y
<
< >
=
(

= = < >

=
=
=
=

=
=
=
=

0,5 x
0 x
y
1 y
y
0,5 x
0 x
y
1 y
y
dx dy 2xe dy dx 2xe 0,5) X 1, p(Y
( | ( |
1
0,5 x
0 x
2 1
0,5 x
0 x
1
0,5 x
0 x
y
1 y
y
e
4
1
x e dx 2xe dx 2xe

=
=

=
=

=
=
=
=

= = = =

Estadstica Empresarial II 143
( |
4
1
x 2xdx 0,5) p(X
0,5 x
0 x
2
0,5 x
0 x
= = = <
=
=
=
=

|
.
|

\
|
<
>
0,5 X
1 Y
p =
0,5) p(X
0,5) X 1, p(Y
<
< >
=
1
1
e
4
1
e
4
1

= = 0,367879 = p(Y>1) por ser independientes.


98) Las ventas diarias de una administracin de lotera (en miles de euros) es una variable
aleatoria que sigue una distribucin normal con media 5 euros y varianza 4.
a) Si se consideran los meses de 30 das cul es la probabilidad de que en siete meses y medio,
las ventas en esa administracin de lotera estn comprendidas entre 1.122.000 euros y
1.125.300 euros?
b) Cul sera el nmero mximo de das para que las ventas de esa administracin no
superen los 56.760 euros con una probabilidad del 97,5%?
SOLUCIN
a) Sea X = Ventas diarias de la administracin en miles de euros N(5, 2)
Siete meses y medio son 225 das (7 x 30 + 15 = 225)
Sea

=
=
225
1 i
i
X Y = Ventas en siete meses y medio de la administracin en miles de euros
Y N(225x5, 225 x2) N(1.125, 30)
=
|
.
|

\
|

=
30
1.125 1.125,3

30
1.125 Y
30
1.125 1.122
p 1.125,3) Y p(1.122 0,0438 0,1) ( (0,01) =
b) Sea V =

=
n
1 i
i
X = Ventas en n das de la administracin en miles de euros N(5n, 2 n )
0,975 )
n 2
5n 56,76
p(Z
n 2
5n 56,76
n 2
5n V
p 56,76) V p( =

= |
.
|

\
|

=
1,96
n 2
5n 56,76
=

5n + 3,92 n 56,76 = 0 n = 3 y n = -3,784


La solucin negativa no tiene sentido, por tanto, n = 9
144 Problemas de examen resueltos
99) Una empresa que reparte a domicilio pedidos de comida rpida estima que el tiempo (en
minutos) que dedica a cubrir cada servicio sigue una distribucin gamma de parmetros a =
0,25 y p = 4. Por otra parte, el ingreso medio por pedido es 24, con una desviacin tpica de 2.
a) Indicar cul es la distribucin de la variable tiempo total dedicado a realizar 100 servicios,
plantear la probabilidad de que en esos 100 servicios el tiempo total supere las 30 horas y
calcularla de forma aproximada.
b) Si la compaa ha de servir 100 pedidos, obtener la probabilidad aproximada de que el
ingreso total que generan sea superior a 2.390.
SOLUCIN
a) Sea X = Tiempo en minutos que se dedica a cubrir un servicio (0,25; 4)
T =

=
100
1 i
i
X = Tiempo total dedicado a realizar 100 servicios (0,25; 400)
P(

=
100
1 i
i
X > 1.800) =

=
=

x
1.800 x
0,25x 399
400
dx e x
399!
0,25
6.400 64 100 X Var 1.600; 16 100 X E
100
1 i
i
100
1 i
i
= =
|
|
.
|

\
|
= =
|
|
.
|

\
|

= =
P(

=
100
1 i
i
X > 1.800) = p(
80
1.600 1.800
80
1.600 X
100
1 i
i

>

=
)
TCL
p( 2,5 Z > ) = 1 (2,5) = 0,0062.
b) Sea Y = Ingreso por pedido Y
1
, , Y
100
v.a.i.i.d., E(Y
i
) = 24 y Var(Y
i
) = 4.
400 4 100 Y Var 2.400; 24 100 Y E
100
1 i
i
100
1 i
i
= =
|
|
.
|

\
|
= =
|
|
.
|

\
|

= =
p(

=
100
1 i
i
Y > 2.390) = p(
20
2.400 2.390
20
2.400 Y
100
1 i
i

>

=
)
TCL
p(Z > 0,5) = 1 (0,5) = 0,6915.

Anda mungkin juga menyukai